■ちょっとした物理の質問はここに書いてね160■

このエントリーをはてなブックマークに追加
1ご冗談でしょう?名無しさん
前スレ
■ちょっとした物理の質問はここに書いてね159■
http://uni.2ch.net/test/read.cgi/sci/1333607768/

★荒らし厳禁、煽りは黙殺
★書き込む前に>>2の注意事項を読んでね
★数式の書き方(参考)はこちら>>3-5(予備リンク:>>2-10

===質問者へ===
重要 【 丸 投 げ 禁 止 】
丸投げは専用スレ「丸投げしたい問題を書くスレ」に
http://uni.2ch.net/test/read.cgi/sci/1284096345/

・質問する前に
1. 教科書や参考書をよく読む
2. http://www.google.com/ などの検索サイトを利用し、各自で調べる
3. 学生は自分の学年、物理科目の履修具合を書く
4. 宿題を聞くときは、どこまでやってみてどこが分からないのかを書く
5. 投稿する前に、ちゃんと質問が意味の通る日本語か推敲する、曖昧な質問文には曖昧な回答しか返せない
・「力」「エネルギー」「仕事」のような単語は物理では意味がはっきり定義された言葉です、むやみに使うと混乱の元
・質問に対する回答には返答してね、感謝だけでなく「分からん」とかダメ出しでもOK
・質問するときはage&ID表示推奨
・高度すぎる質問には住人は回答できないかもしれないけれど、了承の上での質問なら大歓迎

===回答者へ===
・丸投げは専用スレに誘導
・不快な質問は無視、構った方が負け
・質問者の理解度に応じた適切な回答をよろしく
・単発質問スレを発見したらこのスレッドへの誘導をよろしくね
・逆に議論が深まりそうなら新スレ立てて移動するのもあり
・板違いの質問は適切な板に誘導を
・不適切な回答は適宜訂正、名回答は素直に賞賛
2ご冗談でしょう?名無しさん:2012/05/14(月) 19:14:40.19 ID:sSJdr825
書き込む際の注意

【重要】最近このスレは荒らしが常駐してるので、常駐してる回答者はNGワード設定してることがあります。
スレの流れからNGワードを察知して、なるべくその単語を使わないように質問すると返答されやすいです。

以下のような質問に物理板住人は飽き飽きしているので無視されます。しないでください。
まともな質問が流れるので回答・相手もしないでください。
「相対性理論は間違っています」「量子力学は間違っています」
「宇宙論は間違っています」「シュレディンガーの猫は変です」
「永久機関を作りました」「タイムマシンについて教えて」

「どうして〜?・なぜ〜?」:物理で答えられる問題とは限りません。
質問によっては哲学板・雑談系板へ誘導されるかも。
意図的になされた物理と関係ない質問:スルーの方向で

参考サイト
Wikipedia http://ja.wikipedia.org/
物理のかぎしっぽ http://hooktail.sub.jp/
EMANの物理学 http://homepage2.nifty.com/eman/
ときわ台学 http://www.f-denshi.com/
以上のサイトの説明はすべてが正確なわけではありません。
このスレでの受け答えもそうですが。相互に補完しつつ精度を高めましょう。
3ご冗談でしょう?名無しさん:2012/05/14(月) 19:14:44.20 ID:???
海外旅行にでも行ったのかと思ったのは俺だけか
4ご冗談でしょう?名無しさん:2012/05/14(月) 19:16:52.58 ID:???
●微分・偏微分: dy/dx=y', ∂y/∂x=y_x ("∂"は「きごう」で変換可)
●ベクトル微分: ∇f=grad f, ∇・A=div A,∇xA=rot A, (∇^2)f=Δf ("∇"は「きごう」,"Δ"は「でるた」で変換可.)
●積分: ∫[0,1] f(x)dx = F(x)|_[x=0,1], ∫[y=0,x] f(x,y)dy, ∬[D] f(x,y)dxdy, ∬[C] f(r)dl 
("∫"は「いんてぐらる」,"∬"は「きごう」で変換可)
●数列和・数列積: Σ[k=1,n] a(k), Π[k=1,n] a(k) ("Σ"は「しぐま」,"Π"は「ぱい」で変換可)
文脈によっては単に同じ添字が2回出てきただけで a_i b_i = Σ[i] a_i b_i と積の総和をとることも(Einsteinの縮約)
●極限: lim[x→∞] f(x) ("∞"は「むげんだい」で変換可)
●確率・期待値:P(x), <x>=E(x)
●論理・集合: "⇔⇒∀∃∧∨¬∈∋⊆⊇⊂⊃∪∩"は「きごう」で変換
●等号・不等号: "≠≒<>≦≧≪≫"は「きごう」で変換

読みやすい書き方の例:∫[-∞,∞] exp{ -Σ[i,j=1,n] A_[i,j](x_i)(x_j) } dx = √(π^n/det A)

読みにくい書き方の例:∫[-∞,∞]exp(-Σ[i,j=1,n]A_[i,j]x_ix_j)dx=√(π^n/det A)
5ご冗談でしょう?名無しさん:2012/05/14(月) 19:19:04.41 ID:???
数式の書き方の例 ※適切にスペースを入れると読みやすくなります
●括弧: (), [], {}を適切に入れ子にして分かりやすく書く
●スカラー: a,b,...,z, A,...,Z, α,β,...,ω, Α,Β,...,Ω,...(「ぎりしゃ」「あるふぁ〜おめが」で変換)
●ベクトル: V=(v1,v2,...), |V>,V↑, (混乱しないならスカラーの記号でいい。通常は縦ベクトル)
●テンソル: T^[i,j,k...]_[p,q,r,...], T[i,j,k,...; p,q,r,...]  (上下付き1成分表示)
●行列: M[i,j], I[i,j]=δ_[i,j] M = [[M[1,1],M[2,1],...], [M[1,2],M[2,2],...],...], I = [[1,0,0,...],[0,1,0,...],...]
 (右は全成分表示。行または列ごとに表示する。例:M=[[1,-1],[3,2]])
●対角行列: diag(a,b) = [[a,0],[0,b]]
●転置行列・随伴行列:M^T, M†("†"は「だがー」で変換可) ●行列式・トレース:|A|=det(A), tr(A)
●複号: a±b("±"は「きごう」で変換可)
●内積・外積: a・b, a×b
●関数・汎関数・数列: f(x), F[x(t)] {a_n}
●平方根: √(a+b) = (a+b)^(1/2) = sqrt(a+b) ("√"は「るーと」で変換可)
●指数関数・対数関数: exp(x+y)=e^(x+y) ln(x)=log_e(x) (底を省略して単にlogと書いたとき多くは自然対数)
 括弧を省略しても意味が容易に分かるときは省略可: sin(x) = sin x
●三角関数、逆三角関数、双曲線関数: sin(a), cos(x+y), tan(x/2), asin(x)=sin^[-1](x), cosh(x)=[e^x+e^(-x)]/2
●絶対値:|x| ●ノルム:||x|| ●共役複素数:z^* = conj(z)
●階乗:n!=n*(n-1)*(n-2)*...*2*1, n!!=n*(n-2)*(n-4)*...
6ご冗談でしょう?名無しさん:2012/05/14(月) 19:21:17.02 ID:???
質問・回答に標準的に用いられる変数の例

a:加速度、昇降演算子 A:振幅、ベクトルポテンシャル B:磁束密度 c:光速 C:定数、熱・電気容量
d:次元、深さ D:領域、電束密度 e:自然対数の底、素電荷 E:エネルギー、電場
f:周波数 f,F:力 F:Helmholtzエネルギー g:重力加速度、伝導度
G:万有引力定数、Gibbsエネルギー、重心 h:高さ、Planck定数 H:エンタルピー、Hamiltonian、磁場
i:虚数単位 i,j,k,l,m:整数のインデックス I:電流、慣性モーメント j:電流密度・流束密度
J:グランドポテンシャル、一般の角運動量 k:バネ定数、波数、Boltzmann定数 K:運動エネルギー
l,L:長さ L:Lagrangian、角運動量、インダクタンス m,M:質量 n:物質量 N:個数、トルク
M:磁化 O:原点 p:双極子モーメント p,P:運動量、圧力 P:分極、仕事率、確率 q:波数
q,Q:一般化座標、電荷 Q:熱 r:距離 R:抵抗、気体定数 s:スピン S:エントロピー、面積 t,T:時間 T:温度
U:ポテンシャル、内部エネルギー v:速度 V:体積、ポテンシャル、電位
W:仕事、状態数 x,y,z:変数、位置 z:複素変数 Z:分配関数

β:逆温度 γ:抵抗係数 Γ:ガンマ関数 δ:微小変化 Δ:変化 ε:微小量、誘電率 θ:角度 κ:熱伝導率
λ:波長、固有値 μ:換算質量、化学ポテンシャル、透磁率 ν:周波数 Ξ:大分配関数 π:円周率 ρ:(電荷)密度、抵抗率
σ:スピン τ:固有時 φ:角度、ポテンシャル、波動関数 ψ:波動関数 ω:角振動数 Ω:状態密度
7ご冗談でしょう?名無しさん:2012/05/14(月) 19:23:29.43 ID:???
>>4>>5は順番が逆でした。
>>3スレ立て中の割り込みはご遠慮ください・・・。
というかこのスレ消そうか?
8ご冗談でしょう?名無しさん:2012/05/14(月) 21:05:01.20 ID:???
>>1
別にどうでもいいと思うけどなぁ。
どうしても>>3が気に食わないのなら止めないけど。
9ご冗談でしょう?名無しさん:2012/05/14(月) 21:31:10.04 ID:???
間が空いたんでオレも疑問を持ったからな。
代弁としか思わん。
10ご冗談でしょう?名無しさん:2012/05/14(月) 21:36:36.27 ID:???
どうせ今回も
11ご冗談でしょう?名無しさん:2012/05/15(火) 00:04:38.31 ID:???
メコスジの気配
12 忍法帖【Lv=3,xxxP】 :2012/05/15(火) 04:46:34.12 ID:???
物理学の達人は神様にも勝てるですよ。
13ご冗談でしょう?名無しさん:2012/05/15(火) 08:59:19.96 ID:0LK6H4QC
光電効果ってつまり光子に質量があるんですね?どうなんですかと。
14ご冗談でしょう?名無しさん:2012/05/15(火) 09:11:05.54 ID:???
は?
15ご冗談でしょう?名無しさん:2012/05/15(火) 09:18:21.15 ID:???
質問です
速度を求める問題ですが
Fの力と-kvの抵抗を受けながら進む物体について
運動方程式:ma=F-kv
という微分方程式に
V=初速度+at
という高校の頃の公式を当てはめて解いてはいけない理由がわかりません
なにがいけないのでしょうか
16ご冗談でしょう?名無しさん:2012/05/15(火) 09:35:54.09 ID:???
>メコスジの気配
    /\___/ヽ
   //~    ~\:::::\
  . |  (・)   (・)   .:|
  |   ,,ノ(、_, )ヽ、,, .::::|   は?
.   |   `-=ニ=- ' .:::::::|
   \  `ニニ´  .:::::/
   /`ー‐--‐‐―´\
17ご冗談でしょう?名無しさん:2012/05/15(火) 11:20:51.92 ID:???
>>13
光のエネルギーが不連続ということ。
>>15
Vを微分してaになるか?
18ご冗談でしょう?名無しさん:2012/05/15(火) 12:13:52.69 ID:???
>>15
>V=初速度+at
これは、等加速度運動の時の式
>運動方程式:ma=F-kv
この運動はvが増えると加速度が減るので
等加速度運動の式は使えない
ちなみに初速度0からvが増加してv=F/kになると
加速度が0になりその後は等速度運動になる
この v=F/k を終端速度という
試験に出るぞw
19ご冗談でしょう?名無しさん:2012/05/15(火) 12:31:54.76 ID:t99eK3oc
宇宙空間で太陽光が届かないところに30℃の鉄があったとします。
放置しておくと鉄の温度は変わりますか?どれぐらいになりますか?

私の考えでは接する粒子が無いせいで運動エネルギーが外に出ていかないのではと思うので、
温度が変わらないのかなと思うのですが、
月面は寒いものとして映画などで描かれてますね。
20ご冗談でしょう?名無しさん:2012/05/15(火) 12:58:55.85 ID:???
>>13
質量はないよ。
光=電磁波から電子がエネルギーを受けとるときは
電磁波の振動数に比例するエネルギーE=hνに決まっている。
これって、光がエネルギーhνを持った粒みたいだから
アインシュタインは光量子と呼んだ。今の光子のこと。
21ご冗談でしょう?名無しさん:2012/05/15(火) 13:01:23.85 ID:???
>>19
熱の伝わり方は、伝導・対流・輻射だよ。
輻射には熱媒体は不要。
22ご冗談でしょう?名無しさん:2012/05/15(火) 13:12:11.56 ID:t99eK3oc
http://ja.wikipedia.org/wiki/%E7%86%B1%E6%94%BE%E5%B0%84
>地球が太陽から熱を得ているのは熱放射の例である。
太陽光によって熱を得ているんじゃないんですか?
光も電磁波なんですか?
23ご冗談でしょう?名無しさん:2012/05/15(火) 13:18:39.23 ID:???
光は電磁波だよ
24ご冗談でしょう?名無しさん:2012/05/15(火) 15:24:41.79 ID:???
>>22
光=電磁波
可視光の事を言ってるなら可視光は電磁波の一種
25ご冗談でしょう?名無しさん:2012/05/15(火) 17:00:04.69 ID:???
>>19
熱輻射で冷えて最終的には平衡状態になる。宇宙線や星からの電磁波などはわずかだろうから、
宇宙マイクロ波背景輻射の2.7K程度まで冷める。
26ご冗談でしょう?名無しさん:2012/05/15(火) 21:52:28.13 ID:???
白色と銀色の違いを調べて来いといわれたのですが
金属光沢の有無の違いしか分かりませんでした

他にも何かあるのでしょうか?高校レベルでお願いします
27ご冗談でしょう?名無しさん:2012/05/15(火) 21:53:33.11 ID:???
見たまんま
28ご冗談でしょう?名無しさん:2012/05/15(火) 22:25:55.03 ID:???
>>26
電気を通すか通さないか。つまり自由電子があるかないか。
導体と誘電体と言ってもよい。
29ご冗談でしょう?名無しさん:2012/05/15(火) 22:34:04.87 ID:???
Silver is the metallic shade resembling gray, closest to that of polished silver.

The visual sensation usually associated with the metal silver is its metallic shine. This cannot
be reproduced by a simple solid color, because the shiny effect is due to the material's
brightness varying with the surface angle to the light source. In addition, there is no
mechanism for showing metallic or fluorescent colors on a computer without resorting to
rendering software which simulates the action of light on a shiny surface. Consequently in art
one would normally use a metallic paint that glitters like real silver. A matte grey color like the
swatch on this page would not be considered silver.
30ご冗談でしょう?名無しさん:2012/05/15(火) 22:45:48.89 ID:???
>>29
なるほど、白色はもともと無色透明なものの粉(塩など)や不均質な物質(牛乳)で
あって、銀色は鏡面ではあるが細かい凸凹があるわけだ。26の趣旨はどこにあるのだろう。
31ご冗談でしょう?名無しさん:2012/05/15(火) 23:44:35.11 ID:???
>>26
白色も銀色も可視光全体の反射率が高い。色がついてないということ。
で、銀色は鏡面反射で、白色は乱反射。
滑らかな金属の表面は銀色だけど、表面が滑らかでないと白っぽく見える。
アルミホイルの表と裏の違い。
あと金属の粉末が白っぽく見えるのも、乱反射になっているから。
32ご冗談でしょう?名無しさん:2012/05/16(水) 02:47:41.81 ID:???
白は透明。
金属の細かい粉は黒。
33ご冗談でしょう?名無しさん:2012/05/16(水) 05:47:21.94 ID:???
よく固体・液体・気体の説明で気体の粒子が飛び交ってる図があるけど、
 気体 = 粒子が飛び交ってる = 高エネルギー
というイメージがある。すべての気体が高エネルギーなわけがない
単に粒子間力が弱くて流されやすい(常に流動している)ってだけだよね?
きっと勘違いしてる人、多いんじゃないかな

参考図↓
http://www.geocities.jp/don_guri131/tyuugakujoutaihennka.html
http://www2.yamamura.ac.jp/chemistry/chapter2/lecture1/lect2011.html
http://chigaku.ed.gifu-u.ac.jp/chigakuhp/html/kyo/busshitsu/nacl/intro-v112.html
http://ja.wikipedia.org/wiki/%E6%B0%97%E4%BD%93
34ご冗談でしょう?名無しさん:2012/05/16(水) 05:52:14.44 ID:???
と思ったが、よく考えたら
 液体 ⇒ 気体
になる過程では気体は高エネルギーなのか
充分拡散してエネルギーが減少すると安定化するのか
自己解決
35前スレ888:2012/05/16(水) 18:24:40.44 ID:???
36ご冗談でしょう?名無しさん:2012/05/16(水) 21:37:24.17 ID:???
>>35
興味ある動画だ。慣性で回っているのかな。ストローと氷はすべっているように見える。
氷が止まっているところからはじまっていないのが残念だ。
37前スレ888:2012/05/16(水) 22:12:09.76 ID:???
>>36
悪い途中からの動画しか無いんだ
最初はストローと同じ向きに一瞬振れて逆回転してたかな
でも最初から逆回転したりもしてた
38ご冗談でしょう?名無しさん:2012/05/16(水) 22:24:54.53 ID:DNHFiutr
質問です
高校2年生です
電気の分野の問題でわからない問題があります。
問題の内容は「無限に長い直線上に連続的に分布した電荷がつくる電場を求めよ」です
式計算ではなく、文章で証明しなさいと言われたのですがまったくわかりません
協力お願いします。
39ご冗談でしょう?名無しさん:2012/05/16(水) 22:39:21.42 ID:???
一様分布か?
40ご冗談でしょう?名無しさん:2012/05/16(水) 22:46:07.20 ID:DNHFiutr
>>39
そうです。
考えてみても全然わからなくて・・・

ご教授お願いします
41ご冗談でしょう?名無しさん:2012/05/16(水) 22:59:06.57 ID:???
>>35
ちゃっかりストローで押してるように見えたけど違うのんか
42ご冗談でしょう?名無しさん:2012/05/16(水) 23:18:49.86 ID:???
>>41
自分で動画見た時にそう言われると思ったがそうでないと言いたい
言い訳っぽく聞こえるかもだがたまたまそう見えるだけ
43ご冗談でしょう?名無しさん:2012/05/16(水) 23:19:57.25 ID:???
動画撮影のため片手でストローいじってたが両手でブレないようにしても同じ結果だった
44ご冗談でしょう?名無しさん:2012/05/16(水) 23:46:04.28 ID:???
なんとなく表面張力が原因な気がする
45ご冗談でしょう?名無しさん:2012/05/16(水) 23:50:11.23 ID:???
>>35
動画サンクス
ほう、そういう感じなのか!
思ってたより面白いなこれ
遊星歯車機構なら説明がつくが肝心の遊星歯車は何で実現されているのだろうか。。にわかには思いつかない
46ご冗談でしょう?名無しさん:2012/05/16(水) 23:51:23.64 ID:???
作用・反作用でないの
47ご冗談でしょう?名無しさん:2012/05/16(水) 23:54:26.56 ID:???
>>38
電気力線が放射状に出てくる。電場の強さは電気力線の密度であることを利用する。
48ご冗談でしょう?名無しさん:2012/05/16(水) 23:57:31.77 ID:???
>>46
??
49ご冗談でしょう?名無しさん:2012/05/16(水) 23:59:39.25 ID:???
>>48
馬鹿か
50ご冗談でしょう?名無しさん:2012/05/16(水) 23:59:46.40 ID:???
>>46
支店力店作用店だよバカ
51ご冗談でしょう?名無しさん:2012/05/17(木) 00:01:22.79 ID:???
>>50
なんで馬鹿
52ご冗談でしょう?名無しさん:2012/05/17(木) 00:03:59.42 ID:???
>>49
いやいや、力の伝達なのに、作用・反作用以外に何があるってんだよって意味の??
53ご冗談でしょう?名無しさん:2012/05/17(木) 00:10:45.65 ID:???
トリックならば、ストローの中に針金を通し、先から針金を出して、曲げて氷が
おけるようにする。そして、針金とストローは逆向きに回転させる。
54ご冗談でしょう?名無しさん:2012/05/17(木) 00:21:00.93 ID:???
面白いなー今度やってみよう
どんな説明が付けられるかな?
55 忍法帖【Lv=18,xxxPT】 :2012/05/17(木) 00:48:52.98 ID:???
ビックリマンで物理学が解りますか?。
56前スレ888:2012/05/17(木) 00:54:20.66 ID:???
誰かマジ解説できる奴いない?
57ご冗談でしょう?名無しさん:2012/05/17(木) 00:56:37.71 ID:???
>>56
水が増えると現象はなくなる?
58 忍法帖【Lv=40,xxxPT】 :2012/05/17(木) 03:01:37.40 ID:???
水の性質が変化すると大丈夫だよ!?♪。
59前スレ888:2012/05/17(木) 12:22:20.88 ID:???
>>57
確認してないな
60ご冗談でしょう?名無しさん:2012/05/17(木) 15:35:48.23 ID:???
>>35
なるほど面白いな。遊星歯車なのかな?やっぱり。
物理よりも工学系のところで訊いた方がいいかも知れない。
実用的な分野で「困った現象」として把握されてるかも。
61ご冗談でしょう?名無しさん:2012/05/17(木) 16:30:08.49 ID:04CkTkCT
ほかのに比べたら簡単な問題だけど・・・
質量5kgの物体が、傾角30度の斜面を滑り降りている。
動摩擦係数が0,2のとき、斜面と平行物体に作用する力を求めよ。

↑の問題がわからぬ。
f'=μ×Rより
=0,2×W×cos30°
=0,2×5×9,8×cos30°
=8,49
になると思うんだが、答えは16Nなんだ。
どこが間違ってるか教えろください。
明日、テストだから焦ってます
62ご冗談でしょう?名無しさん:2012/05/17(木) 16:31:23.97 ID:???
まあ水でも頭からかぶれ
63前スレ888:2012/05/17(木) 17:32:19.21 ID:???
mg・sin(30゚)-μmg・cos(30゚)
上の式を理解できればおK
64ご冗談でしょう?名無しさん:2012/05/17(木) 17:38:14.79 ID:04CkTkCT
>>63
W・sin30°-μ・W・cos30°
つまりP-Rってことでおk?
(Pはfと反対のやつ)
65前スレ888:2012/05/17(木) 17:40:12.97 ID:???
遊星歯車ググったけどこれ中心の歯車と回りの歯車同じ方向に回らないか?
66ご冗談でしょう?名無しさん:2012/05/17(木) 17:58:40.71 ID:???
>>65 Wikiの「遊星歯車機構」を見た?
67ご冗談でしょう?名無しさん:2012/05/17(木) 18:03:55.29 ID:???
前スレでも書いたけど、間に挟まる遊星歯車が自由に動けると中心と外側は同じ向きに回っちゃうけど、
遊星歯車が固定されてその場で回ると中心と外側は逆向きに回る。
こんな感じ
http://upload.wikimedia.org/wikipedia/commons/7/74/Planeetwielmechanisme.gif
件の動画そっくりにはなるけど、どうやってこれが実現されうるのかはわからん
68 忍法帖【Lv=6,xxxP】 :2012/05/17(木) 20:29:13.38 ID:???
別に不動遊星だよ。
69ご冗談でしょう?名無しさん:2012/05/17(木) 23:03:57.55 ID:I1Lis5WF
『カルノー冷凍機で、冷蔵庫をつくり、室温が27℃のとき庫内の温度を−3℃に保つのに20Wの仕事率で仕事を供給する必要があったとすると、 毎秒何カロリーの熱が庫内にもれて入っていることになるか。』
という問題で、解答が
『冷蔵庫から取り出された熱をdQl、室温側に捨てられた熱をdQhとし、入れた仕事をdWとする。符号を考慮すると
dQh=-(dQl+dW)・・・(1)』
とありましたが、 dQh=-(dQl+dW)となるのがわかりません。このことを説明していただけないでしょうか。よろしくお願いします。
特に符号がマイナスになるのが理解できません。
70ご冗談でしょう?名無しさん:2012/05/18(金) 00:08:48.10 ID:???
おしgooのコピペ乙
71ご冗談でしょう?名無しさん:2012/05/18(金) 00:11:47.77 ID:???
>>69
それほしい、あまでうってる?
72ご冗談でしょう?名無しさん:2012/05/18(金) 04:56:39.41 ID:+yOZ4tB+
超ひも理論http://www.youtube.com/watch?v=y-mB5zFG1vU って動画を
見ててオカルトチックな超素人質問で申し訳ないんだけど気になったので質問させて頂きます。
この2重スリット動画と http://www.youtube.com/watch?v=vnJre6NzlOQ
「進化しすぎた脳」って本で読んだ気がする、「意思は神経の電気反応より後に発生する」みたいな
ことを合わせると、「波」の時は人間は観測しないけど「粒子」の時は観測するって考えは既出?
つまり観測したら粒子になったんじゃなくて粒子だから観測したけども自分の意思だと思ってるとか。
こう考えると量子と脳に少し関連性があれば確率の収束も別に不思議じゃないし。
中二病のチラ裏ですが酔った勢いで投稿します。鼻で笑ってください。
73ご冗談でしょう?名無しさん:2012/05/18(金) 06:00:53.30 ID:???
wwwwwwwwwwwwwww
74ご冗談でしょう?名無しさん:2012/05/18(金) 07:54:22.65 ID:???
>>72
何か面白そうなのでわかる人、これ回答よろ
75 忍法帖【Lv=7,xxxP】 :2012/05/18(金) 08:07:28.19 ID:???
最先端の物理学は荒唐無稽な内容だよ。
76ご冗談でしょう?名無しさん:2012/05/18(金) 13:05:28.85 ID:???
Dr.Quantumの動画はいい加減に消去してほしいね。有害だ。
77ご冗談でしょう?名無しさん:2012/05/18(金) 13:17:52.55 ID:???
重力質量と慣性質量についてちょっと質問

1gの質量を光にして100%反射の鏡の球の中に閉じ込めれたと過程して
この場合1gの質量に相当するだけのエネルギーが球内にあることになるので
1g相当の重力を生んで重力に反応することになるわけだけど、慣性質量のほうは
なにが慣性質量の代わりをすることになるの?

中に封じ込めた光子気体の運動量の差が動かしにくさの代わりになる?
78ご冗談でしょう?名無しさん:2012/05/18(金) 13:45:12.45 ID:???
>>77
光子は質量はもってないから重力は作れない。重力相互作用はあるけれども
光子に加速度を与えたら、光速度が不変でなくなるな
79ご冗談でしょう?名無しさん:2012/05/18(金) 13:50:33.83 ID:???
光子に質量はないがエネルギーはある。
一般相対論ではエネルギー運動量テンソルが重力の源である。
したがって1g*c^2のエネルギー分の光子を閉じ込めたなら
1gの静止物質と同じだけの重力を生み出す。
80ご冗談でしょう?名無しさん:2012/05/18(金) 13:55:01.05 ID:???
「意思は神経活動後に発生する」も「波を観測せず粒子を観測する」も
そう言えるけど、それ以下は何でそう考えるか分からん。
81ご冗談でしょう?名無しさん:2012/05/18(金) 13:58:57.54 ID:???
>>78
キミバカジャナイノ?
思考実験で閉じ込めた光子はそのエネルギーに見合った重力質量を持つよ。
82ご冗談でしょう?名無しさん:2012/05/18(金) 14:08:38.88 ID:???
>>77
光は電磁場だから電磁場の加速度に対する反応が動きにくさ(慣性質量)を生み出す。
今は消えてるけど、物理学者のサイトにコンンデンサの電場が加速度に対してどう反応して運動に抵抗するか計算してたのがあった。
光子運動量の差で求めるのと同じ結果かも知れないが。
運動量差なら簡単に計算できるからやって見せてくれ。
(似た問題で電磁波を浴びながら運動する物体に働く力を計算した事があったが忘れた)
83ご冗談でしょう?名無しさん:2012/05/18(金) 15:25:22.18 ID:kQHTfiTV
ベクトル、スクエア、ベクトル で計算してみて 
84ご冗談でしょう?名無しさん:2012/05/18(金) 15:25:43.83 ID:kQHTfiTV
バカみたいだけど、証明でもいいよ。
85ご冗談でしょう?名無しさん:2012/05/18(金) 16:42:50.10 ID:???
>>35
コップの底の方に液体は残ってる?
そうならば、流体の渦度の保存かな。
角運動量保存だね。
86ご冗談でしょう?名無しさん:2012/05/18(金) 20:30:12.93 ID:???
光子がエネルギー分の重力質量をもつということは、1GeVのγ線の光子
は陽子と同じ重力質量をもつということですか?
87ご冗談でしょう?名無しさん:2012/05/18(金) 20:56:33.91 ID:???
No
88ご冗談でしょう?名無しさん:2012/05/18(金) 21:09:22.72 ID:???
89ご冗談でしょう?名無しさん:2012/05/18(金) 21:56:03.89 ID:2KAkuUzK
>>82
電磁気学の本にある輻射の反作用のことですか
90ご冗談でしょう?名無しさん:2012/05/18(金) 23:06:07.06 ID:???
>>89
そりゃ運動量の方だろ
91前スレ888:2012/05/18(金) 23:38:15.10 ID:???
>>85
吸うとズゴゴゴッってなるくらい
つまり殆ど残ってない
92ご冗談でしょう?名無しさん:2012/05/18(金) 23:43:03.75 ID:UiotkqHc
奥行き550ミリの天板をL型の金物で壁に固定するとします。
金物は補強してあり変形はないものとして壁にアンカにて上下で固定します。高さ方向は380ミリ。
計算上は380ミリの両端にアンカーがあるとする。棚板の先端に120kgの重りが架かったとする。上側のアンカーに掛かる力はとのぐ
93ご冗談でしょう?名無しさん:2012/05/18(金) 23:43:13.20 ID:???
たとえば定常波は運動量ゼロ
この定常波を「動かす」には電磁場に運動量を加える必要がある=力を加える必要がある
そしてその「動かしにくさ」はその定常波のエネルギー(をc^2で割ったもの)に等しい
これを慣性質量と呼びたければ呼ぶといい
94ご冗談でしょう?名無しさん:2012/05/18(金) 23:45:38.13 ID:UiotkqHc
答えとしては120*550/380=173かと 思ったのですが、
友達は120sin55*550的なことを言っ てましたが、
真相はどうなのですか? 考え方なども教えてあげると助かり ます。
宜しくお願いいたします
95ご冗談でしょう?名無しさん:2012/05/18(金) 23:53:41.45 ID:???
まず国語の教科書を読むことから始めよう
96ご冗談でしょう?名無しさん:2012/05/18(金) 23:55:38.89 ID:???
これはペンです。
あなたは週末なにをしていましたか?
冗談をおっしゃる。
97ご冗談でしょう?名無しさん:2012/05/19(土) 00:01:28.54 ID:???
栄光の目子筋
98ご冗談でしょう?名無しさん:2012/05/19(土) 09:21:48.73 ID:Jf1g4CEv
>>80
レスありがとうございます。オカルトチックにいうと運命ってやつです。
色々な量子の状態があるけれどもこの時空にいる限りは量子が粒子を示す時は
必ず測ってしまうという考えです。人間の心も数学の塊で、量子が最小単位だと
仮定すればそこに確率の収束が見られるのは不思議はないというか逆に
量子が最小単位に近いからこそ確率の収束が見られるのではないかと考えました。
過去と未来が全て一瞬一瞬変わってるかはしりませんが。
---------------------
ついでに最近頭から離れない考えをまたしても酔った勢いで。
光速というと光が分散してるように思えるけれども実際は止まってる光に対して宇宙
が膨張してる(この次元ではそう見える)から相対的に光に速度が生まれて時間も
生まれるのではないかという考えです。
だからこの3次元空間で光速に移動した所で(多少の時間の変化はあっても)時間が
止まるというのはありえるんでしょうか。膨張する宇宙空間と同じスピードで
大きくなるか小さくなるとか質量が増減するとか別の次元で何かが変化(逆に言えば
止まっている光に近づく)しないと時間が止まることはないような気がします。
しかしこの素人考えはやはり瞬殺ですよね?
99ご冗談でしょう?名無しさん:2012/05/19(土) 10:37:35.36 ID:???
酔いを覚ませ
100ご冗談でしょう?名無しさん:2012/05/19(土) 14:13:03.44 ID:Jf1g4CEv
まだ酔っぱらってるついでにアホの恥はかき捨て上乗せ質問。
今頭の中にある時間の概念を動画にしようとしたら>>98の後者の考えと
整合性が合わないので疑問がわいたんだけど、
質量をもう一つの次元と考えることはアリですか無しですか?
101ご冗談でしょう?名無しさん:2012/05/19(土) 15:05:30.56 ID:???
なしです
102ご冗談でしょう?名無しさん:2012/05/19(土) 15:52:48.99 ID:???
ぶどうです
103ご冗談でしょう?名無しさん:2012/05/19(土) 16:10:31.90 ID:???
ぞうです
104ご冗談でしょう?名無しさん:2012/05/19(土) 16:19:00.43 ID:???
パォー
105ご冗談でしょう?名無しさん:2012/05/19(土) 17:38:08.89 ID:y2mvo5IB
ニュートリノの速度の測定実験て もうやってるんですか?
結果っていつごろ出るんでしょうか
106 忍法帖【Lv=8,xxxP】 :2012/05/19(土) 17:43:48.27 ID:???
してますよ。
107ご冗談でしょう?名無しさん:2012/05/19(土) 17:48:39.49 ID:???
>>105
ニュートリノなんて無いから、予算獲得のために作った捏造
108ご冗談でしょう?名無しさん:2012/05/19(土) 18:45:14.02 ID:???
遊星歯車の作動は3つの軸のどれか1つを固定
あとの2軸は入出力軸だよ

あと単純でも先人の知恵に関心するのは
電気回路の3路回路と4路回路

しらんかったらググってみ
109ご冗談でしょう?名無しさん:2012/05/19(土) 21:27:44.85 ID:???
何で青が一番散乱しやすくて赤が一番散乱しにくいの?
波長の関係?
110ご冗談でしょう?名無しさん:2012/05/19(土) 21:34:22.52 ID:???
yes
111ご冗談でしょう?名無しさん:2012/05/20(日) 14:56:49.29 ID:???
万有引力による楕円運動で、
軌道の長径をa、離心率をeとして、動径の時間平均を求める問題ですが方針が分かりません。
計算は無くて構わないので方針だけでも教えていただきたいです。
宜しくお願いします
112ご冗談でしょう?名無しさん:2012/05/20(日) 16:13:32.45 ID:???
質量m1、運動量p1の粒子1と、質量m2、運動量0の静止してる粒子2の衝突で、
重心系のエネルギーってどう表されるか教えて下さい。
113112:2012/05/20(日) 17:00:21.32 ID:???
自己解決しました
114ご冗談でしょう?名無しさん:2012/05/20(日) 18:35:15.44 ID:???
単純立方格子でk=±π/aのときにブラッグの式を示す。
これどうするんですか?
115ご冗談でしょう?名無しさん:2012/05/20(日) 19:35:51.39 ID:???
>>111
時間平均てのは一周期のあいだ時間で積分して周期で割れば良い。
時間積分は平均近点離角による積分になる。
平均近点離角による積分は真近点離角による積分に変換できる。
動径は真近点離角の関数で表わされる。
116111:2012/05/20(日) 21:15:26.69 ID:???
>>115
ご返答ありがとうございました。
もうちょっと頑張ってみます
117ご冗談でしょう?名無しさん:2012/05/21(月) 02:51:39.29 ID:Dclja11q
こないだNHK高校講座の物理「光の科学史」の回

ttp://www.nhk.or.jp/kokokoza/library/2011/tv/butsuri/archive/resume017.html

見てたら、山梨大学教授の堀 裕和って人が、【光を見た人はいない】っつってました。

何のこっちゃいと思ったら、要するに目の前を光が横切る時に、その経路を
横から見ても「光は見えません」よって、チョー当ったり前のこと。

それ以外にも超絶技巧的でむぱ全開の回でした。面白いのでみなさん
飲み話のネタ仕入れるのに御笑覧下さいw
118ご冗談でしょう?名無しさん:2012/05/21(月) 02:54:38.84 ID:Dclja11q
>>117
あ、肝心の質問ですが、結局この生徒に誤解の種を植えつたいとしか思えない

    【光を見た人はいない】

ってフレーズ、最初に言い出したのはどこのどいつなんですか?

#しかしこんなの劇的なドヤ顔で披露するような格言じゃなかろうに。
119ご冗談でしょう?名無しさん:2012/05/21(月) 03:34:28.80 ID:???
あ、肝心の膣悶ですが、

    【メコスジ野郎を見た人はいない】

ってフレーズ、最初に言い出したのはどこのどいつなんですか?
120 忍法帖【Lv=10,xxxPT】 :2012/05/21(月) 07:51:55.14 ID:???
寧ろ物理学の授業で幸福の科学史をしろよ。
121ご冗談でしょう?名無しさん:2012/05/21(月) 16:42:22.22 ID:9SM24tlr
大学二年です。どうしても分からないのでどなたか解説お願いします。

断面が一辺2ミリの正方形で、長さが20センチのゴムヒも
(ヤング率を2×10^6N/m^2とする)を
30センチに引き伸ばしたときのエネルギーはいくらか。
また、伸ばすときの力の平均値はいくらか。
122ご冗談でしょう?名無しさん:2012/05/21(月) 17:31:52.37 ID:???
(1/2)*E*ε^2*Vじゃないかな
ε=dL/L
V=volume
E=YM?
123ご冗談でしょう?名無しさん:2012/05/21(月) 18:35:11.22 ID:???
質問です

地球の質量はおよそ5.97x10^24(kg)とされていますが、
ケプラーの法則(変形):M=4π^2 r^3/G T^2
(M:地球の質量、r:地球と月との距離、G:万有引力定数、T:月の公転周期)
を月と地球について当てはめて計算すると、
M=4π^2•(3.844x10^8)^3/(2.360448x10^6)^2•6.673x10^-11

これを解くと6.03x10^24(kg)となって少しズレが生まれますが、誤差の範囲でしょうか?
それとも根本的に間違っているのでしょうか。
124ご冗談でしょう?名無しさん:2012/05/21(月) 19:48:08.35 ID:???
>>123
月の質量の分かと
125ご冗談でしょう?名無しさん:2012/05/21(月) 20:08:37.96 ID:???
結局ストローで氷を時計周りにまわしてるだけやんけ
126ご冗談でしょう?名無しさん:2012/05/21(月) 20:09:41.01 ID:???
グラスの底がくぼんでいるからストローを反時計回りにまわすと、ストロー全体は時計回りにまわる
127 忍法帖【Lv=10,xxxPT】 :2012/05/21(月) 20:11:36.85 ID:???
物理学は難解ですよ。
128ご冗談でしょう?名無しさん:2012/05/21(月) 21:38:28.24 ID:YXm/9ENe
121です。
答えが>>122さんのものでよろしいのでしょうか?
良かったら考え方も教えていただきたいのですが…
よろしくお願いします。
129ご冗談でしょう?名無しさん:2012/05/21(月) 22:25:28.59 ID:FlKEh2/H
オイラーの式で対数をとると
iπ=ln(-1)
っていう気持ち悪いものが出て来たんだけど
これって歴史的大発見じゃね?
130ご冗談でしょう?名無しさん:2012/05/21(月) 22:33:11.60 ID:???
なぜ物理で聞く?
131ご冗談でしょう?名無しさん:2012/05/21(月) 22:33:19.89 ID:???
すごーい、ぜひチラシの裏で発表したまえ
132ご冗談でしょう?名無しさん:2012/05/21(月) 22:35:14.52 ID:???
>>129
残念だね。数式処理ソフトのMapleで試したら log(-1)=Iπ となった。
133ご冗談でしょう?名無しさん:2012/05/21(月) 22:50:00.32 ID:???
例えば、光速の半分で走る電車があったとして、
その電車の中で更にその電車を走らせる、とすると
外から観察した場合、一番内側の電車は光速になるんでしょうか?
134ご冗談でしょう?名無しさん:2012/05/21(月) 22:54:47.74 ID:???
なんじゃね?
135ご冗談でしょう?名無しさん:2012/05/21(月) 23:25:02.87 ID:???
>>133
tanhα=1/2, tanhβ=1/2
tanh(α+β)=(tanhα+tanhβ)/(1+tanhαtanhβ)=4/5
で光速の4/5倍
136ご冗談でしょう?名無しさん:2012/05/21(月) 23:56:40.67 ID:???
はいぱぼりぃーーーっく
137ご冗談でしょう?名無しさん:2012/05/21(月) 23:56:56.92 ID:???
>>135 4/5倍になるのか。質問者じゃないけどびっくりした。

ハイパボリックは授業で聞いたものの理解できずにスルーしてたから式が意味分からんけど
なんかちょっと興味沸いたよ。ありがとう
138ご冗談でしょう?名無しさん:2012/05/22(火) 01:57:41.71 ID:???
結局ただの指数関数だよ
139 忍法帖【Lv=10,xxxPT】 :2012/05/22(火) 03:27:46.14 ID:???
物理学を研究して居ると達人の領域にも達成するよ。
140ご冗談でしょう?名無しさん:2012/05/22(火) 04:18:09.70 ID:???
栄光の目子筋
141ご冗談でしょう?名無しさん:2012/05/22(火) 08:26:03.34 ID:qRit1VU4
すみません、どなたか122ありませんか?
142 忍法帖【Lv=11,xxxPT】 :2012/05/22(火) 11:17:11.36 ID:???
別に物理学は最強ですよ。
143ご冗談でしょう?名無しさん:2012/05/22(火) 11:18:17.24 ID:???
ありません
144ご冗談でしょう?名無しさん:2012/05/22(火) 12:46:35.09 ID:???
145133:2012/05/22(火) 16:27:25.72 ID:???
ありがとう、すっきりしました
146 忍法帖【Lv=23,xxxPT】 :2012/05/22(火) 23:23:24.98 ID:???
どっきりしました。
147ご冗談でしょう?名無しさん:2012/05/22(火) 23:46:16.35 ID:???
量子力学では物理量を微分でかいたりしますが、
一体全体どういう根拠があってこんな馬鹿げたことをやるんですか。
やるなら証明をしてからにしてください。

と思うわけですが。いかが?
148ご冗談でしょう?名無しさん:2012/05/22(火) 23:47:12.28 ID:???
もし、ちゃんとみんなの見解を聞きたいなら具体例をどうぞ
149ご冗談でしょう?名無しさん:2012/05/22(火) 23:48:47.24 ID:???
あたらしいな
150ご冗談でしょう?名無しさん:2012/05/23(水) 00:08:50.08 ID:???
>>147
交換関係だけが真実。
151ご冗談でしょう?名無しさん:2012/05/23(水) 00:13:10.47 ID:???
鶏が先か卵が先かそれが問題だ
152ご冗談でしょう?名無しさん:2012/05/23(水) 01:27:10.00 ID:???
>>147
証明はできない。計算値と実験結果と比べて妥当な値が得られるかどうかがすべて。
ただ、シュレディンガー方程式を得るのに古典的なハミルトニアンの中の運動量を微分演算子に
置き換えるという操作には古典力学にモデルがある。ハミルトン=ヤコビの方程式というやつ。
153ご冗談でしょう?名無しさん:2012/05/23(水) 04:56:36.49 ID:???
>>147
普通の物理学者や学生はフォン・ノイマンの証明で満足してるが、それで不足なら好きにしろ。
154 忍法帖【Lv=12,xxxPT】 :2012/05/23(水) 06:55:55.38 ID:???
質問の問題集は物理学で死後の世界を発見出来ますか?。
155ご冗談でしょう?名無しさん:2012/05/23(水) 15:10:08.52 ID:???
>>147
古典力学では物理量を実数で書きますが、それは馬鹿げたことと思いますか?
思わないならその根拠を証明してください。
156ご冗談でしょう?名無しさん:2012/05/24(木) 05:24:22.08 ID:27AOmLVe
1.光は重さを持たないのに何故重力の影響を受けて曲がってしまうのですか?

2.光(電磁波)は何故媒質中だと速度が遅くなるのですか?

3の1.可視光線が不透明な物体にあたって見えなくなったということは可視光線が可視光域から
 不可視域の電磁波に変わったということですか?

3の2.3の1が可視光線が不可視域の電磁波に変わる際に、波長は長くなりますか?
   
3の3.3の2が正しいとすると、波長が限界まで(地球上のあらゆる物体にあたってしまって)
長くなった電磁波はどうなってしまいますか?

宜しくお願いします。
157 忍法帖【Lv=13,xxxPT】 :2012/05/24(木) 07:00:21.07 ID:???
物理学の研究を情熱で推進するよ。
158ご冗談でしょう?名無しさん:2012/05/24(木) 07:55:04.54 ID:???
>>156
1,2はよくわからんというかそれっぽい解説サイトいくらでもあるのでそちらをどうぞ
3は海の波を想像してみたらいいんじゃないかな
押し寄せてくる波が全部光だと思うんだ
岸壁に当たったらエネルギーを放出して残りは跳ね返ったりするだろう
砂浜に達したらエネルギーを吸収されて残りは跳ね返るだろう
それと同じことが光でも起こってる
159ご冗談でしょう?名無しさん:2012/05/24(木) 12:45:42.56 ID:???
>可視光線が不透明な物体にあたって見えなくなった
意味不明だなー
不透明な物体だって可視光線で見えるだろ
160ご冗談でしょう?名無しさん:2012/05/24(木) 14:22:51.88 ID:???
> 1,2はよくわからんというかそれっぽい解説サイトいくらでもあるのでそちらをどうぞ

どうぞっていうんなら誘導くらいしろボケ。
出来ないなら書くなクズ。
161ご冗談でしょう?名無しさん:2012/05/24(木) 14:31:01.42 ID:???
>>160
googleで上の質問入力してエンター押すくらいの頭つかえよカス
162ご冗談でしょう?名無しさん:2012/05/24(木) 14:38:04.47 ID:???
なら最初からそう書けゴミクズ
163ご冗談でしょう?名無しさん:2012/05/24(木) 15:25:09.67 ID:???
>>156
1.空間が歪んでるからじゃね?

2.媒質が邪魔なんじゃね?
164ご冗談でしょう?名無しさん:2012/05/24(木) 16:49:44.23 ID:???
減衰振動を複素数抜きで解く方法ってあるんでしょうか?
強制振動の特解ならダンピングがあってもなんとかなるんですが。
165ご冗談でしょう?名無しさん:2012/05/24(木) 16:58:28.65 ID:???
複素数抜いてどうするの
166ご冗談でしょう?名無しさん:2012/05/24(木) 17:41:48.67 ID:???
>>156
3は光を吸収して熱になるってことかな?
167ご冗談でしょう?名無しさん:2012/05/24(木) 18:01:23.38 ID:???
>>165
最近高校でろくに複素数教えてないしねぇ。
なんとかならんもんかと思って。
168ご冗談でしょう?名無しさん:2012/05/24(木) 18:20:34.56 ID:???
>>164
exp(-t/τ)×f(t)のように解の形を仮定すれば解くことはできるが。
169ご冗談でしょう?名無しさん:2012/05/24(木) 19:32:35.13 ID:???
>>167
Ae^((a+ib)t)の代わりにBe^(at)cos(bt),Ce^(at)sin(bt)を解にするだけだが
170ご冗談でしょう?名無しさん:2012/05/24(木) 19:49:34.62 ID:???
>>167
単振動を複素数抜きで解いてるなら似たようなもんでは?
171ご冗談でしょう?名無しさん:2012/05/24(木) 23:09:13.96 ID:???
>>156
> 1.光は重さを持たないのに何故重力の影響を受けて曲がってしまうのですか?
重力の大きさはmgと表されるので一見すると重力は物質の質量と密接に関わった力に見えるけど、
運動方程式を書いてみると結局 a=g になって、その物質の質量とは無関係に軌道が決まる
つまり、むしろ重力の本質は、物体の質量とは無関係にどんな物質に対しても同じように軌道を曲げるものだと言える
(普通の力だったら物質の質量によってその力が軌道にどのように影響するかが異なってくる)
そう考えると、質量のない光にも重力が作用するのは自然なこと
これを突き詰めていくと一般相対性理論になる
172ご冗談でしょう?名無しさん:2012/05/24(木) 23:37:23.82 ID:???
プロレスのトップコーナーからのフライングボディアタックやムーンサルトは
物理的には両方とも同じダメージなんでしょうか?
173 忍法帖【Lv=2,xxxP】 :2012/05/25(金) 02:26:21.18 ID:???
未来の将来の進化した物理学はワープ技術とか超光速航法とかワームホールテクノロジーを実用化出来ますか?。
174156:2012/05/25(金) 02:35:14.49 ID:???
解答どうもありがとう。
>>158
探しが甘いということでしょうか。Wikipediaをもう一度見なおしてみました。
1について:重力レンズの項より"光が曲がることは一般相対性理論から導かれる現象で、
光は重力にひきつけられて曲がるわけではなく、
重い物体によってゆがめられた時空を進むために曲がる。"

自分には時空が歪むという事が難しく理解できませんでした。

2について:http://soudan1.biglobe.ne.jp/qa4643517.html
マクスウェル方程式は理解できませんでしたが、ここにある例え
「量子電磁力学によると、媒質が存在するとそれの電子に光子が取り込まれ
その後再び電子が光子を放つ、媒質の密度が大きいほど距離あたりのその回数は増えてしまい遅くなる」という風に理解しました
。これで大体はあっていますでしょうか
175156:2012/05/25(金) 02:48:40.02 ID:???
>>158
3について:砂浜に与えられたエネルギーは再び波を起こす力になってしまうと思ってしまいました。
>>159
可視光線が物体にあたって反射する際に、不可視域の電磁波に波長が変わってしまうということです。
肉眼で見えなくても電磁波は存在していますから
>>163
1.空間が歪むというのが難しい概念でわかりませんでした
2.は上で調べたとおりです。
>>166
熱も電磁波の一種だと思うので、
太陽が光を与え続けて地球上で物体がそれを受け取り熱に変換し続けたら
地球の気温が永久に上がり続けることになると考えました。
しかし現実にはそうなっていないということは
ある程度まで上に上げたように光が光子→電子→光子のサイクルを続けて伝播し続けると
地球上から放出されてしまうと考えたのです、そして上のサイクルの際に波長が長く変化すると考えたのです。
176156:2012/05/25(金) 03:00:14.81 ID:???
>>171
>運動方程式を書いてみると結局a=gとなって、その物体の質量とは無関係に軌道が決まる
ここが少しわかりませんでした。

但し、上で挙げた重力が等しく力を及ぼす範囲に空間自体が含まれていて、
それが歪んで光がそこを直進しても曲がって見えるという事と、
質量に関係なく(即ち質量0、真空中でも)同じように重力が作用して軌道が曲げられるというのは
整合しているように思えます。
177ご冗談でしょう?名無しさん:2012/05/25(金) 08:08:33.11 ID:???
>>168
>>169
それって、はじめから解の形がわかってないと出来ないですよね。
一からe^(at)の分も含めて導出ってのはやっぱり無理なんですかね。

>>170
単振動なら2回微分して「−自分自身」になる関数ということで三角関数に行きつくのは容易だと思うんですが。
178 忍法帖【Lv=2,xxxP】 :2012/05/25(金) 09:28:29.47 ID:???
僕は物理学を愛用して居ますよ。
179ご冗談でしょう?名無しさん:2012/05/25(金) 11:00:48.83 ID:ICNV2jdU
あいまいだが波動関数を、数列計算(大学の行列狽ナもいいが)して、ベクトル、スクエア、ベクトルにして計算してみて。量だけど、波計算がわかるよ。さらに、ベクトル、スクエア、ベクトルにして、わかりやすくしてもいいけど 登記
180ご冗談でしょう?名無しさん:2012/05/25(金) 11:01:41.24 ID:ICNV2jdU
あいまいだが波動関数を、数列計算(大学の行列狽ナもいいが)して、ベクトル、スクエア、ベクトルにして計算してみて。量だけど、波計算がわかるよ。さらに、ベクトル、スクエア、ベクトルにして、わかりやすくしてもいいけど 中級だけどね 登記
181ご冗談でしょう?名無しさん:2012/05/25(金) 11:49:56.82 ID:???
>>171
一般相対論で解くと大局的にはその倍光が曲がるんだが…
182ご冗談でしょう?名無しさん:2012/05/25(金) 11:55:27.04 ID:qzvYIqGi
丸投げのほうにも書いてしまったけどこちらでも質問させてください。
ttp://uni.2ch.net/test/read.cgi/sci/1284096345/803

向こうとは文章変えてしまいますが、
半導体中に入力する電気信号の波形が変わると内部量子効率も変化する。
ということを調べているんですが、波形を変えた場合の記述がなかなかなくて困ってます。
おそらくキャリアの動きが変わってくると思うのですが、詳しい方おりましたら調べ方のヒントだけでも教えていただけると幸いです。
183ご冗談でしょう?名無しさん:2012/05/25(金) 11:56:52.53 ID:???
> 丸投げのほうにも書いてしまったけどこちらでも質問させてください。

ダメ
184ご冗談でしょう?名無しさん:2012/05/25(金) 12:20:25.88 ID:ICNV2jdU
あいまいだが波関数を、数列計算(大学の行列狽ナもいいが)して、ベクトル、スクエア、ベクトルにして計算してみて。量だけど、波計算がわかるよ。さらに、ベクトル、スクエア、ベクトルにして、わかりやすくしてもいいけど 中級だけどね 登記
185ご冗談でしょう?名無しさん:2012/05/25(金) 14:19:36.15 ID:???
奥さんなみかんすうですってオホホ
186ご冗談でしょう?名無しさん:2012/05/25(金) 14:26:58.99 ID:ukmMULuw
うちににロンギニスの槍あるよ、
187ご冗談でしょう?名無しさん:2012/05/25(金) 14:55:17.31 ID:???
うちににメコスジスの槍曼あるよ、
188ご冗談でしょう?名無しさん:2012/05/25(金) 15:11:50.98 ID:???
>>156

1.等電位面によるもの
2.光の伝達方法を理解すべき
3-1.ラマン分光による現象
3-2.変わるものもあれば、変わらないものもある
3-3.原子や分子に吸収される
189ご冗談でしょう?名無しさん:2012/05/25(金) 15:35:52.19 ID:idqSSC02
真空中を進む電磁波は, 一般に平面波として近似して取り扱って良いらしいですが
その理由をおしえて下さい。


190ご冗談でしょう?名無しさん:2012/05/25(金) 15:37:49.36 ID:???
>>177
>単振動なら2回微分して「−自分自身」になる関数ということで三角関数に行きつく
それは解になる関数をあらかじめ知ってたってことで導出はしてないのでは?
191ご冗談でしょう?名無しさん:2012/05/25(金) 16:00:28.04 ID:???
まぁ、高階微分方程式は解になる関数をあらかじめ知ってないと解けないのがほとんどだ罠。
厳密解が得られる形は限られてるから、漸近形を予想してあらかじめくくり出しておいたり
その他いろいろな変数変換したり積分変換したりして既知の形に追い込んでいくしかない。
192ご冗談でしょう?名無しさん:2012/05/25(金) 16:26:26.82 ID:???
193ご冗談でしょう?名無しさん:2012/05/25(金) 16:40:15.01 ID:???
>>189
>真空中を進む電磁波は, 一般に平面波として近似して取り扱って良い
が嘘
194 忍法帖【Lv=2,xxxP】 :2012/05/25(金) 20:19:05.36 ID:???
進歩した物理学でワープ技術とか超光速航法とかワームホールテクノロジーを実用化出来ますか?。
195ご冗談でしょう?名無しさん:2012/05/25(金) 21:25:21.23 ID:???
>>177
線型常微分方程式を解くことは、独立な解を求めることです。
二階の常微分方程式の解空間は二次元です。二つの独立な解を求めれば終わりです。
ady^2/dx^2+bdy/dx+c=0
の独立な解はaλ^2+bλ+c=0の複素解λ1、λ2でe^(λ1t)、e^(λ2t)と書けます。
196ご冗談でしょう?名無しさん:2012/05/25(金) 21:29:15.86 ID:???
>>195
発端は>>164
197ご冗談でしょう?名無しさん:2012/05/25(金) 21:36:28.49 ID:???
>>196
loop{
hogehoge();//疲れた
} until (1)
198補足:2012/05/25(金) 21:39:13.93 ID:???
これで高校の先生がつとまるのか、いいなー
199ご冗談でしょう?名無しさん:2012/05/25(金) 23:30:42.16 ID:???
予備校の先生には大学程度じゃ大した事は勉強はできないって言われましたね・・・
200ご冗談でしょう?名無しさん:2012/05/26(土) 00:38:02.86 ID:YKMNSM8T
コイルの両端の電圧を測定したら、一定の(時刻によらない)値であった。ただし0ではない。
この時のコイルに流れていた電流はどのようなものであったと言えるか。
これってどんな感じで解答したらいいのでしょうか・・・
201ご冗談でしょう?名無しさん:2012/05/26(土) 00:39:45.75 ID:???
一定の割合で電流が増加していた
でFA
e=-Ldi/dt
202 忍法帖【Lv=25,xxxPT】 :2012/05/26(土) 00:44:07.65 ID:???
もっと更に物理学でデジモンを産み出したいですよ。
203ご冗談でしょう?名無しさん:2012/05/26(土) 00:47:33.35 ID:???
>>202
Get lost!
204ご冗談でしょう?名無しさん:2012/05/26(土) 01:10:48.65 ID:YKMNSM8T
>>201
ふむ、なるほど、ありがとうございます
205 忍法帖【Lv=26,xxxPT】 :2012/05/26(土) 09:53:12.76 ID:???
物理学を応用してワープ技術とか超光速航法とかワームホールテクノロジーを実用化して欲しいですよ。
206ご冗談でしょう?名無しさん:2012/05/26(土) 17:20:54.40 ID:???
実験で120.236179と109.704522という2つの測定値を得て、どうやらこの間あたりに真値があるらしいです。
真値として平均値を取れと指示されているのですが、どこまで桁を見てやれば良いのでしょうか。
207ご冗談でしょう?名無しさん:2012/05/26(土) 17:23:16.54 ID:???
>>206
使ってる測定器の精度があるだろ。そこから誤差評価をはじめる。
208ご冗談でしょう?名無しさん:2012/05/26(土) 17:40:42.45 ID:???
120と110の間っていうひどい測定精度で測定器の精度なんて見ても無意味。
有効桁2桁ていどになるのが見えてるので、念のため3桁+1桁で計算しておけば十分。
209ご冗談でしょう?名無しさん:2012/05/26(土) 18:15:03.01 ID:???
>>206
1桁くらい。良くて2桁。データ点は最低4つ、出来れば10点は欲しい。
210ご冗談でしょう?名無しさん:2012/05/26(土) 21:12:11.02 ID:???
900 :774ワット発電中さん [↓] :2012/02/05(日) 20:52:50.86 ID:gUlk7tHv
何を言ってるんだ、こいつは?と思った時は

参考にする=>コピペする
考える=>検索する
調べる=>解説しているサイトを探す
勉強する=>模範解答を丸暗記する
(書いてあることが)よくわからない=>読む気がしない
詳しく書いてあるところはありませんか?=>自分のレベルに合わせてくれるべきじゃないんですか?
・・・・

という具合に読み替えてみると、意味が分かることが多い。
211ご冗談でしょう?名無しさん:2012/05/26(土) 22:23:31.72 ID:???
大学でPowerpointでプレゼンをすることになったのですけど、
数式が上手くプレースホルダに入りません。
どのレイアウトを使用すれば良いのでしょうか?
初めてですのでいろいろ教えて欲しいです。
一応、数式はWordの数式エデイタで作りました。
212ご冗談でしょう?名無しさん:2012/05/26(土) 22:34:00.80 ID:???
>>210
それ以前のことがたたある
213ご冗談でしょう?名無しさん:2012/05/26(土) 23:27:20.91 ID:???
また質問すいません。
1(F)のコンデンサに電流i=sin(wt)[A]を流したとき、コンデンサの両端の電圧v(t)はどうなるか。
t=0における両端の電圧は0(V)とする。
という問題です。お願いします。
214ご冗談でしょう?名無しさん:2012/05/27(日) 00:03:49.09 ID:???
Q=CVを微分して代入して積分すればいい
215ご冗談でしょう?名無しさん:2012/05/27(日) 00:10:14.08 ID:zulyl3oo
電流について疑問なんですが、

LEDと抵抗を直列繋ぎにする場合、抵抗があればLEDに流れる電流が
少なくなってLEDが壊れないんですが、感覚的に抵抗をつけていたとしても
LEDを先に繋いでしまうと、スイッチ入れた瞬間に壊れてしまう気がするんですが
どうでしょう?

電源がスイッチを入れた瞬間の瞬間は、LEDだけかと勘違いして
電流を膨大に流してしまう的な事は発生しないんでしょうか?
216ご冗談でしょう?名無しさん:2012/05/27(日) 00:20:23.16 ID:???
>>215

電流の流れ始めの電位変化の速度は光速だから、
勘違いする暇もない。
217ご冗談でしょう?名無しさん:2012/05/27(日) 00:23:13.42 ID:???
LEDが壊れるのは、過剰な電流が一定時間流れ続けることによる発熱が主な原因です。
しかし、電磁気現象はほぼ光速で伝搬するため、抵抗の接続順がどうであれ、
抵抗値が適切であればLEDに過剰な電流が流れ続けることはないため壊れません。
ただし、回路の規模が太陽系単位になれば、マイナス極側に過剰電流が流れ続けることがあるかもしれません。
218ご冗談でしょう?名無しさん:2012/05/27(日) 00:26:42.24 ID:zulyl3oo
>>217
あぁ
そうか、10Aとか言ってるけど、あれって1秒で10Aという意味で
10Aという量を持った個体が一遍に来るって意味じゃないんですよね。
中2の理科で電流習ったと思う、今でもそうだと思うんですが
A=C/sって事中学では教えてるんでしょうか?
219ご冗談でしょう?名無しさん:2012/05/27(日) 00:27:49.02 ID:Df+mBY/B
ワームホールについて知りたいのですが何かよい本ないですか
220 忍法帖【Lv=26,xxxPT】 :2012/05/27(日) 00:28:59.89 ID:???
遊戯王の魔法カード。
221ご冗談でしょう?名無しさん:2012/05/27(日) 00:29:33.20 ID:???
>>218
横だけど高校から
222ご冗談でしょう?名無しさん:2012/05/27(日) 00:55:05.29 ID:???
>>218
空のホースに水を流仕込むようなイメージで電流を捉えているのだと思いますが少し違います。
あえて例えれば、水が詰まっているけど流れていないホースの蛇口を開けて水を流しだすイメージです。
つまり電線にはいたるところ電気が既に詰まっていて電池のプラス極からマイナス極まで配線がつながった瞬間に全ての部分で流れ出すと考えたほうが良いと思います。
また電流は「1秒に10A」ではなく、ある瞬間で10Aです。
個体が移動するというより上に書いたようにぎっしり詰まった液体が押し出されていくとイメージしたほうが良いです。
液体の量(リットル)にあたるものを電気量(クーロン)といいます。
1秒あたり1クーロンの量で流れている状態が1Aです。
223ご冗談でしょう?名無しさん:2012/05/27(日) 01:03:41.03 ID:zulyl3oo
>>222
そうですよね。
224ご冗談でしょう?名無しさん:2012/05/27(日) 01:09:43.25 ID:zulyl3oo
A=C・sか。
225ご冗談でしょう?名無しさん:2012/05/27(日) 01:11:20.50 ID:zulyl3oo
違うw
C/sであってる。

10Aは1sに10cか。
226 忍法帖【Lv=15,xxxPT】 :2012/05/27(日) 04:46:35.08 ID:???
てす。
227 忍法帖【Lv=3,xxxP】 :2012/05/27(日) 05:04:06.48 ID:???
物理学の定義ではワープ技術とか超光速航法とかワームホールテクノロジーを実用化して欲しいですよ。
228ご冗談でしょう?名無しさん:2012/05/27(日) 05:51:21.94 ID:???
ダウンタウンととんねるずはどっちの方が凄いのでしょうか?
229ご冗談でしょう?名無しさん:2012/05/27(日) 06:15:47.59 ID:???
初めてガラスが発明された時
一番最初にそれで海の中を見た人は怖くなかったんでしょうか
230ご冗談でしょう?名無しさん:2012/05/27(日) 08:51:18.92 ID:226ExpC+
>>211に回答できる奴はいないのかw
231ご冗談でしょう?名無しさん:2012/05/27(日) 09:06:27.14 ID:???
>>230
そんなこと本人に言われてもw
232ご冗談でしょう?名無しさん:2012/05/27(日) 09:22:15.80 ID:???
股間のPowerspotでセックスをすることになったのですけど、
珍歩が上手くメコスジホールに入りません。
233ご冗談でしょう?名無しさん:2012/05/27(日) 09:33:51.16 ID:???
TeXでプレゼンしろ、以上
234ご冗談でしょう?名無しさん:2012/05/27(日) 09:34:52.44 ID:???
>>230
丁寧に回答してあげな。
私はパワポ使ったことないから分からん。
235ご冗談でしょう?名無しさん:2012/05/27(日) 09:39:37.06 ID:???
フジテレビの韓国爆ageなどマスコミの偏向報道。
元を辿れば国際金融資本家達の支配によるものだ。

金融マフィア大量逮捕のテレビ報道近し
http://www.youtube.com/watch?v=28-ABN3l4D0

ジョン・レノンのインタビュー―世界は狂人によって支配されている
http://www.youtube.com/watch?v=8t5awfxTbmM

銀行家の搾取システム
http://www.youtube.com/watch?v=yp0ZhgEYoBI#t=57m7s

中川昭一 財務大臣が失脚し、暗殺された理由
http://www.youtube.com/watch?v=Cat1y9C4ytU

なぜ、今叫ばれる「維新」は「偽物」なのか(藤井教授講演)
http://www.youtube.com/watch?v=Sj9ZmL3U-ZE
http://www.youtube.com/watch?v=SMSKIGUgY0M

初めて実行員が暴露した、アメリカの国際陰謀・工作秘史
http://www.youtube.com/watch?v=BhfCgO0ItY8

カダフィの真実を知ってほしい  リビア 新世界秩序 NATO
http://www.youtube.com/watch?v=aggieI4YAVw

日本は財政破綻なんてしません。公務員天国も大嘘。
http://www.youtube.com/watch?v=6KorePC3xWQ

ほとんどの人が知らされていない、知る機会を奪われていただけなのである。
インターネットは庶民の最大の武器だ。
236ご冗談でしょう?名無しさん:2012/05/27(日) 09:57:48.91 ID:???
>>230
よしっ、お前に任せたぞ!
237ご冗談でしょう?名無しさん:2012/05/27(日) 10:02:38.66 ID:???
物理の質問じゃねーから無視されてんだろ
馬鹿じゃねーの
238ご冗談でしょう?名無しさん:2012/05/27(日) 11:41:12.88 ID:M2Ie4rKy
超弦理論では、コリオリの力や遠心力と、重力はまったく別の力として扱われるのか?
つまり等価原理は成立しなくなるのでしょうか?
239ご冗談でしょう?名無しさん:2012/05/27(日) 13:13:25.82 ID:???
超弦理論まで行かなくても、すでに量子力学で等価原理は限定された意味しかもっていないとか。
ttp://www.aa.alpha-net.ne.jp/t2366/第8回          曲がった時空中の量子力学.htm
240ご冗談でしょう?名無しさん:2012/05/27(日) 13:30:28.49 ID:vodVr9Y1
>>180
あいまいだが波動関数を、数列計算(大学の行列狽ナもいいが)して、ベクトル、スクエア、ベクトルにして計算してみて。量だけど、波計算がわかるよ。さらに、ベクトル、スクエア、ベクトルにして、わかりやすくしてもいい、さらに、ベクトル、スクウア、
べクトルにして、さらにわかりやすくしてもいけど 中級だけどね 登記
>>180以下それで、読んでみて
241ご冗談でしょう?名無しさん:2012/05/27(日) 13:36:17.46 ID:vodVr9Y1
その先は、幾何になり、数学で解く。帝京大学の学長が、図形であらわした大学数学の教科書が参考になると思う。
242ご冗談でしょう?名無しさん:2012/05/27(日) 13:37:28.39 ID:u5gBUFi9
>>211に回答できる奴はいないのかw
243ご冗談でしょう?名無しさん:2012/05/27(日) 13:55:38.61 ID:vodVr9Y1
>>240
さらに、あいまいだが波動関数を、数列計算(大学の行列狽ナもいいが)して、ベクトル、スクエア、ベクトルにして計算してみて。量だけど、波計算がわかるよ。さらに、ベクトル、スクエア、ベクトルにして、わかりやすくしてもいい、さらに、ベクトル、スクウア、
べクトルにして、さらにわかりやすくしてもいけど 中級だけどね 登記
244ご冗談でしょう?名無しさん:2012/05/27(日) 13:55:55.69 ID:???
すれちだろ、ちえおくれできいたら
245ご冗談でしょう?名無しさん:2012/05/27(日) 14:04:24.05 ID:vodVr9Y1
熱、力学など問題が解けるよ。
246ご冗談でしょう?名無しさん:2012/05/27(日) 14:08:37.06 ID:???
>>211
俺はTeXで書いてpdfから切り取ってる
247ご冗談でしょう?名無しさん:2012/05/27(日) 14:13:34.73 ID:vodVr9Y1
登記
248ご冗談でしょう?名無しさん:2012/05/27(日) 14:30:17.18 ID:vodVr9Y1
バカかもしれないけど、過去スレから粒子の振るまいの音楽かけてみて
249ご冗談でしょう?名無しさん:2012/05/27(日) 14:32:08.14 ID:vodVr9Y1
バカかもしれないけど、過去スレから粒子の振るまいの音楽かけてみて 登記
250ご冗談でしょう?名無しさん:2012/05/27(日) 15:12:26.98 ID:???
登記

だっておwwwwwwwwwwwwwwwwwwwwwwwwwwwwwwwwww
プッwwwwwwwwwwwwwwwwwwwwwwwwwwwwwwwwwwww
251ご冗談でしょう?名無しさん:2012/05/27(日) 15:16:35.34 ID:???
頭大丈夫か
252ご冗談でしょう?名無しさん:2012/05/27(日) 15:36:07.86 ID:vodVr9Y1
大学数Tを証明してみてから、言ってみろ、本当のバカども
253ご冗談でしょう?名無しさん:2012/05/27(日) 15:59:48.49 ID:???
時間って相対論的な場合でも一様なんですか?
254ご冗談でしょう?名無しさん:2012/05/27(日) 16:02:15.87 ID:???
>>246
パワポのどのレイアウトをよくしていますか?
255ご冗談でしょう?名無しさん:2012/05/27(日) 16:33:21.66 ID:vodVr9Y1
専用のソフト使いなさい。学会発表で、それを使ったら、笑われるよ。過去にそういう教授がいて、笑われたことがあると聞いているよ。
256ご冗談でしょう?名無しさん:2012/05/27(日) 16:53:40.03 ID:???
>>254
にほんごつかいなさい
257ご冗談でしょう?名無しさん:2012/05/27(日) 17:23:24.76 ID:???
powerpointはレイアウト選ばないと作成できないだろ。
プレースホルダの形状とか。
258ご冗談でしょう?名無しさん:2012/05/27(日) 18:04:09.69 ID:???
横磁場イジング模型の相関関数ってどうやって求めるんですか?
259ご冗談でしょう?名無しさん:2012/05/27(日) 19:48:21.59 ID:???
物理板の皆さん
とある試験の物理の問題の解答をお願いしてもよろしいですか?
260ご冗談でしょう?名無しさん:2012/05/27(日) 19:53:12.15 ID:???
解答が用意されてるような軟弱な問題は考える価値もない
261ご冗談でしょう?名無しさん:2012/05/27(日) 20:07:54.28 ID:???
おーー
262ご冗談でしょう?名無しさん:2012/05/27(日) 20:49:10.80 ID:Cw2dqij/
>>254に回答できる奴はいないのかw
263ご冗談でしょう?名無しさん:2012/05/27(日) 20:57:13.31 ID:???
パワポ関連に質問はyahoo知恵遅れでどうぞ
ここは物理の質問を扱うスレです
264 忍法帖【Lv=4,xxxP】 :2012/05/27(日) 21:05:43.06 ID:???
物理学は非常に面白い内容ですよ。
265ご冗談でしょう?名無しさん:2012/05/27(日) 21:15:58.31 ID:???
うるさい。消えろ。
266ご冗談でしょう?名無しさん:2012/05/27(日) 21:26:55.49 ID:???
物理学よりここの住人を見てる方がおもしろい。
267ご冗談でしょう?名無しさん:2012/05/27(日) 21:47:49.71 ID:EqAAYlWh
>>263
物理を学ぶ学生にとって必要な知識なので
>>254への回答は必要。
268ご冗談でしょう?名無しさん:2012/05/27(日) 22:00:48.40 ID:???
本当の物理の学生か研究者ならtexをpdfにして発表するので問題ありません。
パワポの質問ならyahoo知恵遅れでどうぞ。
269ご冗談でしょう?名無しさん:2012/05/27(日) 22:41:43.91 ID:???
【パワーポイント】Powerpoint総合相談所5
http://toro.2ch.net/test/read.cgi/bsoft/1228006144/

270ご冗談でしょう?名無しさん:2012/05/28(月) 01:21:17.02 ID:???
>>211
>どのレイアウトを使用すれば良いのでしょうか?
>初めてですのでいろいろ教えて欲しいです。
>一応、数式はWordの数式エデイタで作りました。

タイトルページ以外は白紙しかつかわん。
作り付けのプレースホルダなど使う気がしない。
だって、発想がプレースホルダを埋める方向に固定化されちゃうから、邪魔くさい。

数式は、マイクロソフトの数式エデイタはバージョンアップが上手くいってないので、
そもそもの開発元の製品のMathTypeに乗り換えた。
今となってはWordの数式エデイタなぞクソ食らえだ。
TeX−>pdf数式も使うけど。

他人と共同作業しないなら、パワポでなくKeynoteを使うことが多い。趣味の問題だが。
立体図形だけはパワポかGoogle Sketchupで作るけど...
271ご冗談でしょう?名無しさん:2012/05/28(月) 02:40:06.14 ID:???
香港と東京はどっちの方が都会なんでしょうか?
孔子とビル・ゲイツはどっちの方が凄いのでしょうか?
老子とアルキメデスはどっちの方が凄いのでしょうか?
釈迦とキリストとムハンマドとガウスとオイラーとアルキメデスとエウクレイデスとリーマンとニュートンとゲーデルとアリストテレスとアブラハムとモーセを、凄い順に並べてください。
アメリカ大統領とロシア大統領と中国国家主席とイギリス君主とフランス大統領とロスチャイルド家当主とマイクロソフトCEOとローマ教皇とアル・アズハル大学総長とコンスタンディヌーポリ総主教と
ロックフェラー家当主とドイツ大統領とイスラエル大統領とサウジアラビア国王とエジプト大統領とブラジル大統領を、偉い順に並べてください。
272 忍法帖【Lv=4,xxxP】 :2012/05/28(月) 04:57:35.38 ID:???
当然物理学で死後の世界を発見する可能性の確率も有るよ。
273ご冗談でしょう?名無しさん:2012/05/28(月) 07:09:45.65 ID:???
どこの国のマフィアが一番強いのでしょうか?
274ご冗談でしょう?名無しさん:2012/05/28(月) 07:29:35.73 ID:???
ゲッターエンペラーの知能指数はどのくらいだと思われますか?
275ご冗談でしょう?名無しさん:2012/05/28(月) 08:24:06.64 ID:???
>>268
学部生なら普通にパワポだよw

>>270
>タイトルページ以外は白紙しかつかわん。
作り付けのプレースホルダなど使う気がしない。
だって、発想がプレースホルダを埋める方向に固定化されちゃうから、邪魔くさい。

白紙のレイアウトでは、パワポでは文字など入力できないはずだが?
どうやって白紙に直接入力するのでしょうか?
276ご冗談でしょう?名無しさん:2012/05/28(月) 09:29:54.26 ID:???
マジレスしてほしいのですが、孔子とガウスはどっちの方が凄いのでしょうか?
また、釈迦とニュートンはどっちの方が凄いのでしょうか?
277ご冗談でしょう?名無しさん:2012/05/28(月) 09:41:21.34 ID:???
釈迦とキリストとムハンマドを、凄い順に並べてください。
278ご冗談でしょう?名無しさん:2012/05/28(月) 11:43:40.03 ID:???
荒らしは無視
279ご冗談でしょう?名無しさん:2012/05/28(月) 13:33:34.83 ID:YyPw8WzW
連結ばね振り子のモードってどう考えればいいんでしょうか
280ご冗談でしょう?名無しさん:2012/05/28(月) 14:57:53.80 ID:???
>>276 当然、孔子、ニュートン。こんなこともわからないのか。
281ご冗談でしょう?名無しさん:2012/05/28(月) 15:36:29.88 ID:???
莫迦とメコスジストとムハムハマタを、絵呂い順に並べてください。
282ご冗談でしょう?名無しさん:2012/05/28(月) 17:53:27.81 ID:0BxW6XiH
>>257に回答できる奴はいないのかw
283ご冗談でしょう?名無しさん:2012/05/28(月) 20:27:30.04 ID:???
>>280
理由は?
284ご冗談でしょう?名無しさん:2012/05/28(月) 21:01:36.47 ID:???
学生でもpdf使うだろ
どんだけ低レベルなんだよ
285ご冗談でしょう?名無しさん:2012/05/28(月) 21:03:50.49 ID:???
粘着できても知恵はない
286ご冗談でしょう?名無しさん:2012/05/28(月) 21:13:54.67 ID:???
>>271
こんなところだろ?

アメリカ大統領=サウジアラビア国王=ブラジル大統領=イギリス君主=フランス大統領=ドイツ大統領=エジプト大統領
>ロスチャイルド家当主=ロックフェラー家当主>コンスタンディヌーポリ総主教>ローマ教皇>アル・アズハル大学総長
>マイクロソフトCEO>イスラエル大統領>中国国家主席>ロシア大統領

理由
イルミナティは自ら解散と申しておりました
イスラエルは空中分解
近い将来ロシア大統領は刑務所行き
近い将来中国は国家解体
近い将来ローマ法王は対象年齢を若くする
EU圏は近々問題があって
287ご冗談でしょう?名無しさん:2012/05/28(月) 21:39:25.21 ID:???
なぜヒマラヤ君を炊遁しないのしょうか?
288ご冗談でしょう?名無しさん:2012/05/28(月) 22:05:31.62 ID:???
>>286
もっと真面目に答えろ。
あと、>>271の他の質問にも答えろ。
289ご冗談でしょう?名無しさん:2012/05/28(月) 22:05:40.92 ID:???
板があれてもしょうがないな
290ご冗談でしょう?名無しさん:2012/05/28(月) 22:12:06.78 ID:???
バッブル望遠鏡は月面のアポロ着陸船を撮影できないのですか?
291ご冗談でしょう?名無しさん:2012/05/28(月) 23:34:31.10 ID:Mq+ABNJH
>>257に回答できる奴はいないのかw
292ご冗談でしょう?名無しさん:2012/05/28(月) 23:48:17.26 ID:???
86 LEVIN(Wataru) vs めこすじ豆腐店
293ご冗談でしょう?名無しさん:2012/05/28(月) 23:56:56.47 ID:???
「全て」と「無」はどっちの方が至高か分かる人いる?
294ご冗談でしょう?名無しさん:2012/05/28(月) 23:58:12.06 ID:???
295ご冗談でしょう?名無しさん:2012/05/28(月) 23:58:12.51 ID:???
がんばれひまれや、れべる100になるまで
296ご冗談でしょう?名無しさん:2012/05/29(火) 00:27:42.87 ID:???
>>294
>>293の質問に答えてください。お願いします。
297ご冗談でしょう?名無しさん:2012/05/29(火) 04:57:53.04 ID:???
荒らしは無視
298ご冗談でしょう?名無しさん:2012/05/29(火) 05:08:16.26 ID:???
死ねば無になると同時に君は全てにもなれる
299ご冗談でしょう?名無しさん:2012/05/29(火) 05:33:58.68 ID:???
>>298
>>293の質問に答えてください。お願いします。
300 忍法帖【Lv=2,xxxP】 :2012/05/29(火) 06:30:37.96 ID:???
デジモンが至高ですよ。
301ご冗談でしょう?名無しさん:2012/05/29(火) 07:17:17.48 ID:???
真面目に教えろレベル2の雑魚。
302!ninja:2012/05/29(火) 07:47:01.89 ID:???
質問しなさい
303ご冗談でしょう?名無しさん:2012/05/29(火) 08:05:01.23 ID:???
タラバガニはヤドカリ科
304ご冗談でしょう?名無しさん:2012/05/29(火) 08:13:42.00 ID:???
>>302
>>293の質問に答えてください。
305 忍法帖【Lv=40,xxxPT】 :2012/05/29(火) 09:21:26.26 ID:???
>>304
お答えします
答えは「います」です
306イムジンリバー ◆NOsnoCwlwY :2012/05/29(火) 09:54:59.79 ID:???
>>290
分解能からいっても無理ですが、そもそもアポロ宇宙船は月にはありません
307ご冗談でしょう?名無しさん:2012/05/29(火) 11:44:51.74 ID:???
それに月は動いてるしね。
バッブル望遠鏡は止まってる物しか撮影できない。
308ご冗談でしょう?名無しさん:2012/05/29(火) 12:13:21.09 ID:???
振り子が
ax+ay=0    と近似できる時
どのような軌道を取りうるか教えてください
309ご冗談でしょう?名無しさん:2012/05/29(火) 12:15:31.65 ID:???
3次元です
zを鉛直下としたときの
xy平面に射影した時の軌道です
310ご冗談でしょう?名無しさん:2012/05/29(火) 12:55:32.74 ID:???
a=0ならあらゆる軌道を取り得る。
a≠0ならa(x+y)=0の両辺をaで割って
y=-x
この平面内で運動する。
311ご冗談でしょう?名無しさん:2012/05/29(火) 14:00:46.62 ID:???
あ、すいません
aは加速度のことです
aのx成分とy成分です
312ご冗談でしょう?名無しさん:2012/05/29(火) 14:06:09.59 ID:???
>>308
運動方程式と初期条件を全部書いて
313ご冗談でしょう?名無しさん:2012/05/29(火) 14:07:44.22 ID:???
a_x+a_y=0 の両辺をtで2回積分して
y=-x+Ct+D (C,Dは時間によらない定数)
この平面内で運動する。
314ご冗談でしょう?名無しさん:2012/05/29(火) 14:09:19.40 ID:???
>>312
質量m 長さl 振り子の振幅x,yは lに比べて十分小さい
これだけです
315ご冗談でしょう?名無しさん:2012/05/29(火) 14:13:48.76 ID:???
>>314
それだけなら
> ax+ay=0 と近似
できない。
316ご冗談でしょう?名無しさん:2012/05/29(火) 14:17:38.39 ID:???
あら・・・計算ミスってたようですね
どのように近似されますか?
317ご冗談でしょう?名無しさん:2012/05/29(火) 14:36:57.08 ID:???
>>316
>振幅x,yは lに比べて十分小さい
で x''=-k*x, y''=-k*y の形になるだけでは?
318ご冗談でしょう?名無しさん:2012/05/29(火) 14:44:11.82 ID:???
> バッブル望遠鏡は止まってる物しか撮影できない。

じゃあ、何も撮影できない無用の長物じゃん。
319ご冗談でしょう?名無しさん:2012/05/29(火) 15:49:00.44 ID:???
ハッブルで月面を撮影したことはある
http://hubblesite.org/newscenter/archive/releases/1999/14
320ご冗談でしょう?名無しさん:2012/05/29(火) 18:53:57.06 ID:???
何でクッションを挟んで板を叩くと痛くないのですか?
作用反作用の法則より、クッションを伝わって、直に殴った時の
力と同じ大きさの力が手に伝わると思うんですが。

クッションを挟んで板を殴ると、全く痛くないです。
直に殴ると手が折れるくらいに痛いです。
321ご冗談でしょう?名無しさん:2012/05/29(火) 19:26:44.63 ID:???
クッションによって力積が面積的にも時間的にも分散されるからでしょう
322ご冗談でしょう?名無しさん:2012/05/29(火) 20:31:28.71 ID:???
小説板から誘導されてきました
かなり漠然とした内容ですが、質問させてください

宇宙ができた時からすべての原子は一定の法則に従って動いており、
人間も例外ではない。
人生は化学変化の連続に過ぎず、
すべての物質の未来はあらかじめ決まっており、人が心や意志と呼ぶ存在ですら、単なる化学変化に過ぎない

みたいな概念のことを、物理学的には何というのでしょうか?
ものすごく漠然とした質問で心苦しいのですが、どなたかご存知の方がいたら教えてください
323ご冗談でしょう?名無しさん:2012/05/29(火) 20:53:56.53 ID:???
>>322
あんたの未来を決定するほど宇宙は小心者じゃないだろ

ってどう
324ご冗談でしょう?名無しさん:2012/05/29(火) 20:55:27.31 ID:???
決定論だろ
325ご冗談でしょう?名無しさん:2012/05/29(火) 22:41:30.60 ID:???
>>305
真面目に教えてください。
326ご冗談でしょう?名無しさん:2012/05/29(火) 22:50:46.87 ID:0/6NE0o2
>>257に回答できる奴はいないのかw

327ご冗談でしょう?名無しさん:2012/05/29(火) 22:53:29.79 ID:???
粘着(ねんちゃく)とは、特定の物、人物に執拗に嫌がらせ・誹謗中傷などを繰り返すことである。
328ご冗談でしょう?名無しさん:2012/05/29(火) 23:36:50.74 ID:???
86 LEVIN(Wataru) vs めこすじ豆腐店
329ご冗談でしょう?名無しさん:2012/05/29(火) 23:50:28.53 ID:???
マジな話、東京と香港はどっちの方が都会なんでしょうか?
330 忍法帖【Lv=40,xxxPT】 :2012/05/30(水) 00:08:24.80 ID:???
マジな話、東京と香港でどっちが都会だ?なんて面白くもない質問しないでくれるか?
331ご冗談でしょう?名無しさん:2012/05/30(水) 00:19:07.48 ID:???
日本語もまともに使えない馬鹿は書き込まないでくれるか?
332ご冗談でしょう?名無しさん:2012/05/30(水) 01:39:42.02 ID:???
荒らしは書き込まないでくれるか?
333ご冗談でしょう?名無しさん:2012/05/30(水) 01:41:35.10 ID:???
>>322
唯物論と決定論のMIX
334ご冗談でしょう?名無しさん:2012/05/30(水) 01:44:48.06 ID:???
右肘左肘交互に見て。
335 忍法帖【Lv=16,xxxPT】 :2012/05/30(水) 06:55:43.11 ID:???
てす。
336ご冗談でしょう?名無しさん:2012/05/30(水) 07:14:48.41 ID:w6Y1XXF2
抵抗・キャパシタンスの直列回路に100V、3kHzの電圧を加えたところ、大きさ(振幅)5Aの進み電流(π/6)が流れた。

この回路の抵抗とキャパシタンスを教えて下さい!
337ご冗談でしょう?名無しさん:2012/05/30(水) 07:19:34.49 ID:???
>>326 いる。はい次の人
338ご冗談でしょう?名無しさん:2012/05/30(水) 07:20:31.73 ID:???
常温超電導体が出来たら電源切っても扇風機は回り続けるの?

教えて素敵なエロいひと
339ご冗談でしょう?名無しさん:2012/05/30(水) 07:47:11.71 ID:Yu4KUIzv
>>257に回答できる奴はいないのかw
340ご冗談でしょう?名無しさん:2012/05/30(水) 08:14:54.85 ID:???
>>324 >>333
ありがとうございます
普段縁のない学問なので、これから勉強してみます
341ご冗談でしょう?名無しさん:2012/05/30(水) 08:15:43.34 ID:???
てかさ、>>257って、質問??w
>>257に対しては、そうだねとしか言えないと思うけど。。
342ご冗談でしょう?名無しさん:2012/05/30(水) 08:58:00.34 ID:???
プレースホルダなんぞなくとも好きなように配置できるわ、としか言いようがない
343ご冗談でしょう?名無しさん:2012/05/30(水) 10:08:35.09 ID:???
>>338
空気抵抗・軸受けの摩擦などにより、そのうち止まる
344ご冗談でしょう?名無しさん:2012/05/30(水) 10:29:06.56 ID:???
>>338
常温超電導ができてもエネルギー保存則はそのまま。
エネルギー消費すれば有限時間内に止まる
345イムジンリバー ◆NOsnoCwlwY :2012/05/30(水) 12:34:09.33 ID:???
>>318
元々地球の周回軌道にあるから常にガイドしてなきゃならんけど、
他の銀河や恒星メインに観測してる≒止まってる物ってことじゃ?
木製にすい星が衝突したのとかも撮影したけど、
長時間露光できないのでたいしたことなかった
346ご冗談でしょう?名無しさん:2012/05/30(水) 12:52:09.43 ID:???
> 木製にすい星が衝突した

そりゃすごい。
で、どこの木にぶつかったの?
347ご冗談でしょう?名無しさん:2012/05/30(水) 13:22:33.57 ID:???
宇宙を購入したいのですが、購入できるのでしょうか?
また、購入できるとしたら、幾らぐらいなんでしょうか?
348ご冗談でしょう?名無しさん:2012/05/30(水) 14:34:46.56 ID:???
349ご冗談でしょう?名無しさん:2012/05/30(水) 14:37:34.29 ID:???
宇宙の本ではなくて、宇宙そのものを購入できるかという質問です。
また、購入できるとしたら、幾らで購入できるのかが知りたいのです。
真面目に教えてください。
350ご冗談でしょう?名無しさん:2012/05/30(水) 14:38:03.29 ID:1ps0xoPY
http://rocketnews24.com/2010/09/10/%E8%84%B3%E6%B3%A2%E3%81%A7%E3%80%8Eiphone%E3%80%8F%E3%82%92%E6%93%8D%E4%BD%9C%E3%81%99%E3%82%8B%E9%A9%9A%E7%95%B0%E3%81%AE%E6%A9%9F%E5%99%A8%E7%99%BA%E5%A3%B2%EF%BC%81-%E9%A0%AD%E3%81%A7%E8%80%83/
アメリカのある技術開発会社が、手を使わずにスマートフォン『iPhone』、デジタル音楽プレーヤー『iPod Touch』、タブレット型コンピュータ『iPad』を操作できる技術を開発した。
http://www.pickandbuzz.com/?p=4121
タッチ操作はもう古い!? 脳波でiPhoneをコントロールする「XWave」
NTTの携帯
NECのキャンビーの会話通話とプログラム
あと、プログラム組めば、脳波で通話、会話できるヘッドホンができる。人体実験なし。
登記
351ご冗談でしょう?名無しさん:2012/05/30(水) 14:40:09.45 ID:???
できません。はい次の人
352ご冗談でしょう?名無しさん:2012/05/30(水) 14:41:15.28 ID:???
真面目に教えろヴォケ。
353ご冗談でしょう?名無しさん:2012/05/30(水) 14:44:53.10 ID:???
真面目に教えたのにwww
354ご冗談でしょう?名無しさん:2012/05/30(水) 14:52:10.24 ID:???
孔子様とアルキメデス様はどっちの方が天才なんでしょうか?
理由付きで詳しく教えてください。
355ご冗談でしょう?名無しさん:2012/05/30(水) 14:54:22.90 ID:1ps0xoPY
通話名  フォン 登記
356ご冗談でしょう?名無しさん:2012/05/30(水) 14:56:21.12 ID:???
登記さん、>>354の質問に答えてください。

あと、アルキメデス様とエウクレイデス様はどっちの方が天才なんでしょうか?
教えてください。
357イムジンリバー ◆NOsnoCwlwY :2012/05/30(水) 15:42:52.64 ID:???
>>349
購入はできないけど、価格をつけるなら、宇宙の全エネルギーが2.4×10^80GeV
エネルギーと価格の変換式は、例えば東京電力なら1kwh=0.5円くらい
あとは単位の変換と乗除算だけなので自分でどうぞ
但しこれは、その辺の石ころも全てエネルギーに換算してるので、かなりのプレミア価格
358イムジンリバー ◆NOsnoCwlwY :2012/05/30(水) 15:46:00.82 ID:???
>>356
「天才」を数値化できないから比較はムリ
IQが高いか?とかなら、推測はできるかもしらんが
359ご冗談でしょう?名無しさん:2012/05/30(水) 16:10:18.44 ID:???
(((゜д゜;)))1Kwh=0.5円

どこの先進国だよ
360 忍法帖【Lv=4,xxxP】 :2012/05/30(水) 16:25:40.39 ID:???
波動エネルギーの様な無尽蔵エネルギーを考えて欲しいですよ。
361ご冗談でしょう?名無しさん:2012/05/30(水) 20:48:42.18 ID:???
>>342
う、うむ
割とガチで空白レイアウトで使うことが多い
362ご冗談でしょう?名無しさん:2012/05/30(水) 20:49:54.56 ID:???
>>360
無尽蔵ではないと思いました
363ご冗談でしょう?名無しさん:2012/05/31(木) 00:09:07.35 ID:???
あの水の細いののカッターみたいので
タービン回す事は出来ないんですか?
あれで循環してれば永久エネルギーかと
364ご冗談でしょう?名無しさん:2012/05/31(木) 00:14:54.57 ID:???
「あの」って?
365ご冗談でしょう?名無しさん:2012/05/31(木) 00:19:12.64 ID:???
ウォーターカッターのことか
366ご冗談でしょう?名無しさん:2012/05/31(木) 00:22:26.49 ID:???
タービンも切れるから回せない。
367ご冗談でしょう?名無しさん:2012/05/31(木) 00:24:07.37 ID:???
あのその切れないようにというか上手い事
何ていうか説明しにくいですが無理なんでしょうね
368ご冗談でしょう?名無しさん:2012/05/31(木) 00:29:15.70 ID:???
ジェットの当たる部分だけ特殊に加工してみたり
まあ発電のタービンもウォータジェットも全然わからんのですが
クルクル回りそうな気がしてw
369ご冗談でしょう?名無しさん:2012/05/31(木) 00:33:36.61 ID:???
それが船なのかort
370ご冗談でしょう?名無しさん:2012/05/31(木) 00:44:28.25 ID:???
いいかげんマジレスしとくが、水を噴き出すエネルギーは
どこから来ると思っているのかと。
371ご冗談でしょう?名無しさん:2012/05/31(木) 05:51:32.02 ID:???
>>358
じゃあIQを教えてくれ。
372ご冗談でしょう?名無しさん:2012/05/31(木) 06:43:14.80 ID:???
ゲッターエンペラーのIQはどのくらいなんでしょうか?
誰か真面目に教えてください。
373ご冗談でしょう?名無しさん:2012/05/31(木) 07:11:43.46 ID:Oc+rgGMr
ヘレンケラーも、会話できるよ。この機械.
374ご冗談でしょう?名無しさん:2012/05/31(木) 08:02:55.87 ID:Oc+rgGMr
>>350
これか http://www.nec.co.jp/solution/telescouter/index.html これhttp://www.nec.co.jp/press/ja/1110/1701.htmlを上に取りつけて、プログラムして、?げて、視覚化もできるy 登記
375ご冗談でしょう?名無しさん:2012/05/31(木) 08:23:17.74 ID:???
香港の俳優になりたいのですが、日本人じゃ無理ですか?
また、香港のマフィアにも入りたいのですが、日本人じゃ無理ですか?
可能だとしたら、それぞれどうすれば良いのでしょうか?教えてください。
376ご冗談でしょう?名無しさん:2012/05/31(木) 08:57:52.14 ID:Oc+rgGMr
>>350
これか http://www.nec.co.jp/solution/telescouter/index.html これhttp://www.nec.co.jp/press/ja/1110/1701.htmlを上に取りつけて、プログラムして、?げて、プログラムして、視覚化もできるy 登記
377ご冗談でしょう?名無しさん:2012/05/31(木) 08:58:23.30 ID:Oc+rgGMr
>>350
これか http://www.nec.co.jp/solution/telescouter/index.html これhttp://www.nec.co.jp/press/ja/1110/1701.htmlを上に取りつけて、プログラムして、付けて、プログラムして、視覚化もできるy 登記
378ご冗談でしょう?名無しさん:2012/05/31(木) 08:59:51.05 ID:Oc+rgGMr
?は、?げて
379ご冗談でしょう?名無しさん:2012/05/31(木) 09:00:07.35 ID:Oc+rgGMr
?は、つなげて
380ご冗談でしょう?名無しさん:2012/05/31(木) 09:01:58.48 ID:???
無限大のラオウと無限大のゲッターエンペラーが戦ったらどっちが勝つのでしょうか?
381ご冗談でしょう?名無しさん:2012/05/31(木) 09:06:51.62 ID:???
生きている限りは絶対たどり着けない
それが無限
宇宙になれ
382ご冗談でしょう?名無しさん:2012/05/31(木) 09:08:00.07 ID:???
ゼウスとガウスはどっちの方がIQが高いのでしょうか?
383ご冗談でしょう?名無しさん:2012/05/31(木) 09:19:29.73 ID:???
宇宙は有限。
これは常識中の常識。
384ご冗談でしょう?名無しさん:2012/05/31(木) 09:21:04.28 ID:???
考えてるから有限なんだよ
自分の知覚がとても小さい事を知れ
385ご冗談でしょう?名無しさん:2012/05/31(木) 09:24:46.56 ID:???
「全て」と「無」はどっちの方が至高なんでしょうか?
いい加減決着をつけたいです。
誰かヘルプミー。
386ご冗談でしょう?名無しさん:2012/05/31(木) 09:55:11.00 ID:???
六本木ヒルズのレジデンスの最上階に住みたいのですが、
どのような職業に就けば良いのでしょうか?
やはり、実業家としてある程度成功しないと無理なんでしょうか?
それ以外に道はないのでしょうか?
387ご冗談でしょう?名無しさん:2012/05/31(木) 10:24:16.59 ID:Oc+rgGMr
ダイバーシステム 
3D、4Dの立体テレビシステムに((商)丸紅開発など)に、光水を付けて、通話、視聴機械システムいれれば、魔法世界作れる 登記
388ご冗談でしょう?名無しさん:2012/05/31(木) 10:33:43.45 ID:???
登記さん、>>385-386の質問に答えてください。お願いします。
389ご冗談でしょう?名無しさん:2012/05/31(木) 10:35:56.18 ID:???
>>386

別に何の職業にもつかなくても住めるよ。金さえ出せば。
俺は無職だけど住もうと思えば簡単に住める。そんなことに金を使おうと思わないけどね。
390 忍法帖【Lv=4,xxxP】 :2012/05/31(木) 10:37:18.42 ID:???
無限大の松本零士と無限大の槇原敬之が対決するとどうなるのよ?。
391ご冗談でしょう?名無しさん:2012/05/31(木) 11:06:23.25 ID:???
>>389
いや、だから、その金が必要なわけじゃん。
その金を手に入れるにはどんな職業に就けば良いのかという質問。
392イムジンリバー ◆NOsnoCwlwY :2012/05/31(木) 11:33:56.76 ID:???
>>359
5円くらいに訂正♪
393イムジンリバー ◆NOsnoCwlwY :2012/05/31(木) 11:36:48.56 ID:???
>>384
とりあえずビッグバン宇宙論が正しいと仮定すれば、
宇宙は有限、但しその果てを我々が認知することは難しい
394ご冗談でしょう?名無しさん:2012/05/31(木) 11:38:38.99 ID:???
イムジンリバーさん、>>385-386の質問に答えてください。お願いします。
395イムジンリバー ◆NOsnoCwlwY :2012/05/31(木) 11:52:55.32 ID:???
>>394
>>385は知らん、そもそも全てと無に優劣の関係はない
>>386>>389のいうとおり、職業とカネ持ちかどうかにも確たる相関関係はない
まあ職業別に統計をとれば優劣の差がでるかもしらんが、あくまで平均値でしかない
396イムジンリバー ◆NOsnoCwlwY :2012/05/31(木) 12:34:37.96 ID:???
質問がありマス
ウチにアナログの掛け時計があって、1秒に1回カチッと動く
フとその時計に目をやってから、秒針が動くまでの時間は0〜1秒のハズよね
ところがマレに、フと目をやってから2秒くらい(感覚的に)経ってから動くことがある
気のせいかなと思って友人にそのハナシをしたら、半数が「あるある!」と賛同してくれた
これって何か理由があるのかな?(パッと見たときの画像処理に時間がかかるとか)
それとも気のせいなのかな?
397ご冗談でしょう?名無しさん:2012/05/31(木) 12:49:28.94 ID:???
>>396
それって意外と深い認知科学の話だったような気がする。
398ご冗談でしょう?名無しさん:2012/05/31(木) 13:50:58.58 ID:???
>>396
瞬間的な脳のクロックアップと関係あるのかもしれん
優秀なプロスポーツ選手はそれを意識的に準備できたり
399ご冗談でしょう?名無しさん:2012/05/31(木) 14:40:03.00 ID:???
>>396
主観的時間じゃない?
400ご冗談でしょう?名無しさん:2012/05/31(木) 15:31:47.00 ID:r6VAGEJA
>>396
こんなことはあるでしょうか?
目をやったときからわずかの時間は秒針は認識されてないので動きも認識されてない。
頭の中では目をやったときに認識したことになっているように辻褄を合わせている。
401ご冗談でしょう?名無しさん:2012/05/31(木) 16:51:54.94 ID:WVcy7UMz
大学1年です。
空気抵抗を考慮した落下運動の微分方程式の解き方が分かりません。粘性抵抗、慣性抵抗の片方だけを考慮した方程式の解き方は分かるんですが、両方考慮する場合はどうすればよいのか分からず困っています。
dv/dt=g-kv-Kv^2 k,Kは違う比例定数です。
どのようにして解けばいいのでしょうか?
402ご冗談でしょう?名無しさん:2012/05/31(木) 16:55:14.33 ID:???
>>401
ヒント:部分分数分解
403ご冗談でしょう?名無しさん:2012/05/31(木) 17:02:32.93 ID:???
積分するだけだから片方の場合と変わらん
分数の積を和になおすやつ
高校でよくやるあれ
404ご冗談でしょう?名無しさん:2012/05/31(木) 17:10:30.36 ID:phABPI8V
材料力学の段付き棒の引っ張り問題なんだけど
分からん
外力F=2000N ポアソン比=0.3 ヤング率=206GPa 丸棒の直径d1=8o d2=6o 棒の長さl1=13o l2=14o
の時の段付き棒の伸びλおよびそれぞれの棒の直径の変化量δ1、δ2を求めよ
ヒント
λ=λ1+λ2=(P×l1/A1×E1)+(P×l2/A2×E2)
単位はμmとする
405ご冗談でしょう?名無しさん:2012/05/31(木) 17:33:41.34 ID:???
>>275
>白紙のレイアウトでは、パワポでは文字など入力できないはずだが?
>どうやって白紙に直接入力するのでしょうか?

白紙のレイアウトからスタートして、
自分でテキストボックスを作って入力するなり、
他からコピペするなりで、できたボックスの書式を変えればいいだろ。
画像なら挿入を使えばいいわけだし。

レイアウトにあらかじめ作り付けとしてあるプレースホルダの必要はないでしょ。

それがどうしてもいやならマスタスライドを自分の都合の良いように編集すれば?

いずれにしても、数式エディタは進化が止まっているのでおすすめしない、
数式エディタぽいものにこだわるなら、
数式エディタの開発元が独自にバージョンアップしているMathTypeを購入した方が
時間が買えると思うわ。
406ご冗談でしょう?名無しさん:2012/05/31(木) 17:48:48.14 ID:???
自己解決しました
407ご冗談でしょう?名無しさん:2012/05/31(木) 19:57:28.74 ID:???
>>396
それあるわ
一瞬時計が止まったのか?と思ったら普通に動いてる
最初のタイミングだけ2、3秒止まって見える
408ご冗談でしょう?名無しさん:2012/05/31(木) 21:28:43.18 ID:vRsAYipq
>>405
>自分でテキストボックスを作って入力するなり、
他からコピペするなりで、できたボックスの書式を変えればいいだろ。
画像なら挿入を使えばいいわけだし。


POWERPOIMTが使えねえのは、そのテキストボックスに
数式(数式エディタ等で作ったもの)を挿入できないところ
数式に使用している変数の解説とか同じテキストボックスに
文字と数式と入れられないので面倒くせえ
409ご冗談でしょう?名無しさん:2012/05/31(木) 21:54:43.31 ID:???
>>408
そんなソフト見たことも聞いたこともないずら
410ご冗談でしょう?名無しさん:2012/05/31(木) 21:56:33.01 ID:???
>>409
パワーポイントだよ。nがmになっているが。
411ご冗談でしょう?名無しさん:2012/05/31(木) 22:44:02.43 ID:???
>>396
あるあるwwwwww
ググったら「クロノスタシス」とかなんとかかんとか
412ご冗談でしょう?名無しさん:2012/05/31(木) 22:52:46.90 ID:ot9H2kIb
次の二点がわかりません。
何方か教えていただけないでしょうか?

1効力の式の導出方法
効力F=Cd*S*ρ*v^2
Cd;抵抗係数 S:面積 ρ:密度 v:速度
これがどう導き出されるのかわかる方はおられますか?

2抵抗係数Cdの求め方
Cdとレイノルズ数やマッハ数との関係式としてどういったものがあるんでしょうか?
知っている方がいたら教えて下さい。
413ご冗談でしょう?名無しさん:2012/06/01(金) 00:24:37.35 ID:???
>>396
おれもあるw
事実じゃないが真実だ
414ご冗談でしょう?名無しさん:2012/06/01(金) 02:51:43.72 ID:1DWfVHrV
最近になって速度の二乗に比例する空気抵抗を
考慮したときの質点の運動方程式の解析解が
発見されたみたいですが詳細を知ってる方はいらっしゃいますか?
415ご冗談でしょう?名無しさん:2012/06/01(金) 03:43:07.70 ID:???
>>412
時間Δt の間に当たる空気の体積は S v Δt、質量は m=ρ S v Δt
運動量は m v=ρ S v^2 Δt
力は F=m v/Δt=ρ S v^2
416ご冗談でしょう?名無しさん:2012/06/01(金) 05:22:20.91 ID:1FluKoyc
進化系はみがきグッズ日テレ放送で、バッテリー等応用できるよ。
                                     ビビオ
417ご冗談でしょう?名無しさん:2012/06/01(金) 05:36:49.44 ID:???
にほんごでしゃぶってくらはい
418 忍法帖【Lv=5,xxxP】 :2012/06/01(金) 05:39:52.74 ID:???
クロノトリガー。
419ご冗談でしょう?名無しさん:2012/06/01(金) 06:23:14.61 ID:1FluKoyc
420ご冗談でしょう?名無しさん:2012/06/01(金) 06:32:46.05 ID:???
1Wじゃチリも焦げないね
421ご冗談でしょう?名無しさん:2012/06/01(金) 09:18:18.89 ID:???
■ちょっとしたメコスジ道の膣悶はめこに掻いてね69■
422ご冗談でしょう?名無しさん:2012/06/01(金) 09:19:22.99 ID:???
>>414
最近になって?ニュートン・ライプニッツのころに既に知られてますが
423ご冗談でしょう?名無しさん:2012/06/01(金) 09:32:37.01 ID:???
>>422
落下運動なら解析的に解けるが、2次元は解けない。
ttp://www.geocities.jp/hydrodynamism/ProjectileMotion/index.html
424イムジンリバー ◆NOsnoCwlwY :2012/06/01(金) 16:53:22.05 ID:???
回答くれたひと、ありがと♪
クロノスタシスみたいね
425ご冗談でしょう?名無しさん:2012/06/01(金) 16:57:59.19 ID:???
まやかしの地_方_分_権

テレビで韓国ドラマばかり流れても、見なければいいだけなので大きな問題はありません。  
しかし地方分権で警察組織、権力を地方の犯罪については国から委譲し
採用条件、組織等も地方で決めれるようにたらどうなるでしょう。
今のテレビ局が数十年前に在日枠を受け入れて、今や完全に在日朝鮮人に乗っ取られ
在日の都合の悪い報道は一切しなくなり、反日政党民主党が与党になったように
地方分権された警察組織が数年後に、反日感情を持った外国人に支配されたらどうなるでしょう。
在日の犯罪は取り締まられず、日本人の犯罪は過大な罰を与えられたりしないと言い切れるでしょうか。
地方分権は日本全国の地方自治体を、反日国家のコントロール化に置くための工作活動に等しいのです。。

維新に近い、みんなの党は道州裁判所を設ける案もだしてます。
橋下氏(維新)の大阪都構想しかり
中京圏の大村氏、河村氏も地域政党を作って国政で候補者をだすみたいです。
まだ時間はあります、一度じっくり検討してください。
426ご冗談でしょう?名無しさん:2012/06/01(金) 18:29:16.00 ID:xI2NPN3a
そもそもtexにプレゼンレイアウトの書式機能を
強化すべきだよ。

それかpowerpointにtexそのものとの互換性を持たせるか。
427ご冗談でしょう?名無しさん:2012/06/01(金) 19:48:16.65 ID:???
>>426
TeXならマクロ次第では?
428ご冗談でしょう?名無しさん:2012/06/01(金) 20:15:48.26 ID:???
BeamerだのPowerdotだの既にあるだろ
429ご冗談でしょう?名無しさん:2012/06/01(金) 21:09:00.58 ID:???
>>428
プレゼンスライドショーの性能だけを考えれば、
それらはpowerpointにまだ水をあけられている。
430ご冗談でしょう?名無しさん:2012/06/01(金) 21:28:13.15 ID:ry7XM8r2
無粒子から、有をつくれすか? 物質までつくれますか?
431 忍法帖【Lv=6,xxxP】 :2012/06/01(金) 21:33:37.98 ID:???
勿論作れます。
432ご冗談でしょう?名無しさん:2012/06/01(金) 21:47:02.86 ID:ry7XM8r2
中間子の無粒子を時計回りに中監視の無粒子を回して、エネルギーを発生させ、エネルギー粒子から、有粒子を作りだし、エネルギーを増大させ、エネルギー粒子を、増加させ、有粒子を多くして、無粒子より有粒子が超えて、
多くして有粒子と無粒子をあわせて消滅させ、有粒子を多くし、物質化させる
登記
433ご冗談でしょう?名無しさん:2012/06/01(金) 21:50:20.35 ID:???
>>429
そりゃ当たり前だ。プレゼンソフトのプレゼン性能が組版ソフトに負けちゃいかんだろ。
434ご冗談でしょう?名無しさん:2012/06/01(金) 22:05:24.52 ID:???
435ご冗談でしょう?名無しさん:2012/06/01(金) 22:10:36.93 ID:???
>>433
そりゃそうなんだが、せめて現時点での
powerpointクラスの性能は欲しい。
Beamerに頑張って欲しいね。
436ご冗談でしょう?名無しさん:2012/06/01(金) 22:29:21.45 ID:???
PowerpointもWordやTeXでつくったもので
容易に切り貼りで作れたらまだ使い勝手いいのに。
現在のバージョンでも切り貼りが面倒臭い。
437ご冗談でしょう?名無しさん:2012/06/01(金) 22:50:07.19 ID:ry7XM8r2
中間子の無粒子を時計回りに中間子の無粒子を回して、エネルギーを発生させ、エネルギー粒子から、有粒子を作りだし、エネルギーを増大させ、エネルギー粒子を、増加させ、有粒子を多くして、無粒子より有粒子が超えて、
多くして有粒子と無粒子をあわせて消滅させ、有粒子を多くし、物質化させる
登記
438ご冗談でしょう?名無しさん:2012/06/01(金) 22:54:16.61 ID:???
おお、言葉の意味はよくわからんがとにかくすごい自信だ
439ご冗談でしょう?名無しさん:2012/06/02(土) 00:03:37.67 ID:???
別にTeXで全く困ってないんだけど、一体世の人はパワポに何求めてんの??
研究発表にアニメーションなんか要らんだろ
440ご冗談でしょう?名無しさん:2012/06/02(土) 00:05:00.92 ID:???
資料のやりとりにこまんだよ
441ご冗談でしょう?名無しさん:2012/06/02(土) 00:08:48.00 ID:???
>>440
PDFにすりゃ無問題
442ご冗談でしょう?名無しさん:2012/06/02(土) 00:14:32.66 ID:sxDgIl8L
無空間を時計回りに回して、エネルギーを発生させ、エネルギー粒子から、有粒子を作りだし、エネルギーを増大させ、エネルギー粒子を、増加させ、有粒子を多くして、無空間より有粒子が超えて、
多くして有粒子を無空間にを多くし、物質化させる
登記
443ご冗談でしょう?名無しさん:2012/06/02(土) 00:31:10.39 ID:???
メコスジブクロ
444ご冗談でしょう?名無しさん:2012/06/02(土) 02:52:46.33 ID:???
言葉の意味が分かってるとは思えんな
445 忍法帖【Lv=6,xxxP】 :2012/06/02(土) 04:42:20.10 ID:???
勿論素晴らしい物凄い物理学は美しいですよ。
446ご冗談でしょう?名無しさん:2012/06/02(土) 09:02:09.92 ID:???
>>439
ちょっとした研究会程度だと、プレゼン作成を簡単に済ませたいので
TEXで作ったものを直接powerpointで簡単に直接切り貼りできればそれに越したことは無い。
今のpowerpointでは簡単に直接切り貼りしづらい。
447ご冗談でしょう?名無しさん:2012/06/02(土) 09:39:49.85 ID:???
TeXでプレゼンて、レイアウトに無駄に時間使わないか?
1ページに二行おさまらん・・・とか。
448ご冗談でしょう?名無しさん:2012/06/02(土) 10:00:13.27 ID:???
質問です
角加速度影響定数とはなんですか?
449ご冗談でしょう?名無しさん:2012/06/02(土) 12:09:05.15 ID:???
>>448 そのような述語は物理にはない。ほかを当たってくれ。
450ご冗談でしょう?名無しさん:2012/06/02(土) 13:28:12.88 ID:???
回転数が変わると遠心力やコリオリ力が変わるから機械装置のバランスが変わって色々と影響が出るんだろうな。
機械によって影響が違うだろうから、それを特定しなきゃ角加速度影響定数の意味なぞ分からんね。
451ご冗談でしょう?名無しさん:2012/06/02(土) 17:03:12.40 ID:???
powerpointにテキストボックスなんて明らかに不要。
プレースホルダをテキストボックスのように
簡単に範囲をしていして挿入させる方がよっぽど在り難い。
もちろん、プレースホルダの中をword化して文字・数式・図画を
全て入るようにして欲しい。
452ご冗談でしょう?名無しさん:2012/06/02(土) 17:14:27.66 ID:???
それだとアウトラインのところに長々と文章が出てくるぞ。

アウトラインなんて見ねぇ、というスタイルで、プレースホルダか
テキストボックスのどちらか一方があればいい、と主張するならわかるが、
プレースホルダでなきゃダメ、というのは理解できんな
453ご冗談でしょう?名無しさん:2012/06/02(土) 17:15:26.82 ID:???
テスト
454ご冗談でしょう?名無しさん:2012/06/02(土) 17:24:11.17 ID:???
抵抗力のある鉛直上向き投げ上げについて教えて下さい
質量m、比例係数mλ、x軸を上向きにとり重力加速度gのとき
運動方程式は場合分けが必要ですか?どういった式になるのでしょうか
あと十分時間経過後の速度は場合分けして落ちていく時の式?をつかうのでしょうか
すみませんどなたか分かる方ご教授お願いします
455ご冗談でしょう?名無しさん:2012/06/02(土) 17:39:24.11 ID:???
数式以外の部分をわざわざTeXで作って貼り付るメリットがわからん。
で、数式だけならTeXclipあるし・・・

論文スタイルのレイアウトを作りたいならTeXは最強だけど、
プレゼン用のレイアウトとはおのずと違うもの
456ご冗談でしょう?名無しさん:2012/06/02(土) 18:10:20.89 ID:???
>>455
論文から発表用のプレゼンスライド作る
場合がほとんどだから、切り貼りできるに越したこと無い。
457ご冗談でしょう?名無しさん:2012/06/02(土) 18:19:00.38 ID:???
>>454
場合分けなんてイランでしょ。
十分時間が経ってる場合の速度は、力の釣り合いでOK。
458ご冗談でしょう?名無しさん:2012/06/02(土) 18:34:43.03 ID:???
上昇と落下では抵抗力の方向が逆だから場合分けは必要では?
459ご冗談でしょう?名無しさん:2012/06/02(土) 18:54:57.16 ID:???
>>458
抵抗って速度に反する力だろ。
バネと錘の振動でも力の方向は変わるが、運動方程式分けるか?
460ご冗談でしょう?名無しさん:2012/06/02(土) 19:01:20.05 ID:???
単振動はたまたま変位に比例する力だから単純な式に帰着したけど、速度の方向に依存する抵抗力は単振動のようにはならない。
461ご冗談でしょう?名無しさん:2012/06/02(土) 19:04:41.11 ID:???
>>452
>プレースホルダか
テキストボックスのどちらか一方があればいい、と主張するならわかるが、


プレースホルダかテキストボックスかだったら、
考えるまでも無くプレースホルダに軍配を挙げるだろ。
プレースホルダの方が階層分けしやすいし。
テキストボックスってそれらの機能皆無だからw


アウトライン?端から見ねーよw
462ご冗談でしょう?名無しさん:2012/06/02(土) 19:05:19.93 ID:???
そういえば、減衰振動って速度に比例する抵抗しか見たことないけど、
速度の二乗に比例する抵抗で解いている教科書とか演習書とかってあるのかな。
あるいはPCで解く物理みたいな本とか。
463ご冗談でしょう?名無しさん:2012/06/02(土) 19:12:34.50 ID:???
>>460
質問者の言う「抵抗力」が空気抵抗の様に速度に反する力として記述出来るのもなら、
運動方程式も同様に場合分けなどせず記述出来る。
464ご冗談でしょう?名無しさん:2012/06/02(土) 19:25:35.35 ID:???
>>46
なるだろ。-kvでいれとけは自動的に正しい方を向く。
k v ^2なら場合わけ必要。
465ご冗談でしょう?名無しさん:2012/06/02(土) 20:00:30.96 ID:???
やはり線形は扱いやすいな
466ご冗談でしょう?名無しさん:2012/06/02(土) 20:05:35.48 ID:???
最後までチョコたっぷりだもんな
467ご冗談でしょう?名無しさん:2012/06/02(土) 20:16:36.34 ID:???
>>462
物体に働く抵抗は速度が遅いときは速度に比例し、速いときは速度の二乗に比例する。
振動の場合、最大速度に近いときに抵抗が速度の二乗に比例したとしても、速度がゼロに
近い状態もあるから単純には行かない。
468ご冗談でしょう?名無しさん:2012/06/02(土) 20:21:34.19 ID:???
>>467
そうか。それ忘れてた。
レイノルズ数か何かが必要なわけか。流体しらんけど。
そもそも、二乗から一乗にデジタル的に切り変わるわけもないか。
数値計算以外無理そうだな・・・・
469ご冗談でしょう?名無しさん:2012/06/02(土) 21:12:11.33 ID:???
powerpointはwordで作成したものでさえ
切り貼り面倒だからな。
学部生の時、wordでレジュメを作っていたんだが
powerpointでプレゼンしてくれといわれたので、
それを文字と数式をまとめて切り貼りしようと思ったら、
文字だけしか貼れなかったのでそのpowerpointの糞さに辟易したw
まあ、それからはtex+beamerで論文とスライド作成するように
なったからよかったけど。
せめて、powerpointとwordくらいはもっと効率的に
切り貼りできるようにするべきだろ。
470ご冗談でしょう?名無しさん:2012/06/02(土) 21:13:40.82 ID:???
もりあがってるとこわりぃ、慣れの問題
471ご冗談でしょう?名無しさん:2012/06/02(土) 21:31:16.10 ID:???
もりあがってるとこわりぃ、すれち
472ご冗談でしょう?名無しさん:2012/06/02(土) 22:01:53.49 ID:???
電気回路で格子形回路のFパラメータを求める問題です
インピーダンスの位置は

   Z1
Z2 Z3
Z4

左側が1-1'間V1、右側が2-2'間V2
ここで、パラメータのBを求める時に、V2=0(短絡)としてV1/I2で出しますよね?
その時どうやって値を出すか迷っています
並列回路のように変形して

___Z1_________ Z3___
l
___Z2___l______Z4____

この形が意味がわかりません…
2-2'間を短絡させたらこうなりますよね?これでI2を求めるのが謎です
どうやって最終的にBを求めれば良いのでしょうか?
473472 図が間違えました:2012/06/02(土) 22:04:07.74 ID:???
   Z1
Z2    Z3
   Z4


___Z1______Z3___
     l
___Z2___l___Z4___
474ご冗談でしょう?名無しさん:2012/06/02(土) 22:27:38.54 ID:???
>>408
>数式(数式エディタ等で作ったもの)を挿入できないところ

だからさ、俺はイライラしたんでMathTypeにスイッチしたんだよ。
MathType入れると、PowerPointにMathTypeメニューが出来て、
そこから好きに数式挿入できるし再編集もできるからさ。

俺はパワポとかKeynoteとかMacでしか使わんが、
Windows版でもできるだろ?

そもそもマイクロソフトの数式エディタとMathTypeは、
同じコードから派生したが、
マイクロソフトはやる気なしのメンテナンスモードでぶん投げているからさ。
数式エディタってMathTypeのモンキーバージョンだもの、
正直、関わり合いになりたくない。
475ご冗談でしょう?名無しさん:2012/06/02(土) 22:35:22.28 ID:???
安上がりなら、やっぱTeXで数式でしょ。
476472:2012/06/02(土) 22:52:21.10 ID:???
I2=1と求められました!
最終的にBパラも求められて解いていったら答えとあっててよかった…
477ご冗談でしょう?名無しさん:2012/06/02(土) 23:16:37.13 ID:???
物理実験のデータの取り扱いについて質問です。
ある長さの測定を行い,3.123のような,1桁+小数第3位までのデータが10個くらい得られたとします。
しかし実験の目的はこの値を求めることではなくて,これらのデータを元に処理した(2乗したり,測定値同士を足し算したり…)値を報告することです。
この場合,たとえば計算過程で測定値の2乗が必要となるとします。
データが3.123ならば,2乗すると9.753129となりますね。これをレポートではそのまま報告すべきでしょうか?
それとも,元のデータが4桁なので,4桁目までで十分でしょうか?

実験中に計算もやったのですが,その時は,誤差がどのくらいかわからない=有効数字がどのくらいかわからないから
記録には電卓で出た値を全て書き,それを元に計算したのですが…レポートにどう書くべきか,教えてください
478ご冗談でしょう?名無しさん:2012/06/02(土) 23:17:36.23 ID:???
>>474
PowerPointに数式挿入はできる。
しかし、>>408の言っていることは、
テキストエディタやプレースホルダに数式を
挿入できない点だ。
479ご冗談でしょう?名無しさん:2012/06/02(土) 23:22:06.44 ID:???
そろそろぷいたへ
480ご冗談でしょう?名無しさん:2012/06/02(土) 23:22:52.83 ID:???
>>477
有効数字の計算を中学からやり直せ。
481ご冗談でしょう?名無しさん:2012/06/02(土) 23:31:02.80 ID:???
>>477
有効数字4ケタ同士のかけ算なら答も4ケタ。
足し算や引き算は単純じゃないから注意。
482ご冗談でしょう?名無しさん:2012/06/02(土) 23:43:08.36 ID:???
>>477
>3.123のような,1桁+小数第3位までのデータ
誤差がどのくらいかわからないのなら、そもそもこう言えないはずだが。
483ご冗談でしょう?名無しさん:2012/06/02(土) 23:45:26.12 ID:???
>>477
測定値が3.123で誤差がδならば、2乗すると(3.123+δ)^2 = 3.123^2 + 2*3.123*δ + δ^2 となります。
普通はδ^2は無視すると誤差は2*3.123*δとなりますδ=0.0005と仮定すると誤差は0.003123であり、
小数点以下3桁でも3くらいの誤差となる。
一般には微分で計算し、f(x)の誤差は|f'(x)|Δxとなる。Δxについては測定のばらつきの標準偏差を使う。
実験結果のレポートならば誤差の評価をきちんとしないといけない。

484ご冗談でしょう?名無しさん:2012/06/02(土) 23:54:26.77 ID:???
>>480-483
ありがとうございました。
485ご冗談でしょう?名無しさん:2012/06/03(日) 00:32:01.24 ID:???
太陽の膨大な燃焼エネルギーは一体どこに消えるんですか?
486ご冗談でしょう?名無しさん:2012/06/03(日) 00:40:54.42 ID:???
>>485
別に消えないさ
一部は人類の生命維持にしっかり使われてるし
残りは、宇宙へ放射されて、太陽系の惑星はじめ宇宙全体を温めているのさ
487ご冗談でしょう?名無しさん:2012/06/03(日) 02:20:39.11 ID:???
地球上では空気も地球と一緒に自転してます
だからコリオリの力は働きませんですね?
488ご冗談でしょう?名無しさん:2012/06/03(日) 04:00:15.59 ID:???
真空にしたガラス瓶の中なら働くのか
489ご冗談でしょう?名無しさん:2012/06/03(日) 04:54:51.17 ID:???
>>487
台風の進路がコリオリ力で曲がると言う話を聞いた事ないか?
TVでフーコーの振り子がコリオリ力の証拠になってるのを見たことないか?(真空じゃないぞ)
490 忍法帖【Lv=8,xxxP】 :2012/06/03(日) 09:40:55.57 ID:???
リアルエル・シャダイ。
491ご冗談でしょう?名無しさん:2012/06/03(日) 10:08:42.26 ID:???
今年の高校1年生から高校数学の行列が消えるらしいですが、物理の人は困りませんか?
大学に入ってから行列の概念を一から教える・・・・・無理でしょ。
492ご冗談でしょう?名無しさん:2012/06/03(日) 10:58:57.95 ID:???
代わりに何やるかによる

高校の行列は数字の塊を機械的にいじるだけで意味わかんないし
大学からでも問題ないといえばない

行列消して二重積分とかベクトルの外積とかやってくれるほうがいい
493ご冗談でしょう?名無しさん:2012/06/03(日) 11:23:02.58 ID:oUt0FaYw
http://getnews.jp/archives/63552

アメリカのある技術開発会社が、手を使わずにスマートフォン『iPhone』、デジタル音楽プレーヤー『iPod Touch』、タブレット型コンピュータ『iPad』を操作できる技術を開発した。
http://www.pickandbuzz.com/?p=4121
タッチ操作はもう古い!? 脳波でiPhoneをコントロールする「XWave」
NTTの携帯
NECのキャンビーの会話通話とプログラム
あと、プログラム組めば、脳波で通話、会話できるヘッドホンができる。人体実験なし。
登記
これか http://www.nec.co.jp/solution/telescouter/index.html これhttp://www.nec.co.jp/press/ja/1110/1701.htmlを上に取りつけて、プログラムして、?げて、プログラムして、視覚化もできるy 登記
494ご冗談でしょう?名無しさん:2012/06/03(日) 11:28:15.05 ID:???
せめて自分で考えた事を登記しろよw
495ご冗談でしょう?名無しさん:2012/06/03(日) 11:36:07.72 ID:???
>>492
代わりに入るのはガウス平面
496ご冗談でしょう?名無しさん:2012/06/03(日) 11:38:47.16 ID:???
座標の回転はこの行列でっていうと、行列は知らん、とか。
量子力学で固有値とかっていうと、知らん、ぼけ〜〜〜とされるようになるんですけどね。
497ご冗談でしょう?名無しさん:2012/06/03(日) 11:41:08.06 ID:???
外積を教えるのこうやって行列式で覚えると簡単(サラスの方法)…っていうのも出来なくなる。
498ご冗談でしょう?名無しさん:2012/06/03(日) 12:18:31.31 ID:???
以前から2×2を超えるサイズの行列は加減と乗算しか扱ってないのでどっちにしろそれは無理
代わりに増えたのは複素平面と統計で、特に統計は多くの高校生に教える意義あるからいいだろ
499ご冗談でしょう?名無しさん:2012/06/03(日) 12:21:45.08 ID:oUt0FaYw
500ご冗談でしょう?名無しさん:2012/06/03(日) 12:32:42.48 ID:oUt0FaYw
ガオガイガーしかねえのかよ。
501ご冗談でしょう?名無しさん:2012/06/03(日) 13:45:06.01 ID:???
行列式は5次以上の行列だと、
事実上手計算では実用的な計算方法が無い。
502イムジンリバー ◆NOsnoCwlwY :2012/06/03(日) 13:51:07.11 ID:???
>>489
>台風の進路がコリオリ力で曲がると言う話を聞いた事ないか?
台風ってそれ以外の力でも曲がるから、例えとしてはわかりずらいと思う
台風(低気圧)が、北半球と南半球で回転方向が違うという例えのほーが、
目に見える効果としては説得力があると思う
503イムジンリバー ◆NOsnoCwlwY :2012/06/03(日) 13:58:03.10 ID:???
高校生程度だと、行列の計算方法はわかっても、
応用問題(実際の学問や事象の解法でつかう)までいくかって問題もあるからね
504イムジンリバー ◆NOsnoCwlwY :2012/06/03(日) 14:07:53.14 ID:???
質問デス
ホーキング放射って要約すると次のようなカンジだよね
@事象境界付近で対生成した粒子の片方がブラックホールに落ち、片方が逃げ延びて我々に観測される場合がある
A我々に観測された粒子のぶん、保存則によりブラックホールはエネルギーを失い、最後には蒸発する

でもチョットおかしくない?
@ブラックホールも粒子を飲み込んでるんだから、ブラックホールのエネルギーも増えてない?
Aそもそも対生成ってプランク時間内に対消滅しないと保存則自体が崩れない?
Bだとすると、保存則自体を自分勝手に崩しておいて、「保存則によりこうなる」って虫が良すぎない?
505ご冗談でしょう?名無しさん:2012/06/03(日) 14:14:48.11 ID:???
ハイゼンベルグも知らなかった
506ご冗談でしょう?名無しさん:2012/06/03(日) 15:43:22.54 ID:???
>>487
地球と一緒に回転してるから、コリオリの力の様な慣性力が現れるのだが?
507ご冗談でしょう?名無しさん:2012/06/03(日) 16:24:08.57 ID:???
>>501
LU分解ってそんなに手計算で難しい?
面倒くさいが、5次程度なら非実用的とも言えないだろ
508ご冗談でしょう?名無しさん:2012/06/03(日) 19:38:37.15 ID:???
(熱力学)
定圧かつP=Pexまたは、定圧かつdV=0をみたせば
dH=dqとなるのですね?
509ご冗談でしょう?名無しさん:2012/06/03(日) 20:53:07.39 ID:???
もし宇宙のエントロピーが保存されるような物理法則だったら
現象としては今とどのような違いが出てきますか?
510ご冗談でしょう?名無しさん:2012/06/03(日) 22:28:12.28 ID:yuKaVYV0
すみません、いままで相対性理論の議論で出てくる光時計の話について
特に違和感もなく受け入れていたのですが、ふと疑問が湧いてしまいましたので、
皆様の智識をお借りしたく、質問させていただきます。

よく例に出される話として、電車に乗っているAさんと同じく電車に
乗っている光時計A'が、地球の地面に対して亜光速で動いている状況を
考えることがあると思います。
光時計A'は高さが15万kmで、往復に1sかかります。

この時、Aさんにとっては、Aさんと同じように動いている光時計A'では、
光は、電車の床面の鏡から光が上に出て、0.5秒後、上面の鏡に到達し
その0.5秒後、また床面の鏡に光が戻ってくると言います。
でも、本当にそうなるんでしょうか?
電車のスピードが光速に近ければ、電車の床に設置した鏡から光が
上に発射されて、上面の鏡に到達する前に、光時計は進んでしまい、
光が15万km上空にある地点に到達するときは、そこに光時計A'の
鏡はいない状況が考えられると思います。

この点は考えなくてよいのでしょうか?Aさんにとっても、光が進行方向
にそって斜めに発射されないと、ちゃんと反射されて戻ってこない気がします。
でも、斜め上に光を発射した訳ではないはずです。
どういうことでしょうか??
511ご冗談でしょう?名無しさん:2012/06/03(日) 22:42:12.12 ID:???
>>504
低エネルギーのホーキング放射は電磁波だから、その説明は対生成しない光子に通用しない。
それにブラックホールでなくても加速度系があれば起こる現象(アンルー効果)だから片方がブラックホールに落ちるのも通用しない。
インチキ説明だと思った方が良いでしょう。
512ご冗談でしょう?名無しさん:2012/06/03(日) 22:48:21.01 ID:???
>>510
Aさんにとって光が真上に発射された状況を地面から見ると斜めに発射している。
電車の中で物を上に投げて受けた事ない?
513510:2012/06/03(日) 22:56:49.43 ID:yuKaVYV0
>>512
早速のレスありがとうございます。
ボールでは、Aさんが真上に投げても、地上にいるBさんから見ると進行方向に斜めに
投げ上げているように見えるのはわかります。
私の疑問を言い換えると、光においても、x方向成分(電車進行方向成分)に慣性を
持つのでしょうか?
となります。
どうも、この点に違和感を感じてしまっています。

一般的に言えば、波源が動けば、そこから発射された波も波源の持つ速度成分を保持
するのでしょうか?
という疑問です。
光速度一定なのに、光の進行方向以外の成分には波源速度に応じた自由度が
許されているというところに疑問を持ってしまいました。
514ご冗談でしょう?名無しさん:2012/06/03(日) 23:29:42.42 ID:???
>>513
電車に乗っているAさんからみればx方向成分は観測されません。
x方向成分が観測されるのはあくまでも地上から観測した場合です。
観測者の立場を混同すると迷宮に入り込みます。
515510:2012/06/03(日) 23:32:35.62 ID:yuKaVYV0
>>514
つまり、Aさんからx方向成分が観測されないということは、ボールと同様、
光子に慣性があるということでしょうか?
516ご冗談でしょう?名無しさん:2012/06/03(日) 23:32:55.86 ID:???
光速度一定って言い方はややミスリーディングだよね
速さが一定なのであって、ベクトルとしての速度は観測者によって異なりうる
517514:2012/06/03(日) 23:37:21.07 ID:???
>>515
すみません、ちょっと説明の方向が違ったみたいですね。
>慣性がある
という表現がちょっとわかりづらいですが、
Aさんからみれば光は真上に発射されて真下に反射してきますので
慣性など外部の力を感じることはありません。

地上からこれを観測した場合、光は斜め上に進むように観測されるわけですが
これは光が電車のx方向成分の影響を受けて斜めに進むのではなく、
元から斜め上むけて発射されているように観測されるのです。
518ご冗談でしょう?名無しさん:2012/06/03(日) 23:38:51.54 ID:???
長さ1cmの筒状の光線発生器が上を向いているとして、
これが右方に一定スピードで動いているときに、筒の壁にぶつからずに光がちゃんと出るためには、光は斜めに発射されると考えなければならない
たとえば○が各時刻の光の位置だとして、時間経過に沿って、
| |
| |
|○|

 | |
 |○|
 | |

  |○|
  | |
  | |
これで○の軌跡を追ってみると、斜めになってる
もしこれが斜めでなくてまっすぐ上に向かってたら、筒の壁にぶつかっちゃう
これは筒と一緒に動いている人から見ると筒の壁にぶつからずに光がちゃんと発射されるのと矛盾する
519510:2012/06/03(日) 23:45:04.12 ID:yuKaVYV0
>>517
>Aさんからみれば光は真上に発射されて真下に反射してきますので
私も、当初は、何の疑いもなく上記のようになるだろうと思っていたのですが、
こうなるためには、ボールを投げ上げたときと同様、光子にも慣性が
なければならないと思いますが、違うのでしょうか?

ボールを電車の中で投げ上げると、また同じ床の位置に落ちてくるのは、
ボールに慣性があり、電車と同じ方向に進みながら上下に動いているからですよね

>>518
その筒に当たらないように光子が右方向に進んでくれるのかな?と
ふと疑問に思ってしまった訳です。。
愚問でしょうか?。。

光子の慣性質量があるのであれば、納得できます。
520ご冗談でしょう?名無しさん:2012/06/03(日) 23:54:46.64 ID:???
>>519
>>518の絵をよく見てもなおわからんと言うならそれは愚問。
521ご冗談でしょう?名無しさん:2012/06/03(日) 23:56:01.21 ID:???
慣性があるという言い方がかなり微妙だけど、そう思っていいよ
慣性があるって結局運動量が保存することだし、光子にも(慣性質量はなくても)運動量はある
522514:2012/06/03(日) 23:58:12.07 ID:???
>>519
たとえば電車に窓がなければ動いてるか止まっているかAさんには知るすべがありません。
慣性運動している場合、それが動いているのか止まっているのか区別できない
というのが相対性原理です。
ボールの例のようにニュートン力学では慣性力という見かけの力を用いて斜めに進むことを
説明しますが、相対性理論ではそもそも慣性力など存在しないと考えます。
上記相対性原理と光速度一定ということだけを前提に慣性運動を説明しています。
ですから相対性理論で運動を考えるときは慣性力と言う概念は忘れてください。
523ご冗談でしょう?名無しさん:2012/06/03(日) 23:58:20.02 ID:???
>>519
光時計を地面に固定して、それを電車から見たらどうなるかを考察してみたら如何
524510:2012/06/04(月) 00:01:08.62 ID:urJTiYtu
>>521
なるほど、運動量保存で考えればよいのですね
それなら光子にも当てはまる
運動量が保存するから、電車から発射した光は電車進行方向にも
運動量をもつので、地上から見ると斜め方向に進んで見える

納得できました

>>520
>>518の絵はわかっています。ただ、慣性のあるボールなら
そうなるだろうけど、光(波)ってどうなんだっけ?と思ってしまい
はまりました。
>>521さんの運動量保存で、ちゃんと筒に当たらないように、
電車の動きに応じて上に進んでいくんだなと合点がいきました。

貴重なお時間を割いていただき、どうもありがとうございましたm(_ _)m
525510:2012/06/04(月) 00:06:35.80 ID:urJTiYtu
>>522
相対性原理を忘れていました。
ニュートン力学での慣性力は考えないようにします。
ありがとうございます。

>>523
光時計を地面に固定した場合、それを電車から見ると、
結局、地面から電車上の光時計を見たときと同じように見えますね。
(合ってるかなw)
526ご冗談でしょう?名無しさん:2012/06/04(月) 02:07:58.90 ID:???
>>522
> 慣性運動している場合、それが動いているのか止まっているのか区別できない
> というのが相対性原理です。
それはニュートン/ガリレイの力学でも同じで、慣性系に対して加速度運動する
系でなければ慣性力は登場しないのでは?
527ご冗談でしょう?名無しさん:2012/06/04(月) 03:16:22.07 ID:???
電気回路で質問があります

三相交流のY型起電力とY型負荷を連結します
負荷側の中性点を抵抗つきで接地した時、そこに流れる電流が知りたいです

これは中性点の電位Vnを求めて接地との電位差で電流を求めればいいだけでしょうか?
528ご冗談でしょう?名無しさん:2012/06/04(月) 04:10:42.27 ID:???
>>510
慣性というものを誤解している。
光だろうと音だろうと運動量を持たない現象であろうと電車で真上なら地面からは斜め。
慣性じゃなく単なる座標変換。
(現象の例は、遠くからサーチライトを当てたスポットの動きなど、それが超光速で動いても問題ない)
529ご冗談でしょう?名無しさん:2012/06/04(月) 08:01:07.34 ID:???
>>527
はい、そうです
530イムジンリバー ◆NOsnoCwlwY :2012/06/04(月) 08:33:45.72 ID:???
>>511
回答ありがと
ワタシも少し調べてみたけど、ホーキングの威光のせいか殆どが既に認められた理論みたいに書かれています
Yahoo知恵袋にも沢山質問が出てたけど、否定的なアンサーはベストアンサーにはなれない様ですね
531イムジンリバー ◆NOsnoCwlwY :2012/06/04(月) 08:58:22.71 ID:???
>>525
例えばすいてる通勤電車が時速100km/hで走ってるとします
左右の席に座った2人がキャッチボールを始めました
左右のヒトにとっては単に相手にボールを投げればよいだけですが、
電車外のヒトにとっては、ボールの軌跡はジグザグに見える

さあここで問題
相対論のソの字も知らないヒトに説明を求めたらこう答えるでしょう
「ボールは既に時速100kmで前方に動いてるから、ボールはその慣性と投球速度のベクトルで斜めに進み、ボールの軌跡はジグザグとなる」
このヒトの頭の中では、線路は止まっていて電車が動いてるという固定観念があります

相対論でもこの説明を引きずると、理解が難しくなります
全ての慣性系に優劣関係はないのですから、
時速100km/hで走る電車のヒトにとって、
「我々が静止していて、線路(地球)が電車の後方に時速100km/hで動いてる」
と考えてよいのです
この慣性系で、相手のミットめがけて球を投げれば、コントロールさえ正確ならキチンと球はミットに収まります
532ご冗談でしょう?名無しさん:2012/06/04(月) 09:04:16.50 ID:???
>>531
質問です。なぜミットが必要なんでしょうか。素手ではだめなんですか?
それとグラブではなくミットなのはなぜ?
533ご冗談でしょう?名無しさん:2012/06/04(月) 09:05:29.04 ID:???
素手だと痛いから
534ご冗談でしょう?名無しさん:2012/06/04(月) 09:10:19.74 ID:???
電車の幅くらいの間隔で手が痛いほどの球を投げたらキャッチできないでしょ。
ふつうはトスをすると思います。
535ご冗談でしょう?名無しさん:2012/06/04(月) 09:11:07.53 ID:???
だからミットをつけるんだよ馬鹿か
536ご冗談でしょう?名無しさん:2012/06/04(月) 09:16:06.34 ID:???
いや、そんなに近過ぎたら受け取る側が反応できない。
537ご冗談でしょう?名無しさん:2012/06/04(月) 09:16:51.30 ID:???
デブなら反応できない
538ご冗談でしょう?名無しさん:2012/06/04(月) 09:19:01.96 ID:???
素手ならミートだろ
539ご冗談でしょう?名無しさん:2012/06/04(月) 09:25:54.86 ID:???
3メートル弱しかない電車の幅の中で二人の人間が向かい合ってミットが必要なほど
速い球でキャッチボールをするというのは非現実的。
540ご冗談でしょう?名無しさん:2012/06/04(月) 09:26:53.21 ID:???
>>531
地球が後方に等速直線運動するなら、そのうちに電車が地球から離れる?
でも線路は測地線に沿ってるので離れない?
541ご冗談でしょう?名無しさん:2012/06/04(月) 09:28:06.67 ID:???
時速1500kmくらいで打ち出してもミットでとれるから問題ない
野球ボールが8tくらいになるけど
542ご冗談でしょう?名無しさん:2012/06/04(月) 09:33:55.00 ID:???
>>540
近似って知ってるか?
543イムジンリバー ◆NOsnoCwlwY :2012/06/04(月) 09:35:22.59 ID:???
>>540
そこまで言うなら電車は慣性系じゃなく加速系になる
せっかくわかりやすく説明してるんだからさ
544ご冗談でしょう?名無しさん:2012/06/04(月) 09:54:59.59 ID:8YDI/PF3
ガラスの水槽などで全反射が見れますが
あの光も偏光してるのですか?
545 忍法帖【Lv=9,xxxP】 :2012/06/04(月) 10:47:04.83 ID:???
勿論変更して居るよ。
546ご冗談でしょう?名無しさん:2012/06/04(月) 12:35:45.12 ID:???
日本のマスコミの報道は偏向しているのですか?
547ご冗談でしょう?名無しさん:2012/06/04(月) 12:38:45.33 ID:???
偏向してない報道はボランティアだけだよ
548ご冗談でしょう?名無しさん:2012/06/04(月) 12:51:48.89 ID:???
蒸し返しの便乗質問で恐縮ですが、
>>510のシチュエーションで車外に受光装置Bがあって、A'が光を発した
瞬間にちょうどA'に対向する車外位置にBがあったとしたとき、Bは光を受け取れるのでしょうか。
A'の光源は点光源とかではなく狭い指向性を持ったビームだとします。
549ご冗談でしょう?名無しさん:2012/06/04(月) 13:36:15.04 ID:???
Bの運動状態は?
Bも同じ速度で動いてるなら受け取れる。違う速度ならダメ。
同様な状況は人工衛星のコーナーリフレクターにレーザー光を当てて反射を観測する時にもある。
コーナーリフレクターは来た光を戻る方向にしか反射しないので、そのままでは速度の違う地上で反射を受け取れない。
そこで衛星速度に応じて曲げた鏡を設計する。
550ご冗談でしょう?名無しさん:2012/06/04(月) 13:36:33.37 ID:???
Bが地面に対して静止しているなら受け取れない
551ご冗談でしょう?名無しさん:2012/06/04(月) 17:55:22.06 ID:???
>>549
>>550
ありがとうございます。
Bは静止しているという前提を書き忘れてました。
やはり受け取れないのですね。
音波であれば媒体が一緒に動いているからと理解できるのですが、光だとエーテル?が一緒に動いているのかと悩んでしまいます。
これじゃ元質問の方から一段低レベルになっただけですね。
失礼しました。勉強し直します。
552ご冗談でしょう?名無しさん:2012/06/04(月) 19:30:13.06 ID:l7m60hps
ヘルダイバーシステム 
3D、4Dの立体テレビシステムに((商)丸紅開発など)に、光水の量を多めに付けて、通話、視聴機械システムいれれば、魔法世界作れる 登記
553 忍法帖【Lv=9,xxxP】 :2012/06/04(月) 21:24:57.81 ID:???
物理学者はリア充だよ。
554ご冗談でしょう?名無しさん:2012/06/04(月) 23:05:10.42 ID:a62GXXE6
ソフトフォノンの凍結に関する質問です。
ある物質の構造相転移が(I4/mmm→Cmca)、
I4/mmmのブリルアンゾーンX点(kの群:D2h)のフォノンの凍結により引き起こされます。
このときX点での小表現
   E C2 C2' C2'' I σ σ' σ''
B2g 1 -1 -1 1 1 -1 -1 1
で表されるモードが凍結します。

論文を読んでいると、最初に転移前後の構造が決定されていて、
I4/mmmからCmcaに落ちるなら、B2gが凍結するということは簡単にわかると書いてありました。
なぜB2gが凍結するとこのような構造の変化(対称性の欠落)が起こるのでしょうか?
ご教授よろしくお願いします。
555ご冗談でしょう?名無しさん:2012/06/04(月) 23:10:57.33 ID:???
そんな難しいこと聞かれてもなあ
556ご冗談でしょう?名無しさん:2012/06/04(月) 23:26:57.12 ID:prUzLs0l
量子力学を勉強しはじめた者です。

一様電場(E)中の水素原子における摂動の問題を考えています。
1次の摂動エネルギーは見よう見まねで 0 (重解), ±eEa(aはボーア半径) が出てきたのですが、
まだ縮退が溶けていないので2次の摂動エネルギーを求めようと思っています。
しかしながら基底状態に対する2次の摂動エネルギーの求め方(どのような永年方程式になるのか)がわからず悩んでいます。

ご教授の程よろしくお願いします。

557ご冗談でしょう?名無しさん:2012/06/04(月) 23:41:00.42 ID:???
>>555
すみません。どうしても分からないもので…
558ご冗談でしょう?名無しさん:2012/06/04(月) 23:51:15.87 ID:???
先生に聞いたほうがいい気がするが
559ご冗談でしょう?名無しさん:2012/06/04(月) 23:54:52.33 ID:???
>>558
訊いたのですが「自明だ」という答えが返ってきまして…
560ご冗談でしょう?名無しさん:2012/06/05(火) 00:06:24.89 ID:9sYWhmpA
下のURLの問13の
AB間の電位差を求める問題の答えをどなたか教えていただけませんか?
出来れば理由もお願いします
http://www.kouku-dai.ac.jp/nyuusi/sogo23.pdf
561ご冗談でしょう?名無しさん:2012/06/05(火) 00:11:01.99 ID:???
電圧の二乗÷周波数って何を意味しますか?
562ご冗談でしょう?名無しさん:2012/06/05(火) 00:11:28.88 ID:???
>>556
1次摂動をどうやって導出したか考えれば自ずと一般化出来る
563ご冗談でしょう?名無しさん:2012/06/05(火) 00:20:14.69 ID:dpB50XzW
>>562
今導出してみてますけど、これって1次の時よりも面倒くさい感じですか…
564ご冗談でしょう?名無しさん:2012/06/05(火) 00:29:06.34 ID:???
>>560の問15.を読んでみたがすげー意地悪な問題だな。物理的な理解を測ってるとは言い難い。
565ご冗談でしょう?名無しさん:2012/06/05(火) 00:55:03.23 ID:???
空が青いのはレイリー散乱という現象によるものだと聞きました。
レイリー散乱は気体や液体中の微粒子によるものだとも聞きました。
結局、空が青いのは大気中の酸素分子、窒素分子、水分子によるものなのですか。
566ご冗談でしょう?名無しさん:2012/06/05(火) 01:05:22.09 ID:??? BE:5243654579-2BP(33)
>>560
<回答>
答え:(4) 2.0V
理由:
閉回路の全起電力Vは左回りを正とすると
V = 1.5 + 1.5 - 1.5 = 1.5 (V)
電池の内部抵抗をRとすると、この閉回路に流れる電流Iは
I = V÷3R = 1.5/3R = 0.5/R (A)
今、B点の電位Vbを0Vとすると、A点の電位Vaは左の電池の内部抵抗における電圧降下と
電池の起電力を足したものになるから
Va = IR + 1.5 = (0.5/R) × R + 1.5 = 0.5 + 1.5 = 2.0(V)
よって、電位差はVa - Vb = 2.0 - 0 = 2.0(V) となる
567ご冗談でしょう?名無しさん:2012/06/05(火) 02:48:49.70 ID:???
>>565
空が青いのは分子による散乱だが、空気が完全に一様なら散乱は打ち消し合って散乱しない。
さらに大気密度の変動があって打ち消し合いが完全でないため散乱する。
分子と密度変動の両方が原因。
568ご冗談でしょう?名無しさん:2012/06/05(火) 02:55:03.98 ID:???
>>554
使われてる専門用語は全然知らんが、フォノンとは格子振動つまり構成粒子の位置変位が伝播するもの。
それが凍結すれば元の粒子位置からの変位が固定される訳だから新しい配置つまり新しい構造になる。
569ご冗談でしょう?名無しさん:2012/06/05(火) 03:15:22.44 ID:???
>>568
ご回答ありがとうございます。
対称性が変化する理屈は分かります。
凍結モードが分かっていれば、
空間群まで決定できる理由が分からないのです…。

しかしどうも僕は難しく考えすぎな気がします。
一旦貴方のおっしゃった視点に戻って考えてみると
何か分かるかもしれない気がしてきました。
改めて考えてみます。
570ご冗談でしょう?名無しさん:2012/06/05(火) 04:47:26.81 ID:gndDtR2N
中心力場でのシュレディンガー方程式を解きたいのですが
Ψ(r,θ,φ)=R(r)Y(θ,φ)と表し
極座標のシュレディンガー方程式をR(r)の関数で表しましたが
ここからどうすればr,θ,φが導けるのか分かりません。
どなたか教えて頂けないでしょうか。
物理学科ではなく、量子力学の基礎知識はほぼありません。
571ご冗談でしょう?名無しさん:2012/06/05(火) 04:57:19.89 ID:???
そんなもん、教科書みろよ。
572ご冗談でしょう?名無しさん:2012/06/05(火) 05:13:28.73 ID:???
>>571
量子力学の教科書を持っておらずネットでは調べてみましたが
r=という形で書かれているものが見つけられず
こちらで質問させて頂きました。
教科書に載っている初歩的な問題のようですみません・・・
573ご冗談でしょう?名無しさん:2012/06/05(火) 07:09:12.31 ID:???
>>564
確かに物理的理解よりは単なる注意力を測っているだけだな
574ご冗談でしょう?名無しさん:2012/06/05(火) 07:11:59.39 ID:9sYWhmpA
>>566
なるほど!
分かりやすい解説ありがとうございました。
575ご冗談でしょう?名無しさん:2012/06/05(火) 08:45:34.43 ID:mCn8QJ2L
場が変化して、粒子が動くから、固定される。わかりずらいかな?
576ご冗談でしょう?名無しさん:2012/06/05(火) 09:19:05.36 ID:???
>>572
>r=という形で書かれているものが見つけられず
R(r)の関数形を求めるのが方程式を解くという作業であって
r=という形で書かれるものじゃないから見つからなくて当然
577ご冗談でしょう?名無しさん:2012/06/05(火) 10:43:32.27 ID:???
578ご冗談でしょう?名無しさん:2012/06/05(火) 11:43:31.37 ID:STfRwrUT
>>567
ゴルフプレイヤー・・・竹林・・・敵

ゴルフプレイヤーが竹林に向かってゴルフボールを大量に打ち込むと、
ゴロフボールは跳弾となって、あちこちの方角がから敵に向かって飛んでいく

これが青い光の挙動であり、空が青い理由
579ご冗談でしょう?名無しさん:2012/06/05(火) 12:01:20.57 ID:???
最近読んだ本に 
角運動量がうんたらかんたらで、月が地球の自転速度に影響を与えてるって書いてあったのですが
地球の地軸の傾きに関しては影響を与えてないのでしょうか

580ご冗談でしょう?名無しさん:2012/06/05(火) 12:07:08.87 ID:???
月の存在が地軸の傾きを安定化させているという話もある。
そういう意味では影響を与えているね
581ご冗談でしょう?名無しさん:2012/06/05(火) 12:25:07.12 ID:???
すいません学校の物理の問題で瞬時値の求め方がわからないので質問させていただきます。
pは粒子速度だとして

p = q(e^-jkx)(e^jwt)
という式を瞬時値で表すと
p(x,t) = √2|q|cos(wt-kx)
となるそうなのですがこの計算過程を教えていただけないでしょうか?
当方物理関係の知識が全くなく実効値などの説明も合わせてしてくれるとありがたいです。
582ご冗談でしょう?名無しさん:2012/06/05(火) 14:57:44.67 ID:???
>>578
全然「青い」理由の説明になっていない件
583ご冗談でしょう?名無しさん:2012/06/05(火) 19:35:09.52 ID:???
ttp://www.ganshodo.co.jp/mag/moon/moon-a.html

ここに月の動きを離れたところからみるとらせんっぽい動きじゃなくて波線の動きに見えるって書いてありますが

ttp://www.youtube.com/watch?v=8fj8tK1kEB8

この動画何回見ても月の動きが上のページにあるような波線の動きに見えないんです
どうしてなんでしょうか・・・
584ご冗談でしょう?名無しさん:2012/06/05(火) 19:41:01.85 ID:???
その動画のタイトルどおり、デフォルメしてあるからでしょう
585ご冗談でしょう?名無しさん:2012/06/05(火) 19:45:37.78 ID:???
な、ナンダッテー

どこかに正確な動きみれる動画ないのかなぁ
586ご冗談でしょう?名無しさん:2012/06/05(火) 19:53:43.01 ID:Iou/sPNr
振り子の回転の条件が分かりません。
長さlの糸の先に質量mの球が付いている条件で
球が最下点を通過するときに最低どれだけの速度であれば
振り子がくるっと一回転するのでしょうか?
587ご冗談でしょう?名無しさん:2012/06/05(火) 19:55:30.53 ID:???
>>585
正確な縮尺でやると地球と月の距離が1ミリで地球と太陽の距離が40センチ
とかになるから、波線にすら見えないかも
588ご冗談でしょう?名無しさん:2012/06/05(火) 20:06:54.15 ID:???
>>586
最下点の運動エネルギーが最上点の位置エネルギーに等しい。
v^2/2=2gl
589ご冗談でしょう?名無しさん:2012/06/05(火) 20:09:04.61 ID:???
最上点では速度ゼロでいいのですか?
590ご冗談でしょう?名無しさん:2012/06/05(火) 20:12:04.47 ID:???
>>581
粒子速度が複素数だったり実効値なんて言葉が出てくる物理の問題に心当たりはないな
一体何の物理?
591ご冗談でしょう?名無しさん:2012/06/05(火) 20:16:04.79 ID:???
>>585
大まかに地球も月も円軌道、月の公転周期を(1/13)年、軌道半径を(1/400)天文単位
とすると↓こんな軌道。
http://www.wolframalpha.com/input/?i=ParametricPlot[{{Cos[t]%2CSin[t]}%2C{Cos[t]%2B%281%2F400%29Cos[13t]%2CSin[t]%2B%281%2F400%29Sin[13t]}}%2C{t%2C0%2C2Pi}]

月の軌道半径を10倍にデフォルメすると↓こう。
http://www.wolframalpha.com/input/?i=ParametricPlot[{{Cos[t]%2CSin[t]}%2C{Cos[t]%2B%281%2F40%29Cos[13t]%2CSin[t]%2B%281%2F40%29Sin[13t]}}%2C{t%2C0%2C2Pi}]
592ご冗談でしょう?名無しさん:2012/06/05(火) 20:17:01.37 ID:???
>>589
最低速度を聞いたんだろ?
速度ゼロだと最上点からどっちへ落ちるか運任せ。
確率1/2で一回転する。
593ご冗談でしょう?名無しさん:2012/06/05(火) 20:19:25.38 ID:???
けど糸でつないである設定でしょ?糸がゆるみませんか?
594ご冗談でしょう?名無しさん:2012/06/05(火) 20:19:49.22 ID:???
>>590
固体物理とか半導体の話だろ
595ご冗談でしょう?名無しさん:2012/06/05(火) 20:24:12.56 ID:Iou/sPNr
糸がゆるまない条件でお願いします
596 忍法帖【Lv=10,xxxPT】 :2012/06/05(火) 20:24:44.01 ID:???
物理学サイドが最強ですよ。
597ご冗談でしょう?名無しさん:2012/06/05(火) 20:32:20.14 ID:???
>>594
半導体で使う?
音じゃないの?
598ご冗談でしょう?名無しさん:2012/06/05(火) 20:59:18.07 ID:???
>>591
おーなるほど わざわざありがとうございます
599581:2012/06/05(火) 21:03:48.32 ID:???
>>590>>594>>597
返信が遅れてすいません。
一応音響学の波動方程式という分野の一部です。
そこで先ほどの>>581の質問をしたのですが
実際は実行値は関係ないかもしれないです。
ググッた所,瞬時値を求めるには実行値もいるっぽかったので一応聞いてみました。
自分でも色々計算しているのですが物理は高校レベル以下なためなかなか解けないです。
何か必要な情報があればできうる限り答えます。
長文失礼しました。
600ご冗談でしょう?名無しさん:2012/06/05(火) 21:13:40.56 ID:Iou/sPNr
>>588
どうも実験によると
v^2/2=2.5gl
だとゆるまず一回転するらしいです
2.5の意味がよく分からないです
601ご冗談でしょう?名無しさん:2012/06/05(火) 21:17:43.79 ID:???
最上点で遠心力が重力を超えていることが糸がたるまない条件。
それを課すと自動的に出てくる>2.5
602ご冗談でしょう?名無しさん:2012/06/05(火) 21:27:10.02 ID:???
>>567
散乱が打ち消しあうというのはどういうことなんでしょうか。
一度散乱してしまったものは元に戻りようがないように思えます。
大気密度の変動ということですが、それにしては空は安定して一様に青いように思えます。
603ご冗談でしょう?名無しさん:2012/06/05(火) 21:42:54.45 ID:WArraa2h
you tubeなど動画サイトをHPやメールなどにつけてテレビ局をつくること 登記
604ご冗談でしょう?名無しさん:2012/06/05(火) 22:32:06.27 ID:Iou/sPNr
>>601
mv^2/2=mgh 運動エネルギーと位置エネルギー
mv^2/r=mg 遠心力と重力のつり合い

計算してもよく分かりませんでした
どのようにすればよいのでしょうか?
605ご冗談でしょう?名無しさん:2012/06/05(火) 22:49:08.05 ID:???
>>604
最高点と最低点では速さが違うから別々の記号を割り当てる。
606ご冗談でしょう?名無しさん:2012/06/05(火) 22:56:39.33 ID:???
>>604
1行めの式が変。vは最上点の速度なんだろ?
(下での運エネ)=(上での運エネ)十(位置エネ)
という式にしないと。
607ご冗談でしょう?名無しさん:2012/06/05(火) 23:29:19.83 ID:Iou/sPNr
なんか色々計算してたら2.5が出ました
ありがとう
608ご冗談でしょう?名無しさん:2012/06/05(火) 23:31:20.51 ID:???
偶然出たみたいないい草だなあ
609ご冗談でしょう?名無しさん:2012/06/06(水) 03:43:27.29 ID:???
電気回路のテブナンの定理について質問です

テブナンの定理は理解しているんですが、定理の一部の文章で疑問が残る部分があります

端子対abから見た回路網のインピーダンスZと端子対abにかかっている電圧Vがあり、ab間に抵抗Rを加えるとそこに流れる電流は

I=V/(Z+R)

になるとのことですが
この端子対abから見た回路網のインピーダンスZを求めるときに、その内部に存在する電源などを無視して
回路網の合成インピーダンスZを求めていますが、なんで無視できるのでしょうか?
610ご冗談でしょう?名無しさん:2012/06/06(水) 04:48:00.89 ID:???
>>602
波には干渉という現象がある。
一度方向を分けた光を位相が反対になるように一点に当てると、その点には光が来ない。
それが全空間で起これば散乱しなかったことになる。
大気密度の変動は星のまたたきとして見る事ができる。
611ご冗談でしょう?名無しさん:2012/06/06(水) 08:15:00.69 ID:???
>>610
すみません、もう少し教えてください。
干渉というのは結局レイリー散乱のことなのでしょうか。
星またたきと大気の変動の関係はわかるのですが空が青いこととの関連がわかりません。
612ご冗談でしょう?名無しさん:2012/06/06(水) 08:57:23.52 ID:???
つ波長
613ご冗談でしょう?名無しさん:2012/06/06(水) 09:18:54.07 ID:???
海が青いのと一緒だお
スキューバでちょっと深く潜ると赤い世界が広がるんだお

魚とか海底の石を揚げると全然色がちがうお ∈(゚◎゚)∋
614ご冗談でしょう?名無しさん:2012/06/06(水) 12:10:39.31 ID:???
>>611
ググって出てきたこんな説明では納得できないのか
http://salmon.nict.go.jp/snews/rayleigh/omake/index.html
615 忍法帖【Lv=11,xxxPT】 :2012/06/06(水) 12:15:17.01 ID:???
進化した物理学は中二病と区別は付かないですよ。
616ご冗談でしょう?名無しさん:2012/06/06(水) 12:39:33.44 ID:???
ディラックの量子力学の最初のほうにある

†The probability of a particular result for the state formed by
superposition is not always intermediate between those for the
original states in the general case when those for the original
states are not zero or unity, so there are restrictions on the
'intermediateness' of a state formed by superposition.

という注釈の意味が分かりません。
確率がゼロでも一でもない一般的な場合って何ですか?
それ以前に確率がゼロや一の場合って何ですか?
教えてくださいエロい人。
617ご冗談でしょう?名無しさん:2012/06/06(水) 13:02:42.45 ID:???
>>613
海が青いのは赤い光が吸収されるから。
空とは違う。
618ご冗談でしょう?名無しさん:2012/06/06(水) 13:09:45.66 ID:???
>>611
一度読むと前の説明は忘れるのか?
>>567を読み返せ。
619ご冗談でしょう?名無しさん:2012/06/06(水) 13:17:17.61 ID:???
>>616
状態の重ね合わせを説明しているだけ
620ご冗談でしょう?名無しさん:2012/06/06(水) 13:20:16.93 ID:???
>>616
言葉通り確率が0より大きく1より小さい場合だ。
確率0は起こりえない事で確率1は必ず起こる事。
621616:2012/06/06(水) 13:34:12.96 ID:???
>>619>>620
それが「intermediateness」の制約とどうつながるのかが分かりません。
確率が0<p<1でintermediateにならないのってどういう場合ですか?
622619:2012/06/06(水) 14:21:39.08 ID:???
>>621
式書くのめんどい
混合状態は二つの純状態の線型結合
波動関数の絶対値の二乗が存在確率
a.wikipedia.org/wiki/重ね合わせ
http://en.wikipedia.org/wiki/Quantum_superposition
623565:2012/06/06(水) 14:59:06.17 ID:???
>>614
もともと御紹介いただいたようなHPをみてレイリー散乱だとは理解していたのですが>567,>610のような説明をいただいてまたわからなくなりました。
624ご冗談でしょう?名無しさん:2012/06/06(水) 19:17:54.57 ID:U6ixysHI
全長400kmの線路において、受電端を短絡したとき、送電側からの
インピーダンスがj250Ω、受電端を開放したとき、送電側からの
アドミタンスはj1.5*10^(-3)Sだったとする。

このとき1kmあたりのリアクタンスと、サセプタンスを求めよという
問題ですが、答えがX=0.56Ω/km、B=3.4×10^(-6)/kmなんですが、
全然わかりません。どうやったらこの答えが出るのでしょうか?
625ご冗談でしょう?名無しさん:2012/06/06(水) 19:24:15.40 ID:4KNN6aSY
電気回路のスレで聞いた方がいいんじゃね?
626ご冗談でしょう?名無しさん:2012/06/06(水) 19:47:02.89 ID:???
半径Rの円形レールのループの頂点を質量mのボールが速度v=√g√Rで通過するとき(※g重力加速度)
ボールの回転エネルギーがmgR/5になるらしいです。
v=√g√Rは遠心力と重力の釣り合いの条件です。
ボールの半径は与えられておらずどんな半径のボールでもこうなるみたいです。
なぜボールの回転エネルギーがmgR/5になるのですか?
627ご冗談でしょう?名無しさん:2012/06/06(水) 19:54:55.70 ID:???
まず頂点でのエネルギー保存の式を書き下せ。
628ご冗談でしょう?名無しさん:2012/06/06(水) 20:02:33.00 ID:U6ixysHI
>>625さん
http://uni.2ch.net/test/read.cgi/denki/1321082778/
このスレでしょうか?
勢いがやや遅く、理論だけじゃなく組み立てに関してとか
実用的な質問が多いので、このスレで質問させてください。


問題については実はこれ分布定数回路の問題なんです。
受電端を短絡するという事で開放しているときとどう違うかも分かりません。
一番知りたいのはどういった計算をすれば例えばX=5.6kΩ/mと出てくるかわかりません。
400kmですから400で割るという事は分かるのですが.....
629ご冗談でしょう?名無しさん:2012/06/06(水) 20:15:26.67 ID:U6ixysHI
開放と短絡はわかります。
ただ、受電側を短絡したからといって、線路のキャパシタンスと
インダクタンスにどういった影響がでるかわかりません。

短絡した場合、最後につながってるのがキャパシタンスだったら
短絡したら、そのキャパシタンスの両端が同電位なため、電流は
流れませんよね。
でも400kmにおいて間に入ってるほとんどのキャパシタンスには影響
はないような気がします。
630 忍法帖【Lv=11,xxxPT】 :2012/06/06(水) 20:55:20.16 ID:???
全く影響は無いよ。
631ご冗談でしょう?名無しさん:2012/06/06(水) 21:00:14.12 ID:???
>>628
物理学科で使う教科書に分布定数回路のことなんかほとんど載ってないぜ。
それでもこっちのほうがいいのか?
632ご冗談でしょう?名無しさん:2012/06/06(水) 21:01:26.22 ID:jB1859qu
中性子って波長が決まれば速度も決まるんですか?
式変形しててふと疑問に思いましたので、どなたかお答えくださいmm
633ご冗談でしょう?名無しさん:2012/06/06(水) 21:23:53.53 ID:???
>>632
決まっちゃいかんの?
634ご冗談でしょう?名無しさん:2012/06/06(水) 21:34:29.71 ID:niBn+HNv
質問いいですか?長くてすみません…

x軸上に支点がある、長さL、質点mの振り子があります。
支点にはその位置ξに応じた復元力-kξがかかります。
振り子の鉛直方向から反時計回りの角度をθとし、
(ξ,θ)を一般化座標としてラグランジュ方程式を導きました。
結果、
g sinθ+ξ'' cosθ+Lθ'' = 0
kξ+mξ''+mLθ'' cosθ−mL(θ')^2・sinθ = 0
を得ました。

微小振動を考えθ<<1 とし、sinθ=θ、cosθ=1 、(θ')^2 = 0 とすると、上式は
gθ+ξ''+Lθ'' = 0 ...@
kξ+mξ''+mLθ'' = 0 ...A
となりました。
m@ - Aより、mgθ= kξ、すなわちξ'' = mgθ''/k 。これを@に代入して
gθ+( L+mg/k )θ'' = 0
となります。
したがって固有角振動数ωは
ω = √( kg / (mg+kL) )
となったのですが、
k→ 0 でω→ 0 になってしまい、直感と矛盾してしまいます。

どこがおかしいか教えてもらえまんでしょうか
よろしくお願いします。
635ご冗談でしょう?名無しさん:2012/06/06(水) 21:46:43.07 ID:???
>>633
ええんやで
636ご冗談でしょう?名無しさん:2012/06/06(水) 22:21:22.43 ID:U6ixysHI
>>631
一応大きい範囲では電気回路も物理の範囲だと思うのでどうかお願いします。
計算方法が分かれば理屈は分かります。
どういう計算で0.56Ω/kmって出ましたか?

開放してても、短絡してても、キャパシタンスは関係あるんですよね。
短絡したときインピーダンスがj250だから400で割ってみようと思って
しても0.625Ωなんですよね....
もっと複雑な計算をしないといけないのかな。
637ご冗談でしょう?名無しさん:2012/06/06(水) 22:21:45.14 ID:???
>>634
直感の方が怪しい
638ご冗談でしょう?名無しさん:2012/06/06(水) 22:29:47.95 ID:???
>>616
|ψ> = c0|φ0> + c1|φ1>
という重ね合わせ状態に対して物理量Aを測定して測定値aを得る確率は、
|φ0>に対してaを得る確率と|φ1>に対してaを得る確率の中間の値とは限らないよってことを言っている
639ご冗談でしょう?名無しさん:2012/06/06(水) 22:30:49.87 ID:niBn+HNv
>>637
でも k = 0 って支点が自由に動くって意味ですよね?
支点が自由に動いても振り子は振動するような気もしますが・・・
640ご冗談でしょう?名無しさん:2012/06/06(水) 22:33:16.99 ID:niBn+HNv
あ、もしかしてθ= 0 のままξが等速運動するとか?
641ご冗談でしょう?名無しさん:2012/06/06(水) 23:40:59.83 ID:4B/c2fMq
宇宙エレベーターの開発ってコリオリ力は考慮されてるの?
642ご冗談でしょう?名無しさん:2012/06/07(木) 00:01:23.90 ID:???
>>637
すいません詳しく教えてもらっていいですか?
643565:2012/06/07(木) 00:17:40.63 ID:???
>>641
なんだよ、それ。
644ご冗談でしょう?名無しさん:2012/06/07(木) 00:29:58.06 ID:???
>>638
それはc0、c1が一般には複素数だから(正の実数とは限らないから)ですか?
645ご冗談でしょう?名無しさん:2012/06/07(木) 00:42:26.46 ID:???
>>643
誤爆。スマソ
646ご冗談でしょう?名無しさん:2012/06/07(木) 00:56:34.17 ID:???
>>641
基本赤道上だろうから問題ないんじゃね
647ご冗談でしょう?名無しさん:2012/06/07(木) 01:08:27.92 ID:???
>>644
c0やc1が実数でもありうる
648ご冗談でしょう?名無しさん:2012/06/07(木) 04:43:01.81 ID:???
地表の台風と違って鉛直に伸びるから問題になるはず。多分赤道上空の静止衛星に伸ばすんだろうけど真上に打ち上げたときの自転方向の速度を考えたときにズレが生じるはずで
振り子の原理がどうたらで大丈夫とかいう説はあるけど厳密な説明は無い
649ご冗談でしょう?名無しさん:2012/06/07(木) 04:48:28.77 ID:???
>>623
>>567はレイリー散乱が有効になる理由だから、レイリー散乱だと理解するだけで充分なら>>567も干渉も忘れた方が良い。
650ご冗談でしょう?名無しさん:2012/06/07(木) 04:58:41.27 ID:???
>>641
宇宙エレベーターを上る時は西向きの力を受け、下る時は東向きの力を受けて、
速度が大きいほど力も大きくなるから、コリオリ力を考慮しない開発なんて危ない。
651 忍法帖【Lv=11,xxxPT】 :2012/06/07(木) 06:14:08.95 ID:???
エル・シャダイ力も気にした方が良いよ。
652616:2012/06/07(木) 09:26:59.01 ID:???
>>647
中間の値にならない例としては、
二重スリット実験で干渉の谷の部分に粒子が来る可能性が、
一方のスリットを通った場合の可能性、もう一方のスリットを通った場合の
可能性のいずれよりも小さい、というのが相当しますか?
653 忍法帖【Lv=12,xxxPT】 :2012/06/07(木) 09:38:00.14 ID:???
俺の右手に封印した闇の力ダークエネルギーが解放される。
654ご冗談でしょう?名無しさん:2012/06/07(木) 14:35:26.32 ID:???
>>653
1mは飛ぶな
655ご冗談でしょう?名無しさん:2012/06/07(木) 15:02:58.19 ID:???
フェルミ分布についての質問です。

フェルミ分布が階段関数になる説明として、
多くの教科書では、kT→0だとexpがゼロになるから、
というのが多いですが、
高密度領域では、Tが高温でも縮退するそうです。

高密度で階段関数になることを
f(E)=(1+exp(E-μ/kT))^-1
を式変形して導くにはどうしたら良いですか?
どこにも密度の情報が入っていないように思うのですが。
656ご冗談でしょう?名無しさん:2012/06/07(木) 15:11:40.20 ID:V1IyzS3L
宇宙戦艦ヤマトで波動砲から出てくる火は何ですか?あんなもので惑星がふっとぶんですか?
657ご冗談でしょう?名無しさん:2012/06/07(木) 15:49:27.04 ID:???
間隔の「d」を英語で表すと何ですか?
658 忍法帖【Lv=40,xxxPT】 :2012/06/07(木) 16:36:44.79 ID:???
distanceじゃねの?
659ご冗談でしょう?名無しさん:2012/06/07(木) 16:37:35.80 ID:???
>>657
聞き方が変
何の略ですかと言えばdistance
660 忍法帖【Lv=40,xxxPT】 :2012/06/07(木) 17:10:58.60 ID:???
デジモンの攻撃でも多元宇宙が消し飛ぶぞ!?♪。
661ご冗談でしょう?名無しさん:2012/06/07(木) 17:11:20.23 ID:???
>>569

どうだろねー。
数学的にはそうなんだろうけど、実際にはよほど頭のいい人でない限り、
International Tableとかで、
I4/mmmのサブグループを見ていくと
Cmcaがあって、両者を比較すると、
自由に動かせるようになった原子とその方向がわかる。

その制約の中でありそうな波を考えると、それはB2gだと気づいた、
そしてさらに考えると、B2gしか作れない!とかそんなんでないのかな−。

それを、I4/mmmからCmcaに落ちるなら、
B2gモードの凍結であることは数学的に自明である。とか書くとかっこいい上に、
レフリーもうなるじゃん?
662 忍法帖【Lv=40,xxxPT】 :2012/06/07(木) 17:23:13.48 ID:???
確かに美しいよ!?♪。
663ご冗談でしょう?名無しさん:2012/06/07(木) 17:57:36.96 ID:???
>>655
μが密度依存
664 忍法帖【Lv=12,xxxPT】 :2012/06/07(木) 18:25:03.25 ID:???
物理学は芸術。
665ご冗談でしょう?名無しさん:2012/06/07(木) 20:30:45.73 ID:eV8Pb0Ta
フェルミ分布が階段関数に密度の情報を入れるんだよ。後計算しろ、おまいら。佐藤勝彦先生が本出しているから、嫁め。
666ココ電球 _/::o-ν ◆tIS/.aX84. :2012/06/07(木) 20:32:52.58 ID:LHBuC8bM
よめめ言われても
667 忍法帖【Lv=12,xxxPT】 :2012/06/07(木) 20:33:03.09 ID:???
物理学は絵画。
668ご冗談でしょう?名無しさん:2012/06/07(木) 20:34:01.97 ID:eV8Pb0Ta
フェルミ分布が階段関数に密度の情報を入れるんだよ。後計算しろ、おまいら。佐藤勝彦先生が本出しているから、嫁め。中級だけどね。
669ご冗談でしょう?名無しさん:2012/06/07(木) 20:35:14.00 ID:???
>>663は人間の頭脳
>>665は昆虫の頭脳
670ご冗談でしょう?名無しさん:2012/06/07(木) 20:36:48.57 ID:???
物理学は生き様
671ご冗談でしょう?名無しさん:2012/06/07(木) 20:45:26.64 ID:eV8Pb0Ta
フェルミ分布が階段関数に密度の情報を入れるんだよ。後計算しろ、
フェルミ分布が階段関数に密度の情報を入れるんだよ。後計算しろ。
この手順でいいか
672ご冗談でしょう?名無しさん:2012/06/07(木) 20:49:29.46 ID:???
昆虫は条件反射で同じことを繰り返すだけ
673ご冗談でしょう?名無しさん:2012/06/07(木) 20:54:07.32 ID:eV8Pb0Ta
複多重計算ってしってるか?天文学で使うんだけど、佐藤勝彦線先生の本をおまいらよめ。
674イムジンリバー ◆NOsnoCwlwY :2012/06/07(木) 21:01:22.93 ID:???
>>666
ココ電球って何処でも出てくるね
675イムジンリバー ◆NOsnoCwlwY :2012/06/07(木) 21:10:18.45 ID:???
>>655
細かいよーだケド、E-μは同じディメンションならカッコでくくるんじゃない?
答えがゼンゼン違ってくるし
676イムジンリバー ◆NOsnoCwlwY :2012/06/07(木) 21:34:24.34 ID:???
>>655
>高密度領域では、Tが高温でも縮退するそうです。
○高密度領域では、Tが非常に高温であれば、準位が上がってるので縮退できるそうです。

>高密度で階段関数になることを
○高密度高温領域では、階段関数にならない

じゃない?
違ったらメンゴ
677ご冗談でしょう?名無しさん:2012/06/07(木) 21:47:08.25 ID:A95QCDgC
ブラックホールから、0ポイント、ホワイトホール
678ご冗談でしょう?名無しさん:2012/06/07(木) 21:55:30.10 ID:A95QCDgC
スペシャルスーパーフォローだけど、ブラックホールから、0ポイント、ホワイトホール イギリスの物理屋の理論だけど
679ご冗談でしょう?名無しさん:2012/06/07(木) 22:52:25.22 ID:???
>>663
なるほど!ありがとうございます。

もう一つ関連した質問なのですが、
高温になってくると階段関数がだんだん崩れていきますが、
非常に高温な状態でも、
フェルミエネルギー以上は粒子が少なく、以下だと粒子が多い、
といった粒子分布の目安の基準を
フェルミエネルギーにすることはできますでしょうか?

それとも、全くバラバラな分布になりますか?
680ご冗談でしょう?名無しさん:2012/06/07(木) 23:01:58.25 ID:???
低温でほとんどが最低エネルギーにあるからフェルミエネルギーが意味を持つんだ。
681ご冗談でしょう?名無しさん:2012/06/07(木) 23:08:53.78 ID:A95QCDgC
ベクトル、スクエア、ベクトル使えば良い。
682ご冗談でしょう?名無しさん:2012/06/07(木) 23:13:30.37 ID:A95QCDgC
ベクトル、スクエア、ベクトルで逆算計算しておくこと
683ご冗談でしょう?名無しさん:2012/06/07(木) 23:14:11.01 ID:A95QCDgC
ベクトル、スクエア、ベクトルで逆算計算しておくこと。
684ご冗談でしょう?名無しさん:2012/06/07(木) 23:14:24.75 ID:???
スカラー、ベクトル、テンソルで計算すること
685ご冗談でしょう?名無しさん:2012/06/07(木) 23:21:33.51 ID:A95QCDgC
それは、星の計算だろ。帰るのか。
686ご冗談でしょう?名無しさん:2012/06/07(木) 23:35:02.68 ID:???
メコスジブクロ
687ご冗談でしょう?名無しさん:2012/06/07(木) 23:35:59.42 ID:???
>>652
そうだね
688ご冗談でしょう?名無しさん:2012/06/07(木) 23:45:16.89 ID:A95QCDgC
689ご冗談でしょう?名無しさん:2012/06/08(金) 00:14:08.86 ID:???
もういい年なんだからチンカスを掃除しなさい
アニメなんか卒業してはやく大人になりなさい
690ご冗談でしょう?名無しさん:2012/06/08(金) 00:30:39.21 ID:???
>>679
まず国語を勉強してきてくれ
691 忍法帖【Lv=12,xxxPT】 :2012/06/08(金) 01:43:34.27 ID:???
アニメーションを見ないで居たらアイデアが思い付かないですよ。
692ご冗談でしょう?名無しさん:2012/06/08(金) 02:19:04.70 ID:oLsm7sCn
英語わかるか?
693ご冗談でしょう?名無しさん:2012/06/08(金) 02:20:02.82 ID:oLsm7sCn
英語わかるか?答えだよ。
694ご冗談でしょう?名無しさん:2012/06/08(金) 02:24:04.62 ID:oLsm7sCn
サンスクリット語最上級解答してみた。
695ご冗談でしょう?名無しさん:2012/06/08(金) 03:40:00.48 ID:???
とある魔術の禁書目録っていうライトノベルで登場したシーンで

主人公が敵兵の銃にコーヒーかけて

「熱膨張って知ってるか?」

とかいって銃が作動不良になって使えなくなったんですが

これは一体どういうことなんでしょうか

というかそもそもこの作品って物理に関係することが沢山あるんですが…

ちょっといろいろ作者の頭が心配でなりません

外伝の超電磁砲では電撃飛ばしたり

挙句はベクトル操作するとかいってんのに

禁書厨が熱量も操作するとかいいはじめて

もうわけがわからない状態になっていますw
696ご冗談でしょう?名無しさん:2012/06/08(金) 03:40:25.07 ID:???
禁書厨の頭の心配でなりません
697ご冗談でしょう?名無しさん:2012/06/08(金) 03:44:27.21 ID:oLsm7sCn
禁書目録の話がでてくる?
698ご冗談でしょう?名無しさん:2012/06/08(金) 03:51:06.51 ID:oLsm7sCn
なぜ禁書目録の話がでてくる?
699ご冗談でしょう?名無しさん:2012/06/08(金) 03:53:14.85 ID:???
>>652
経路の可能性なんてモノは状態じゃないし、そもそも物理的に意味のあるモノでもないから、観測値の確率とは関係ない。
重ね合わせた状態で例を考えるべし。
700ご冗談でしょう?名無しさん:2012/06/08(金) 03:57:57.95 ID:oLsm7sCn
スリットの可能性を、すべて計算して、比較すればいい。スリッドの問題や、スピンの問題は、解決済み。過去スレを探してみて。本にもなっているよ。
701ご冗談でしょう?名無しさん:2012/06/08(金) 08:46:42.68 ID:???
ライトノベルを現実と混同して読んでたらつまらなくならない?
702ご冗談でしょう?名無しさん:2012/06/08(金) 09:10:24.90 ID:???
そんなことより、イカ娘が触手で遠くのものを持ち上げていましたが、
あんなことが物理的に可能なのか、気になって夜はぐっすり寝ています。
703ご冗談でしょう?名無しさん:2012/06/08(金) 10:48:38.33 ID:???
イカちゃんは体重を制御できるので問題ありません。
704ご冗談でしょう?名無しさん:2012/06/08(金) 11:22:09.17 ID:???
「島」の意味がずっと分からなかったのですがコンデンサーは内部にしか合成電界が出来ないからそれに対して孤立して電気が出入り出来ないところをが島になるって分かったんですがそれであっていますか?
705ご冗談でしょう?名無しさん:2012/06/08(金) 11:26:49.12 ID:???
何の話だ?
706ご冗談でしょう?名無しさん:2012/06/08(金) 16:45:17.69 ID:fiSsxSeU
ラノベで経路積分が出てきたときは少し驚いた
707ご冗談でしょう?名無しさん:2012/06/08(金) 17:19:47.79 ID:???
経路積分みたいな中二ワードは出てきそう
708ご冗談でしょう?名無しさん:2012/06/08(金) 18:22:03.74 ID:???
アニメの「ナデシコ」で真空の相転移が出た時は驚いたよ。
知ったばかりの話だったからな。
709ご冗談でしょう?名無しさん:2012/06/08(金) 19:53:47.80 ID:vO1qGaXe
『第7回 新聞紙で作る高速道路“橋”コンテスト』の参加者募集
http://www.c-nexco.co.jp/news/2678.html
NEXCO中日本 名古屋支社は、昨年に引き続き「新聞紙で作る高速道路“橋”コンテスト」を開催します。

<<以下省略>>
710ご冗談でしょう?名無しさん:2012/06/08(金) 23:54:30.81 ID:???
対称性の自発的破れ:50年の謎解く発展理論に到達
http://mainichi.jp/select/news/20120609k0000m040072000c.html

カリフォルニア大大学院生の渡辺悠樹(はるき)さん(25)は、南部博士の理論を
数学的に発展させ、「対称性の自発的な破れ」をすべて説明できる理論を導き出した。

まあ、シロートに理解は無理だよね
711ご冗談でしょう?名無しさん:2012/06/09(土) 00:01:03.92 ID:???

Spontanius symmetry breakなんてニュートンにもしょっちゅう
出てくる理論じゃん。

ワインボトルの底の話だろ。

本来のきれいな理論に人工的で外圧的な作用を加えたようで
なんか好きじゃないんだよねSSBの考え方って。
712ご冗談でしょう?名無しさん:2012/06/09(土) 03:42:59.31 ID:???
ワインボトルはまるっきり人為的だが、ヒッグス粒子を調べればマトモな原因が分かるだろう。
それまでは「説明できる理論」と言ったって空論と変わらん。
713ご冗談でしょう?名無しさん:2012/06/09(土) 03:48:52.53 ID:OPngXHNc
カリフォルニア大大学院生の渡辺悠樹(はるき)って、裏口入学のパクリ野郎だろ。
対称性の自発的な破れは、解いたし、数学でも、解いたし、書いておいたよ。教授なら誰でも知っているよ。
714ご冗談でしょう?名無しさん:2012/06/09(土) 03:50:55.16 ID:OPngXHNc
カリフォルニア大大学院生の渡辺悠樹(はるき)って、在日朝鮮人だろ。
715ご冗談でしょう?名無しさん:2012/06/09(土) 04:05:06.89 ID:OPngXHNc
2chでも事件で、載ってた人でしょう。鬼女が叩いていた人でしょう。
716 忍法帖【Lv=14,xxxPT】 :2012/06/09(土) 05:49:04.51 ID:???
物理学TRPGを出して欲しいですよ。
717ご冗談でしょう?名無しさん:2012/06/09(土) 06:03:48.09 ID:???
TRPGくらい自分で作れよ
718ご冗談でしょう?名無しさん:2012/06/09(土) 10:50:56.73 ID:???
>>710
論文は?
719ご冗談でしょう?名無しさん:2012/06/09(土) 11:36:35.26 ID:???
>>710 でリンクされたニュースには研究チームとしか書いてないな。
720718:2012/06/09(土) 11:48:34.77 ID:???
721616:2012/06/09(土) 13:21:48.46 ID:???
>>687
逆に確率が0か1で絶対に中間の値になるというのは、
単に「確率は常に0以上1以下」ということを言ってるだけなんですね。
ありがとうございました、ようやく理解できました。

>>699
一方を塞いだ場合の、です。
722ご冗談でしょう?名無しさん:2012/06/09(土) 17:04:48.63 ID:???
もはや誰にも通じなくなってるな
723ご冗談でしょう?名無しさん:2012/06/09(土) 18:44:44.69 ID:???
そんなあなたにコーラック
724ご冗談でしょう?名無しさん:2012/06/09(土) 20:14:47.94 ID:3BDAqQmy
質問失礼します。数学の分野かもしれませんが、量子力学の話題の中で出た問題なのでこちらで質問させてもらいます。

行列A = | 0 1 |
      .| 1 0 |
(カッコが「| |」になってるけど行列式じゃないです)
として、 B = exp( (iπ/2) * A ) を求めるのですが、
自分はAを対角化しA^nを求め、expの定義式に代入することによって
B = | 0 i |
   .| i 0 |
を得ました。

しかし、exp(U†AU)が簡単に計算できることを用いると、
容易に求まるらしいのですがよくわかりません。
(UはAを対角化した際のユニタリ行列、†はエルミート共役)

なにかわかる方がいればよろしくお願いします。
725ご冗談でしょう?名無しさん:2012/06/09(土) 20:43:51.69 ID:???
>>724
あなたがやったやり方と本質的には同じだよ

exp(A) = U†exp(U†AU)U
726ご冗談でしょう?名無しさん:2012/06/09(土) 20:50:37.61 ID:???
>>724
A^2=Eだから簡単だろ
727ご冗談でしょう?名無しさん:2012/06/09(土) 21:22:56.60 ID:nlZraojX
スケートボードのヴァートランプ演技で
選手はトリックを繰り返すためにどこから運動エネルギーを得ていますか?
728ご冗談でしょう?名無しさん:2012/06/09(土) 21:29:44.98 ID:???
>>725
はぁー!exp(A)の固有値はexp(固有値)になるのか!
ありがとうございました!

>>726
それも思いついたのですが、できるだけ一般的に解きたかったもので・・・
レスありがとうございました!
729ご冗談でしょう?名無しさん:2012/06/09(土) 21:33:21.95 ID:???
>>728
一般的話ならコンパクト作用素(エルミートでも可)Aに対してf(A)が定義できる
ちゃんちゃん
730ご冗談でしょう?名無しさん:2012/06/09(土) 21:40:20.95 ID:???
>>725
重箱だけど
exp(A)=Uexp(U†AU)U†
731ご冗談でしょう?名無しさん:2012/06/09(土) 21:41:42.91 ID:???
>>729
ラーメンは味噌ってことですね!
ありがとうございました!
732ご冗談でしょう?名無しさん:2012/06/09(土) 21:43:37.76 ID:???
>>731
塩もすきだけど、家系がいいぞ
733ご冗談でしょう?名無しさん:2012/06/09(土) 22:17:49.70 ID:???
場の量子論を勉強しはじめたのですが、
分からない点があります。

電子と陽電子の散乱を考えます。
四元運動量はそれぞれ、pe+,pe-とします。

qを光子の四元運動量とすると、
pe+ + pe- = q
光子はq^2=0でないといけないので、0でないときは
off-shellの粒子、つまり仮想粒子と言われます

しかし、現実には、電子と陽電子が対消滅して
γ線が観測される現象がありますが、
その場合、上の式から考えると
陽電子と電子のエネルギーと運動量が
それぞれ厳密に一致しなければいけないのでしょうか?

そうすると、対消滅の条件がとても厳しいものになりませんか?
734ご冗談でしょう?名無しさん:2012/06/09(土) 23:23:56.57 ID:???
メコス汁の降る丘
735ご冗談でしょう?名無しさん:2012/06/09(土) 23:28:46.04 ID:???
>>733
古典粒子の散乱と場の理論の散乱をごっちゃにしてないの?
736ご冗談でしょう?名無しさん:2012/06/09(土) 23:44:07.88 ID:???
熱すぎるみそ汁を早くさますには
1.そのまま待つ
2.箸でかきまぜる

答えは2 でいいと思いますが、どういう理由からでしょうか
737ご冗談でしょう?名無しさん:2012/06/09(土) 23:47:09.35 ID:???
3.氷をいれる
4.うちわであおぐ
5.冷蔵庫にいれる
738ご冗談でしょう?名無しさん:2012/06/09(土) 23:58:32.04 ID:???
6. 床にぶちまける
739ご冗談でしょう?名無しさん:2012/06/10(日) 00:43:35.15 ID:???
容器に接触して冷めた味噌汁を強制的に剥ぎ取って換わりに熱い味噌汁を接触させるってことかな
740ご冗談でしょう?名無しさん:2012/06/10(日) 01:22:39.43 ID:???
>>739
ありがとうございます。
外界との境界面の温度を高くすると、伝導や放射で熱が早く出ていくということですね。
741ご冗談でしょう?名無しさん:2012/06/10(日) 01:24:47.16 ID:???
味噌汁を強い磁場の中においてスピンを揃えた後に断熱消磁により冷却する。
742ご冗談でしょう?名無しさん:2012/06/10(日) 01:28:09.97 ID:???
悪魔を一匹雇って運動の激しい分子だけをつまみ出して貰う
743ご冗談でしょう?名無しさん:2012/06/10(日) 01:33:51.46 ID:???
>>741
>>742

アイデア不足
もっと革新的な方法でお願いします
744ご冗談でしょう?名無しさん:2012/06/10(日) 02:01:07.01 ID:???
逆パターンとして
循環式湯沸かし風呂はかき混ぜないほうが効率良く早く沸きます
745ご冗談でしょう?名無しさん:2012/06/10(日) 02:25:42.91 ID:???
>>733
重心系なら厳密に一致するだろ
746 忍法帖【Lv=35,xxxPT】 :2012/06/10(日) 03:26:03.59 ID:???
ヒッグス粒子と重力子の関連性を知りたいですよ。
747ご冗談でしょう?名無しさん:2012/06/10(日) 03:38:07.01 ID:???
両方見つけてから言えよ
748ご冗談でしょう?名無しさん:2012/06/10(日) 08:17:13.19 ID:V8TqlTeF
物理関連トリビアは喜んで紹介するけど応用問題はオタク脳だから無理みたいな?
749 忍法帖【Lv=15,xxxPT】 :2012/06/10(日) 10:25:16.26 ID:???
SF小説の様な物理学を空想して欲しいですよ。
750ご冗談でしょう?名無しさん:2012/06/10(日) 10:36:36.49 ID:???
>>749
ゴミを撒き散らしてないで本買ってきて勉強しろよ
751ご冗談でしょう?名無しさん:2012/06/10(日) 11:02:12.03 ID:F7WrTU8z
皆さんに教えてもらいたいことがあるのですが
電子の自己エネルギーってありますよね
式が簡単なのでアマチュアにもとっつきやすいので私もいろいろ考えて
ブログになぞ書いたんですけど(趣味なのでほとんどイタイ系です)
次のストーリーで自己エネルギーというより電子にたまる値を計算したところ
それらしい値がでたんですけど

1.微小電荷を押込む、とりあえず距離0まで押込めるものとします。
2.とりあえずこの段階では一体にならず押込み力を抜くと反発していきます。
3.押込みに要するエネルギーと反発のエネルギー積分の差を計算する。
4.押込んだとき一体になるものとすると電子に蓄積されていくものは
この差分だろうとしてその総計を計算する。

教えてもらいたいのは3.のエネルギー合計の差を積分するのに発散を避けるため
微小電荷の運動ルートを複素数に拡張して留数(勉強中なのであやふや)を
使ったんですけどエネルギーのような実数の世界にいかに小さい(lim ->0 )とはいえ
複素数を持ち込んでもいいのでしょうか?


http://akimpotos.blogspot.jp/2012/06/blog-post.html
(アフィサイトではないです、念のため)
752ご冗談でしょう?名無しさん:2012/06/10(日) 11:32:35.02 ID:???
正則関数を扱う限り、実数か複素数かは気にしなくて良い。
レッジェ理論じゃ角運動量を複素数にしてるしね。
753ご冗談でしょう?名無しさん:2012/06/10(日) 12:27:49.01 ID:F7WrTU8z
>>752

レスありがとうございます。
あと直線での積分範囲が片側有限でも極をカバーしていれば留数をつかえると
うろ覚えですが記憶してます。これは本当でしょうか?
754ご冗談でしょう?名無しさん:2012/06/10(日) 13:06:43.83 ID:???

280 名前:名無しさん@12周年[sage] 投稿日:2012/01/13(金) 12:26:01.31 ID:sGoAT50E
三年前ごみんすに投票した連中、今どんな気持ち?

287 名前:名無しさん@12周年[sage] 投稿日:2012/01/13(金) 12:27:00.18 ID:P8V2Yy+50
>>280
橋下様なら何とかしてくれるとお経上げてるよ。
755ご冗談でしょう?名無しさん:2012/06/10(日) 13:08:35.68 ID:???
>>733
>電子と陽電子が対消滅してγ線が観測される現象
光子が2個(以上)出る
756イムジンリバー ◆NOsnoCwlwY :2012/06/10(日) 14:26:43.76 ID:???
>>746
ヒッグス粒子は質量そのもののメカニズムを説明したもの=慣性質量の源
重力子は重力のメカニズムを量子論的に説明したもの=重力質量の源
とくに関係はない
757ご冗談でしょう?名無しさん:2012/06/10(日) 14:42:42.86 ID:???
段差型ポテンシャル下で一次元のシュレーディンガー方程式の問題を考えているのですが
ポテンシャルの壁での反射率と透過率が確立密度の流れの比で表現される理由がわかりません
なぜでしょうか
758 忍法帖【Lv=40,xxxPT】 :2012/06/10(日) 14:45:29.86 ID:???
物理学の理論はワクワクするよ!?♪。
759ご冗談でしょう?名無しさん:2012/06/10(日) 14:45:39.61 ID:???
波数ってありますよね
cos(kx)とかsin(kx)のkが波数だっていうのはグラフ的にわかるんですけど
exp(ikx)のkを波数と呼ぶ理由がわかりません
オイラーの式からですか?
760 忍法帖【Lv=40,xxxPT】 :2012/06/10(日) 14:48:10.53 ID:???
オイラー孫悟空!?♪。
761ご冗談でしょう?名無しさん:2012/06/10(日) 15:01:44.72 ID:l6ztCJzQ
佐藤勝彦の分散方程式を見ろ。
762ご冗談でしょう?名無しさん:2012/06/10(日) 15:38:14.85 ID:???
>>757
簡単に言えば、単に入っていくものと出ていくものの比をとっているだけ


たとえば V(x) = V0 (0<x<a), 0 (otherwise) とする
反射率や透過率を、これに左から粒子を入射させたときの反射される確率・透過される確率として定義したい
これは量子力学的には、波束をポテンシャルに向けて入射させたあと、充分時間がたったあとのx<0に粒子がいる確率、x>aに粒子がいる確率にそれぞれ対応する

とりあえず透過率から考えると、 T = ∫[a, ∞] ρ(x, t=∞) dx (ρ(x,t)=|ψ(x,t)|^2)
ここで、 d/dt∫[a,∞] ρ(x,t) dx = j(x=a, t)
t=-∞ではx>aに粒子はいないとすると、 T=∫[-∞, ∞] j(x=a, t)dt

入射波数kを持つシュレーディンガー方程式の解を ψ_k(x)e^(-iω(k)t) と書くことにする(ω(k)=hbar k/2m)
波束ψ(x, t)はこれらの重ね合わせで、 ψ(x,t) = ∫A(k) ψ_k(x) e^(-iωt) dk と書ける(A(k)は重ね合わせの係数)
これから確率流束密度 j を計算してそれを使ってTを計算すると、公式 ∫exp(i(ω(k)-ω(k'))t)dt = 2πδ(k-k')/(∂ω/∂k) を使って、

T = 2π∫dk m/(hbar k) |Ak|^2 j_k(x=a) (ただしj_k(x) はψ_k(x)の確率流束密度とする)
ここで m/hbar k はψkの入射流束密度に他ならないので、
T = 2π∫dk |Ak|^2 j_k(x=a)/j_k_in と表される
これから、Ak があるk=k0 まわりに局在しているようなふつうの波束では、T=j(k0, a)/j_in(k0) となることが分かる
763ご冗談でしょう?名無しさん:2012/06/10(日) 15:42:38.71 ID:???
>>762
横だけど偉い
764ご冗談でしょう?名無しさん:2012/06/10(日) 16:03:23.81 ID:???
偉いとかなんで上から目線なんだよ
765ご冗談でしょう?名無しさん:2012/06/10(日) 16:07:28.63 ID:???
>>764
だって式書くのまんどくさいから
766ご冗談でしょう?名無しさん:2012/06/10(日) 16:16:11.10 ID:???
>>762
ありがとうございます
感動しました
767765:2012/06/10(日) 16:18:56.80 ID:???
確率密度が保存されていることをいってるだけど思ったけど
768 忍法帖【Lv=15,xxxPT】 :2012/06/10(日) 17:48:37.33 ID:???
スタートレックシリーズの様な物理学を実用化して欲しいですよ。
769ご冗談でしょう?名無しさん:2012/06/10(日) 22:50:09.61 ID:???
フェリ磁性の帯磁率の計算がどうやってもできないです。
分子場近似で考えてるんですが、フェリはそれぞれの磁化が違うてんでややこしくてよく解けないです。

http://beebee2see.appspot.com/i/azuYz7DRBgw.jpg
770 忍法帖【Lv=15,xxxPT】 :2012/06/10(日) 23:22:08.74 ID:???
グレンラガンの多元宇宙物理学ですよ。
771ご冗談でしょう?名無しさん:2012/06/10(日) 23:57:14.53 ID:???
>>753
対称性があって一周積分が片側だけで表わされる場合だろ。
772ご冗談でしょう?名無しさん:2012/06/11(月) 00:17:03.12 ID:???
>>769
M_1とM_2について解いてから、平均場近似の定義式にM_1とM_2を代入すればχが求まる。
773ご冗談でしょう?名無しさん:2012/06/11(月) 01:19:00.30 ID:PXwlUPDP
ちょっとした質問ですが、この空間に穴を空けるにはどうしたらよいのでしょうか。
774ご冗談でしょう?名無しさん:2012/06/11(月) 01:22:59.38 ID:???
超高速で正拳、あたたたた
775ご冗談でしょう?名無しさん:2012/06/11(月) 09:53:09.95 ID:???
>>733
>そうすると、対消滅の条件がとても厳しいものになりませんか?
対消滅で出るγ線が1個だけだと、条件が厳しすぎて反応が起こらない。
γ線が2個あるいはそれ以上出るとすると条件が緩くなり
エネルギー・運動量保存をどちらも満たす条件が可能になる。
実際、対消滅時には2個以上のγが出る。
776ご冗談でしょう?名無しさん:2012/06/11(月) 20:26:13.15 ID:???

このサイトの解説気合い入ってるなw

カブリIPMU、南部理論を拡張して同理論の適用例外をなくすことに成功
http://s.news.mynavi.jp/news/2012/06/11/096/index.html
777ご冗談でしょう?名無しさん:2012/06/11(月) 20:38:09.82 ID:???
>>775
対消滅すると、質量がいきなりゼロになるから、超微少だけど重力波が発生するの?
それともエネルギー=質量だから、その時点では発生しなくて、光が移動するとともに
重力波が発生するの?

光の変化=重力波にならないのかな
778ご冗談でしょう?名無しさん:2012/06/11(月) 21:38:04.26 ID:???
>>776
720
779ご冗談でしょう?名無しさん:2012/06/11(月) 22:25:18.76 ID:???
物理初心者なんだけど、誰か答えてくれると嬉しい
ネットで調べてもよくわからなかった・・・

ヒッグス粒子は、物質に質量を与えてる存在なんだよね?
じゃあさ、ヒッグスを除去することができるとしたら、俺らは光速で動けるようになるのかな
光速移動ができるとしたら、その状態で加速すると光速超えは可能?
やっぱり無理なのかなw

780ご冗談でしょう?名無しさん:2012/06/11(月) 22:30:17.25 ID:???
電子を取り除けたら
陽子を取り除けたら
光子を取り除けたら
781ご冗談でしょう?名無しさん:2012/06/11(月) 22:34:32.45 ID:???
ある事象が起こる確率が0から1の間の値でしかないように
物体の移動する速さvは0≦|v|<光速でしかありえない
仮にヒッグス粒子を除去できたとしたら光速とイコールの速さにはなれるが、超えることは不可能
782ご冗談でしょう?名無しさん:2012/06/11(月) 22:44:45.92 ID:???
>>781
ごめん、それを理解するには俺がバカすぎたw
そのへんは理解できるよう調べてみるわー

でも、超光速は無理だけど、光速は可能なんだねー
答えてくれてありがとう!
783ご冗談でしょう?名無しさん:2012/06/11(月) 23:28:00.81 ID:???
ひどくおおざっぱに言って、
ヒッグス粒子は物質に質量を与える粒子で、質量がある物体は光速より遅くしか動けない。
加えて、質量が全くない物体 (光) は光速でしか動けない (光速度不変の原理)。
だから、ヒッグス粒子が存在しないなら、質量がないなら、物体は光速で運動し、そうでないなら光速より遅い速さで運動する。
784ご冗談でしょう?名無しさん:2012/06/12(火) 01:35:50.69 ID:???
>>778
だから、何か?
785ご冗談でしょう?名無しさん:2012/06/12(火) 01:49:29.30 ID:sqTZDNr0
>>727って難しいですか?
スケボーのヴァートランプというのは断面Uの字型のでっかいコースで
選手はそこをスケボーでいったりきたりしながら空中でアクロバティックな技を決めるんですね

最初はU字の最高点より高い地点からコースに入っていくので勢いがついてるのはわかるけど
選手は時間内に何度となく空中でのトリック(わざ)を繰り返すわけです
漕ぐこともできないのにどうやってスピード≒高さを維持しているんでしょうか?
786ご冗談でしょう?名無しさん:2012/06/12(火) 01:55:50.02 ID:???
たぶんブランコと同じ

最高点ではしゃがんだ状態で
最下点を通過するときに立ち上がって位置エネルギーを与える
787ご冗談でしょう?名無しさん:2012/06/12(火) 03:01:17.82 ID:???
今度は屈伸動作でなぜブランコがこげるのかがよくわからんけど、スケボーの方の理由としてはきっとその通りなんでしょう。どうもありがとう
788ご冗談でしょう?名無しさん:2012/06/12(火) 03:06:47.97 ID:???
あ、わかった(ブランコも)
789ご冗談でしょう?名無しさん:2012/06/12(火) 04:00:16.48 ID:???
パラメタ励振
790ご冗談でしょう?名無しさん:2012/06/12(火) 10:02:28.88 ID:???
>>783
>>782だけど、なるほど、理解できたー
身近にそういったこと聞いても答えられる人いないから嬉しいわー
俺のアホすぎる質問にわかりやすく答えてくれてありがとね!





791ご冗談でしょう?名無しさん:2012/06/12(火) 11:03:45.40 ID:???
有効数字についてですが、定義としては位取りの0を除く最初に誤差が入ってくる桁までの数字だと思います。
まずこれ、あってますか?

あってるとして、デジタルのはかりで15.3gを得たとします。
この場合、表示最小桁の3までが有効数字と普通言われてますが、
よく考えると、この数字は内部的には

15.25〜15.34g

を意味しますから、これを15.30gと書いた時に3の桁には誤差が含まれず
最後の0の桁に誤差はありますから、有効数字は4桁じゃないかと思うのですが。
だから、この場合は

15.30g

と表記して有効数字を4桁とするべきではないですか?
どうでしょう?

792ご冗談でしょう?名無しさん:2012/06/12(火) 11:39:06.40 ID:???
3の桁に誤差が入っている。
よって有効数字は3桁。
793ご冗談でしょう?名無しさん:2012/06/12(火) 11:48:44.85 ID:???
>>791
> 15.25〜15.34g
君の頭は
> どうでしょう?
794ご冗談でしょう?名無しさん:2012/06/12(火) 12:08:17.65 ID:???
>>792
どうしてですか?誤差をあらわに表記したら

13.50 +/- 0.05 g

で、誤差の桁は小数第二位じゃないですか。
795ご冗談でしょう?名無しさん:2012/06/12(火) 12:14:42.09 ID:???
>>794
小数第二位の数字は全く信用できないから有効数字じゃない。
796ご冗談でしょう?名無しさん:2012/06/12(火) 12:16:44.10 ID:???
士0.05gてのが手前勝手な仮定だからwww
797ご冗談でしょう?名無しさん:2012/06/12(火) 12:21:54.98 ID:???
>>795
だから最初に定義を確認したんですが。
誤差が含まれる最初の桁までが有効数字なら、
小数第2位までが有効ですよ。

これがちょうど繰り上がるところだから問題になりますが、

15.35 +/- 0.05 g

なら小数第1位の3には誤差はないと、誰もが言うでしょう?
798ご冗談でしょう?名無しさん:2012/06/12(火) 12:35:55.30 ID:???
表示されてるのが少数第一位までなのに、勝手に第二位の数字を付け足しちゃいけない。
799ご冗談でしょう?名無しさん:2012/06/12(火) 12:45:22.23 ID:???
>有効数字についてですが、定義としては位取りの0を除く最初に誤差が入ってくる桁までの数字だと思います。
違う。最初に誤差が入る位の1つ前までが有効数字
800ご冗談でしょう?名無しさん:2012/06/12(火) 12:45:43.04 ID:???
>>797
全く信用できない数字と誤差が含まれる最初の桁は別物www
801ご冗談でしょう?名無しさん:2012/06/12(火) 12:46:31.38 ID:???
>>799
オマエさんも質問者程度のアホだな
802 忍法帖【Lv=40,xxxPT】 :2012/06/12(火) 13:24:47.31 ID:???
僕は物理学の応用でワープ技術と超光速航法とワームホールテクノロジーを実現するのに成功したよ!?♪。
803ご冗談でしょう?名無しさん:2012/06/12(火) 16:39:11.61 ID:???
>>799
1mmの目盛りのものさしで長さを読んだら、
最小目盛りの1/10で0.1mmまで有効数字ですけど、
確実に0.1mmの桁に誤差がありますよ。

>>798
予想される範囲の中央値を代表値とするのは、
とくにおかしくないでしょ。

何度量っても同じ15.3 gしか表示しないとしたら、
系統誤差なしとして、15.2gでもなく、15.4gでもなく
確実に真の値は15.25-15.35未満gの間にあるのに、
なぜそれを有効4桁と呼ぶことに抵抗があるのか、
まるで理解できない。
804ご冗談でしょう?名無しさん:2012/06/12(火) 16:51:12.23 ID:???
表示されてない数値を勝手に付け加えたり
何度測っても、なんていう条件を勝手につけ加えたり
なぜそんなに勝手気ままなのか
まるで理解できない。
805ご冗談でしょう?名無しさん:2012/06/12(火) 16:51:30.41 ID:???
>何度量っても同じ15.3 gしか表示しないとしたら、
>系統誤差なしとして、
系統誤差の意味わかってる?
806ご冗談でしょう?名無しさん:2012/06/12(火) 16:53:39.85 ID:???
1mmの目盛りのものさしで長さを読んだら、
値が152.5mmー153.5mmの範囲にまんべんなくばらついた。

誤差をそのまま広がりで表記して

153.0 +/- 0.5mm

これなら有効数字4桁で何も問題ないはず。

しかし電子天秤はほぼ同じ

15.30 +/- 0.05g

という誤差表記ができるのに、3桁だと言う。

おかしいでしょ。

このばらつき具合が見えているか見えていないかだけで、
本質的に同じなんですけどね。
807ご冗談でしょう?名無しさん:2012/06/12(火) 16:58:53.23 ID:???
>>806
メーカーに聞けよ、厨房
808ご冗談でしょう?名無しさん:2012/06/12(火) 17:14:27.56 ID:???
> 1mmの目盛りのものさしで長さを読んだら、
> 値が152.5mmー153.5mmの範囲にまんべんなくばらついた。
>
> 誤差をそのまま広がりで表記して
>
> 153.0 +/- 0.5mm
>
> これなら有効数字4桁で何も問題ないはず。
思い込みだな
809ご冗談でしょう?名無しさん:2012/06/12(火) 17:33:31.26 ID:???
>>808
では、あなたはこの有効数字を何桁と言いますか?
810ご冗談でしょう?名無しさん:2012/06/12(火) 17:38:07.69 ID:???
有効数字という概念は簡便だが曖昧なので、
(このような議論にまさに定義の曖昧性が現れている)
不確かさ付きの表記法を用いることにすればわざわざ使う必要もないだろう。

なお、内部実装として単なるbanker's roundingを用いているのであれば、
矩形分布の標準偏差を考えることで、
Bタイプ不確かさとして±0.03gが得られる。
別にこれは>>796の言うように勝手な仮定ではなく、
実験屋は普通にやっていることだと思うが。

>>805
表示しない桁を全て切り捨ててるような測定機器があったとすれば、
そのような機器には系統的バイアスが含まれると考えるのが普通だと思うが。
811ご冗談でしょう?名無しさん:2012/06/12(火) 17:38:12.85 ID:???
3けた
812 忍法帖【Lv=17,xxxPT】 :2012/06/12(火) 17:45:44.36 ID:???
物理学者はスタートレックシリーズを馬鹿にするなよ。
813ご冗談でしょう?名無しさん:2012/06/12(火) 18:09:11.91 ID:???
>>810
>>>805
>表示しない桁を全て切り捨ててるような測定機器があったとすれば、
>そのような機器には系統的バイアスが含まれると考えるのが普通だと思うが。
全面的に同意するがなぜそれが漏れへのレスなんだ?
814ご冗談でしょう?名無しさん:2012/06/12(火) 18:23:51.23 ID:???
>>813
>>803
> 系統誤差なしとして
という意味は普通には、「>>810のようなことがない」という意味に解釈できると思うので、なぜ>>805
> 系統誤差の意味わかってる?
というレスをしたのか意味が分からなかった。
815ご冗談でしょう?名無しさん:2012/06/12(火) 18:32:52.80 ID:???
>>805
日本語の意味の解釈ちがい
816ご冗談でしょう?名無しさん:2012/06/12(火) 19:06:40.12 ID:???
>>810
ありがとうございます。
不確かさを1/√12で計算すれば、小数第2位まで使うということですよね。

そうすると有効数字の”常識”にしたがって3桁で考えると、
他の測定値と組み合わせて計算する時に、
他の計測値の有効な情報まで捨ててしまうことになるので、
有効数字の定義云々ではなく、やっぱりよくないような気がします。
817ご冗談でしょう?名無しさん:2012/06/12(火) 19:10:33.14 ID:???
おまえの常識を聞いているわけではない
メーカーが3桁といえば3桁
818ご冗談でしょう?名無しさん:2012/06/12(火) 19:28:41.42 ID:???
有効数字はもともと大雑把
情報の損失を避けるのなら標準偏差を計算する。
万有引力定数だと G=6.67384±0.00080 ×10^(-11) m^3/s^2/kg
となっててGの誤差(標準偏差)を有効数字2桁で出してる。
これをGの有効数字が4桁とすると情報が減って誤差が過小評価になる。
819ご冗談でしょう?名無しさん:2012/06/12(火) 20:44:47.01 ID:???
まとめ

計量をわかっているやつはいない
820ご冗談でしょう?名無しさん:2012/06/12(火) 21:34:42.13 ID:???
どの計量?
821ご冗談でしょう?名無しさん:2012/06/12(火) 21:37:39.22 ID:???
はかりだよーん
822 忍法帖【Lv=17,xxxPT】 :2012/06/12(火) 21:43:03.12 ID:???
物理学的にワープ技術とか超光速航法とかワームホールテクノロジーを実現出来ますか?。
823ご冗談でしょう?名無しさん:2012/06/12(火) 21:44:50.57 ID:???
すいとんしてみたーい!?
824ご冗談でしょう?名無しさん:2012/06/12(火) 21:55:25.34 ID:???
>>818
情報の損失がおこるような有効数字に何の意味があるんでしょう?

本来有効数字って、必要な情報を失わない範囲で最大限の省力化をするものだと思うんですけどね。
そもそも手計算の時代なら、不確かさを計算するまでに有効数字は必要になるので、
計算してみたら桁が足りませんでしたでは困るんですよね。

不確かさを計算するまでは当然正確にどこまで必要かが分からないので、
100%ここから下はいらないという桁と、ここより上は100%正確という桁の間にどうしたってグレーゾーンが出てきます。
有効数字を考える場合、このグレーゾーンは全部取り込んでおかないとあとで桁が足りなくなる恐れがあるので危ない。
にもかかららず、最近はどうもグレーゾーンは全部切ってしまえという考えの人が多くなってる気がするんですよね。
とくに有効数字を数学の人が中学校で教えるようになって以来。
825ご冗談でしょう?名無しさん:2012/06/12(火) 22:10:21.96 ID:???
>>824
ky
826ご冗談でしょう?名無しさん:2012/06/12(火) 22:21:59.93 ID:m6yIXVfl
>>771

>>751 です
ありがとう、対称性ですか。
私のほうも留数でエネルギーが出るようになりました。
何のことはない対数発散のワイスコップ( Weisskopf) の計算したもので
量子電磁力学を用いて電子の自己エネルギーを出す式を使えばその
極近傍の微分が1/x の逆一乗になるのでそれから発散しないエネルギーに
することができました。
(とはいえ相変わらず結果が正しいかどうかは不明ですが)

827ご冗談でしょう?名無しさん:2012/06/12(火) 22:25:24.33 ID:???
>>826
ブジョルケン読めよ
828ご冗談でしょう?名無しさん:2012/06/12(火) 23:23:28.19 ID:bNNqbq/A
>>824
下の桁を全部丸めてる時点で既に情報が欠損しているが。
一回丸めてしまった値を使って更に計算しようと思った際、
有効数字のルールに則って計?
829ご冗談でしょう?名無しさん:2012/06/12(火) 23:29:17.81 ID:???
>>824
下の桁を全部丸めてる時点で既に情報が欠損しているが。
一回丸めてしまった値を使って更に計算しようと思った際、
有効数字のルールに則って計算結果の有効桁数を出して再度丸めたら、丸め誤差が蓄積してしまう。
830ご冗談でしょう?名無しさん:2012/06/12(火) 23:46:16.80 ID:???
有効数字全否定ですか。今の世の中そういう極論もありですかね。
無意味な情報は切り捨てても欠損とはいいませんけどね。
831ご冗談でしょう?名無しさん:2012/06/12(火) 23:52:37.08 ID:???
>無意味な情報は
日本語も読めないやつに物理は無理だな
832ご冗談でしょう?名無しさん:2012/06/13(水) 00:33:35.13 ID:???
延々と長文を書いて自分の馬鹿を晒すとはごくろうなこった。
まあタミなんかはいつもそれだが。
833ご冗談でしょう?名無しさん:2012/06/13(水) 10:03:57.71 ID:???
>本来有効数字って、必要な情報を失わない範囲で最大限の省力化をするものだと思うんですけどね。
失わない範囲で、ではなく、多少失ってもいいから省力化を図るものですね。
情報を失っちゃダメなら有効数字などという大雑把な表現方法に頼ってはいけない。
834ご冗談でしょう?名無しさん:2012/06/13(水) 10:12:41.31 ID:???
あと、丸め誤差の蓄積を避けるため、途中計算では有効数字の桁数の
最低1桁増しまでは残しておいて最後に丸める、というのも
有効数字を習う際には注意されることだと思う。
835ご冗談でしょう?名無しさん:2012/06/13(水) 10:22:57.40 ID:???
>>833
それって言い回しの違いだけで同じことでは?
多少というのは「必要な情報を失わない範囲」でしょ。
最終的に出る不確かさの桁までは残さないといけないということに異論があるとは思えないのですが。

丸めの蓄積というのも、5桁目を丸めて計算し、最終結果は3桁目を丸めたななら、
そんな丸めの蓄積なんて微々たるものだと思いますけどね。
最終的に4桁目を丸めても限定的だと思いますけど。
836ご冗談でしょう?名無しさん:2012/06/13(水) 10:39:29.45 ID:???
>>835
ここでは情報を失うか失わないかが論点なので、同じことではない。
多少であろうが「失う」ことにかわりなく、失わないと思っているなら
認識を改めたほうがよい

>>834の後半はまさに
>丸め誤差の蓄積を避けるため、途中計算では有効数字の桁数の
>最低1桁増しまでは残しておいて最後に丸める
という話であって、何をクレームつけたつもりなのかわからない
837ご冗談でしょう?名無しさん:2012/06/13(水) 12:54:48.73 ID:???
誤差が大きい事も小さい事も情報なんだから、桁を多めに取った所で情報損失に変わりはない。
有効数字なんてその程度の物。
838ご冗談でしょう?名無しさん:2012/06/13(水) 15:50:34.83 ID:???
合成関数の外積が分かりません

∇xf(u)= はなんですか?
839ご冗談でしょう?名無しさん:2012/06/13(水) 15:53:14.67 ID:???
各成分ごとに計算すれば単なる偏微分なので誰でもできる、やれ
840ご冗談でしょう?名無しさん:2012/06/13(水) 16:01:02.97 ID:???
(2)が解けません...
誰か助けてください
http://www.imgur.com/NIVAy.jpg
841ご冗談でしょう?名無しさん:2012/06/13(水) 16:14:52.32 ID:???
>>839

=( (∂u/∂y) ・ (∂f3(u)/∂u ) - (∂u/∂z) ・ (∂f2(u)/∂u ) ,
   (∂u/∂z) ・ (∂f1(u)/∂u ) - (∂u/∂x) ・ (∂f3(u)/∂u ) ,
   (∂u/∂x) ・ (∂f2(u)/∂u ) - (∂u/∂y) ・ (∂f1(u)/∂u ) )

この先がどうしても分からないのです
どなたかお願いします
842ご冗談でしょう?名無しさん:2012/06/13(水) 17:21:43.55 ID:QAXzg5vf
標準偏差を計算しても、そこより下の桁は捨ててるんだから、やっぱり情報の欠落はおこってる。
だから有効数字を使っても同程度の欠落なら何の問題もないのに、
何でむきになって有効数字を否定しにかかってるんだろう?
全く理解できないな。

一切捨ててはならんと言うなら、標準偏差で切ることもやめにゃならんのだが。
843ご冗談でしょう?名無しさん:2012/06/13(水) 17:26:02.12 ID:???
>>841
これ以上、なにを望む?
844ご冗談でしょう?名無しさん:2012/06/13(水) 18:28:15.10 ID:???
>>842
>有効数字を使っても同程度の欠落なら何の問題もないのに、
同程度の欠落なら、な。だけど有効数字は欠落の程度が大きい。
1.0と9.9は0.1の位に同程度の誤差があるとすると相対誤差としては
ほぼ10倍違うが、有効数字で扱うとどちらも区別なく2桁の扱い。

>一切捨ててはならんと言うなら
誰もそんなこと言ってない。有効数字はその程度のものだと言っているだけ。
つまり、その程度のものだと認識して使うなら問題ないと言っているのと同値
845ご冗談でしょう?名無しさん:2012/06/13(水) 18:46:49.24 ID:???
>>842
読解力の無さが不思議
846ご冗談でしょう?名無しさん:2012/06/13(水) 18:54:08.03 ID:???
>>841
=(∂u/∂x,∂u/∂y,∂u/∂z)×(∂f1(u)/∂u,∂f2(u)/∂u,∂f3(u)/∂u)
=(∇u)×(∂f/∂u)
847ご冗談でしょう?名無しさん:2012/06/13(水) 19:12:43.27 ID:og3r2+0X
>>844
だから計算過程では一桁余分に使うんだし、
1かそれに近い時はさらに一桁余分に使うなんてのも常識化してるだろ。
有効数字を考えて同程度にできないって、頭が悪いだけ。
誤差を含んだ桁の4桁も5桁も下に意味があるなんてありえないんだから。

で、話をもとにもどすと、アナログなら注意してそれで使えばいいけど、
デジタルでは最小一桁を有効数字にしてしまうと始めから有効桁を一桁捨ててることになるから、
それはまずいでしょ。で、問題があるなら改善すればいいんだから、最小桁のひとつ下まで有効桁にしたらいいんじゃないの?

あと、天秤はただの例で、これはデジタル一般の話だから。
メーカーの指定があるならそれにしたがうのは当然で今はそれは考えない。
848ご冗談でしょう?名無しさん:2012/06/13(水) 19:20:38.35 ID:+5zM9bC1
まあ、好きにしな
849ご冗談でしょう?名無しさん:2012/06/13(水) 20:17:51.32 ID:???
>>847
> デジタルでは最小一桁を有効数字にしてしまうと始めから有効桁を一桁捨ててることになるから、
> それはまずいでしょ。で、問題があるなら改善すればいいんだから、最小桁のひとつ下まで有効桁にしたらいいんじゃないの?
デジタルで、一桁目を最小桁にすることは、物差しでは目盛の数値を最小桁にすることと同等なんだから、

>>803
> 1mmの目盛りのものさしで長さを読んだら、
> 最小目盛りの1/10で0.1mmまで有効数字ですけど、
> 確実に0.1mmの桁に誤差がありますよ。
のように目盛の1/10を読んだ時と同様ではない
850ご冗談でしょう?名無しさん:2012/06/13(水) 20:39:39.85 ID:???
>>849
だから同等にするために最小桁じゃなくてもう一桁下まで有効にしたら?という話ですけど。
851ご冗談でしょう?名無しさん:2012/06/13(水) 20:39:44.26 ID:???
>1かそれに近い時はさらに一桁余分に使うなんてのも常識化してるだろ。
全面的に同意するが、それは既に有効数字の概念を超えた扱い。
有効数字てこんなもんという指摘に対して、有効数字を超えた扱いを
持ってくる、というのは反論の体をなしていない
852ご冗談でしょう?名無しさん:2012/06/13(水) 20:43:07.21 ID:???
もう一桁下まで使うのは勝手にすればいいが、
一桁下まで有効数字だと勝手に俺様定義したら
話が通じるわけがない
853ご冗談でしょう?名無しさん:2012/06/13(水) 20:48:39.53 ID:???
>>850
> 目盛の1/10を読んだ時
は目盛の1/10まで読むから、読んだ桁である目盛の1/10を最低桁にするんだが…

1/10ではなく、目盛自体を読めば、デジタルはかり同様目盛を最低桁にする
854ご冗談でしょう?名無しさん:2012/06/13(水) 20:57:03.85 ID:???
>>851
有効数字を考慮した実験データの取り扱いを解説している本には書いてありますよ。全部とはいいませんが。
これを有効数字の概念を越えたとか言ってるのは有効数字の扱い方を知らないだけじゃん。
有効桁で打ち切って計算する人などいるわけないのに。

>>852
だから俺様定義じゃないようにするために提案してみて様子を見てるわけですが。
常識に反することを言えばこんなふうに反発を受けるだろうことはわかってましたが。

まあ、結局のところ今さら有効数字を考慮してもしょうがないから、
教える事がそもそも無駄って事ですかね。
855ご冗談でしょう?名無しさん:2012/06/13(水) 21:02:17.64 ID:???
∂(xj)=0  j=電流密度

になるみたいなんですが
どの公式や定理を使ってるのですか?
お願いします
856ご冗談でしょう?名無しさん:2012/06/13(水) 22:24:36.84 ID:???
>有効桁で打ち切って計算する人などいるわけないのに。
だれも途中で有効桁で打ち切って計算しろなどと言ってない。
言ってもないことに文句言われても困る。

1に近いかどうかで扱いを変える話だよ、有効数字の概念を超えると表現したのは。
有効数字は要するに最小桁に不確定な数字があるように表現したもの。
1に近いかどうかは関係ない。最小桁のもう一つ先まで意味がある、という
状況のことを言っているのだと思うけど、それは既に有効数字の定義を
超えて標準偏差的扱いに入り込んでいる。違うというのなら、
1に近いかどうかで有効数字の桁数の定義を変えている教科書を
提示して欲しい。少なくとも漏れは見たことがない。1に近いときは
相対誤差が大きくなるよ、という注意書きなら見たことはあるが。

有効数字の桁数を出す規則(掛け算割り算では最小桁数に合わせなさい、
というやつ)でも、最初の数字が1かどうかで場合分けしたものなど
見たことがない。計算結果で最初の数字が1になったときに
桁数を1つ増やして答えとして提出したら間違いなく×をもらうだろう

最後の部分は全面的に同意で、誤差の定量性が必要なら有効数字などでは考えず、
しかるべき統計的手法を取る。逆に、有効数字で考えているときに定量性を
過度に考慮してたら有効数字で考えることのメリットを捨てるも同然で
あまり賢くない。
857ご冗談でしょう?名無しさん:2012/06/13(水) 22:26:48.80 ID:???
>>855
∂(xj) というのが何を意味しているつもりで書いているのかよく分からない
∂はどの変数での微分で、xjはなに? 二階テンソル?
858ご冗談でしょう?名無しさん:2012/06/13(水) 22:59:39.63 ID:???
>>857
xは位置です
∂xk(xi・jk)が のkが3つ(3次元)のとき
どのiであっても
∂xk(xi・jk)=0 になるそうです
859ご冗談でしょう?名無しさん:2012/06/13(水) 23:05:45.89 ID:???
860ご冗談でしょう?名無しさん:2012/06/13(水) 23:10:34.50 ID:???
∂(xi・jk)/∂xk=0
です
861ご冗談でしょう?名無しさん:2012/06/13(水) 23:13:02.75 ID:???
>>858
一般のjに対して0になるとは思えないけど…

単に、積分の中で出てきたから表面積分に直して消してるとかじゃないの?
積分領域が全空間のときに ∫dV∂_k (x_j j_k) = ∫dS_k x_i j_k = 0 (無限遠で電流密度が充分すみやかに0になることを仮定)
862ご冗談でしょう?名無しさん:2012/06/14(木) 00:11:46.35 ID:???
飛行機が飛ぶ原理の話なんですが、前進し始めた翼の上面後方には負圧が生じるそうなんですが、それはどうしてですか?
お願いします。
863ご冗談でしょう?名無しさん:2012/06/14(木) 00:30:43.10 ID:???
864ご冗談でしょう?名無しさん:2012/06/14(木) 00:35:50.33 ID:???
>>863
すみません、流体の教科書には飛行機の飛行の原理は書いていなくて・・・
大学2年ですが、流体は今履修中で、ほとんど無知に等しいです。
865ご冗談でしょう?名無しさん:2012/06/14(木) 00:52:35.37 ID:???
>>862
翼の上面の前方が上がって後方が下がっている場合、上面を通る気流は翼に沿って流れ、下向きの気流となる。
これは空気に下向きの運動量を与えた事になるから、翼には上向きの力が働く。
気流が翼面に追随しない場合を考えると斜めに下がった翼面の上を水平に気流が流れる事になり、気流と翼面の間の三角形部分の空気も気流に連れ去られて真空になる。
実際はその負圧で気流は下へ翼は上へ吸い上げられる。
866ご冗談でしょう?名無しさん:2012/06/14(木) 01:04:24.11 ID:???
>>855
電流で微分がゼロと言ったら連続の式(電荷保存則)かもしれんが、記法がサッパリ分からんな。
相対論の4元形式なら ∂J^i/∂x^i=0 だが。( J^i は電流密度反変ベクトル, i は反変添字 )
867ご冗談でしょう?名無しさん:2012/06/14(木) 01:49:15.29 ID:???
>>865
よくわかりました。
ありがとうございました。
868 忍法帖【Lv=19,xxxPT】 :2012/06/14(木) 05:01:41.39 ID:???
もっと更に楽観的にワープ技術とか超光速航法とかワームホールテクノロジーとか亜空間移動とか超空間飛行とか恒星間航行とか惑星間飛行を真面目に実現する方法を考えて欲しいですよ。
869ご冗談でしょう?名無しさん:2012/06/14(木) 08:25:26.34 ID:???
あ・・・
どうも積分記号見逃していたようで
∫dV∂_k (x_j j_k) = ∫dS_k x_i j_k = 0
でした
これは何の法則ですか?
870ご冗談でしょう?名無しさん:2012/06/14(木) 08:36:47.60 ID:???
この前TVで水槽の中にピンポン玉入れて密封して、水槽を前に動かしたらピンポン玉が水槽の中で前に進んでたんだ。
水槽引っ張る速度よりピンポン玉のほうが速く動いて物理に疎い自分には驚愕だったんだけど、光子って質量0だったよね確か
光子より重い質量を持つ物質に囲まれた光子がいて、重い物質が動いたら光子は速度+αで光速超えないの?
観測できないとかいうオチ?脳無い自分にわかりやすく教えて欲しいorz
871ご冗談でしょう?名無しさん:2012/06/14(木) 09:11:30.15 ID:???
>>869
k=1,2,3で和をとるとするとガウスの発散定理
872871:2012/06/14(木) 09:20:24.02 ID:???
おっと、ガウスの発散定理は∫dV∂_k (x_j j_k) = ∫dS_k x_i j_kの部分ね。
それが=0になるというのは一般には言えないことなので、xもしくはjに
何らかの仮定が入っているはず。考えている領域の中にjの湧き出しがないとか。
873ご冗談でしょう?名無しさん:2012/06/14(木) 09:55:23.59 ID:???

>>840
粘性抵抗力6πηr v を受けるとして、バネからの力、重力に加えて、運動方程式をたて、解けばいいんだろ?
874ご冗談でしょう?名無しさん:2012/06/14(木) 10:06:25.16 ID:???
>>870
ピンポン玉の動きは水流で決まる。
ピンポン玉がいくら軽くても水流より速くはならない。
875ご冗談でしょう?名無しさん:2012/06/14(木) 11:10:04.21 ID:???

「国の借金」はデマ

日頃メディアや、反日工作員が必死になって「国の借金」という単語を使い
財政破綻論を展開させていますが、現実、現在の日本には「政府の借金」は1000兆円近く存在いたしますが、
「国の借金」は存在いたしません。

朝日新聞やNHKは、雇い主である中国共産党より日本人に対して不安や政府に対する不信を持たせ、煽るために
局内の共産党員を使用して既に数十年間、「国の借金」を連呼し続けております。

<違和感なく「国の借金が1000兆円もある」という幻想に浸ってしまっている一般の方々は、朝日新聞やNHKに見事に騙され続けている訳です>

数十年もテレビや新聞から情報を得てきた方々の中には、「メディアが嘘を付く訳ない」と思う、そう思いたい方もいるでしょう。
  しかし、長い目で見れば、もともと戦前から日本を転覆させるために存在してきた報道機関ですから、
これくらいの嘘は朝飯前で御座います。

それでも、「国の借金は1000兆円ある」と考えをお持ちの方は、複式簿記の勉強をしてから、日本のバランスシートをご覧ください。
 
  実質中国の広報機関であるNHK、朝日新聞は、これからも嘘を付き続けます。デマを流し続けます。捏造し続けます。
   
 ---そのニュース 核心は、デマだ       長文失礼いたしました。---
876ご冗談でしょう?名無しさん:2012/06/14(木) 12:31:44.02 ID:???
>>874
>>870の現象は水流じゃないぞ。浮力だ。
要するにピンポン球が水に浮く現象を横向きに見ているだけ。
正確には水槽を「前に動かしたら」ではなく「前に加速させたら」のはず。
877ご冗談でしょう?名無しさん:2012/06/14(木) 12:59:35.59 ID:???
>>874
ごめんなさい、密閉した水槽の中なので水流とかはあまり…
しかも水槽の速度以上でピンポン玉が実際に動いてました

>>876
確かに置いた状態から動かしてたから正確には「加速させたら」でした
説明ではピンポン玉の質量が水より軽いから云々言ってたんですがいまいちピンとこなくて…
浮力でもピンとこない自分にもう少し詳しく教えてほしいです
878ご冗談でしょう?名無しさん:2012/06/14(木) 13:12:36.18 ID:???
水槽を押した力は水中を音速で伝わる。ピンポン玉の動きは音速よりはずっと遅いだろう。

加速中の水槽を水槽に固定した系でみると横向きの慣性力が加わる。
慣性力は重力と区別がつかないので、慣性力が加わったぶんだけ
浮力も横向きにも働くようになる。それだけの話。
879ご冗談でしょう?名無しさん:2012/06/14(木) 13:18:18.22 ID:???
>日本人に対して不安や政府に対する不信を持たせ、煽るため
「政府の借金」を「国の借金」と言い換えることで
どうやって上記の目的が達せられることになるというのか。
政府に対する不信を煽りたいならむしろ「政府の借金」と
言ったほうがよいのでは?
自説の辻褄くらい合わせろよ、脳足りん
880ご冗談でしょう?名無しさん:2012/06/14(木) 13:32:59.88 ID:???
あちこちで見るコピペにマジレスwww
881ご冗談でしょう?名無しさん:2012/06/14(木) 13:34:53.59 ID:???
>>865
>気流が翼面に追随しない場合

ちなみにそれはいわゆる失速状態ですな。
揚力が急減したりする
882ご冗談でしょう?名無しさん:2012/06/14(木) 13:39:19.15 ID:???
>>877
水槽を前に加速すれば後ろが下になったのと同じ。
ピンポン玉は浮力で前に動こうとし、ピンポン玉の前の水は後ろに動こうとする。
実際に動くにはピンポン玉の前の水が後ろに流れなければならない。
つまり水流が生じる。
水は粘性が小さいから水槽の速度より速く動けるのでピンポン玉も同様に動ける。
883ご冗談でしょう?名無しさん:2012/06/14(木) 13:58:56.05 ID:???
平均・分散・誤差を求めたところでそれらをどのように評価すればいいのかわからん

「理論値」みたいなものがあればいいんだがコンピュータによる多体系の数値計算だからそもそも解析的な解もわからない
884ご冗談でしょう?名無しさん:2012/06/14(木) 18:30:20.01 ID:???
系や近似(ニュートン力学など)を除いた
全物理法則はどれどれでどのくらいの数になりますか?
885ご冗談でしょう?名無しさん:2012/06/14(木) 18:35:56.17 ID:???
エネルギー・運動量・角運動量なんかの保存を見ればいいってこと?
886ご冗談でしょう?名無しさん:2012/06/14(木) 18:39:52.51 ID:???
>>884
近似を除くのなら0だな。
887ご冗談でしょう?名無しさん:2012/06/14(木) 18:41:05.32 ID:???
>>886
いや・・・それはないでしょう
それなら無なわけですから
888ご冗談でしょう?名無しさん:2012/06/14(木) 18:59:07.19 ID:???
マクスウェル
相対性
889ご冗談でしょう?名無しさん:2012/06/14(木) 19:01:44.75 ID:???
一番基本の場の量子論が有限項で近似しないと発散する級数だからな。
それでも繰り込みが発明される前は1次項で発散してたんだから4次項まで計算できて超高精度の結果が出る現在は大進歩。
890ご冗談でしょう?名無しさん:2012/06/14(木) 19:02:23.46 ID:???
>>887
物理法則として人間に認知されている数式は、
おそらく全部、ある特定の範囲でのみ有効な有効理論でしかない

>>888
Maxwell方程式で真空の絶縁破壊記述できないでしょ
891ご冗談でしょう?名無しさん:2012/06/14(木) 19:03:58.22 ID:???
もちろん、マクスウェル方程式も一般相対論も場の量子論の近似だもんね。
892ご冗談でしょう?名無しさん:2012/06/14(木) 19:10:42.30 ID:???
>>884ですがそうなのですか・・・
大学の初歩物理を学んで
「コレが宇宙の本当の法則なのだな」と思ってのですが
まだまだなのですね

何事も基本はシンプルな数式だと思ってたら
それは簡易化されてたのを見ていただけだったのか

場の量子論って
統一理論みたいなものですか?
893ご冗談でしょう?名無しさん:2012/06/14(木) 19:29:47.59 ID:???
みたいですね
場の量子論は現段階ではいくつの
公式であらわされてるんでしょうか?
894ご冗談でしょう?名無しさん:2012/06/14(木) 19:33:02.33 ID:???
場の量子論は場に対して量子力学を適用しただけの物。
対象とする場により色々。
超弦場を対象とした超弦理論で統一理論を作ろうとしている。
895ご冗談でしょう?名無しさん:2012/06/14(木) 19:37:46.47 ID:???
いくつの公式たってね〜
素粒子の数だけ公式があるとも言えるし、全部足して一つとも言える。(使いようがないけど)
896ご冗談でしょう?名無しさん:2012/06/14(木) 21:01:04.40 ID:JtTX3jHx
賢者の石と、フォトジェニック水晶を間に入れ、次に超プラチナを入れて、放射能消去できる 
                                                      mai 登記
897ご冗談でしょう?名無しさん:2012/06/14(木) 23:44:34.02 ID:???
濃度とか温度が違う溶液を混ぜた時にもわもわってなる現象ってどんな名前ですか?
898ご冗談でしょう?名無しさん:2012/06/15(金) 00:05:11.26 ID:VLktp72s
身近にある物音を組み合わせて人間の音声に近いものってつくれますか?
899ご冗談でしょう?名無しさん:2012/06/15(金) 00:59:54.78 ID:???
>>892
>「コレが宇宙の本当の法則なのだな」と思ってのですが

「万物の理論」Theory of Everything. 略してToE.
人類が未だ手にしていない見果てぬ夢だよね。
http://en.wikipedia.org/wiki/Theory_of_everything

ちなみに「ホーキング、宇宙と人間を語る」では、
おそらく1つの理論で説明できる万物理論は存在しないだろう
という見解でしたな。

まあ、どうなることやら。
900ご冗談でしょう?名無しさん:2012/06/15(金) 01:34:01.33 ID:???
ビオサバール積分を四元形式で書くとどうなりますか
901ご冗談でしょう?名無しさん:2012/06/15(金) 04:05:56.93 ID:???
ベクトルポテンシャルとスカラーポテンシャルを並べて1つの4元ポテンシャルにする。
902ご冗談でしょう?名無しさん:2012/06/15(金) 04:06:57.60 ID:???
ビオ=サバールの法則は遠隔作用の表現だから相対論的には書けないでしょ
903ご冗談でしょう?名無しさん:2012/06/15(金) 11:36:35.41 ID:???
>>898
作れる
波の合成パターンなんて無数にあるし

お前のオナラの音を合成するだけで
完璧なオーケストラサウンドでも
ノイズミュージックでも
何で作れる
904ご冗談でしょう?名無しさん:2012/06/15(金) 11:39:37.85 ID:???
正弦波しか出せないチップでもそれなりに聴けるのが作れるしな
905ご冗談でしょう?名無しさん:2012/06/15(金) 11:59:31.22 ID:???
「オウムは統一教会をラジカルにしたもの」
「オウムが行く前に統一教会が、ロシアに進出していました。ところが、そういう連中が、どうも何時の間にかオウム信者とすりかわってしまった。」
層化(公明・維新)、統一(自民)、オウム、総連、民団→朝鮮人だらけの民主党
すべて繋がっている


----------------------------------
↑一連の騒動で「政治とオウムは一心同体」この繋がりご理解頂けただろうか
今日は消費税をどうするか決める日

今日まで泳がせてたのはそのため。
平田の出頭菊池の逮捕、今日迄すべて台本通り 
               この一連の騒動が、自民・民主の「基本合意」の正体
906ご冗談でしょう?名無しさん:2012/06/15(金) 12:01:37.85 ID:???
全部政治的な動きに絡めないと気が済まない低能さんおいすー^q^
907ご冗談でしょう?名無しさん:2012/06/15(金) 12:32:36.77 ID:???
>>902
なるほど、ありがとうございます
908ご冗談でしょう?名無しさん:2012/06/15(金) 17:04:01.01 ID:???
>>897
屈折
909ご冗談でしょう?名無しさん:2012/06/15(金) 17:55:25.28 ID:???
http://www.youtube.com/watch?v=1sP_t8v09Qc

この映像はなぜオモロイのでしょうか
910ご冗談でしょう?名無しさん:2012/06/15(金) 20:21:38.32 ID:???
ワロタw
911ご冗談でしょう?名無しさん:2012/06/16(土) 01:10:20.06 ID:LkZqCFwe
ちょっと質問です
惑星が公転するときの運動エネルギーって重力のポテンシャルからくるんだっけ
なんで重力は減らないんだろ?
初歩的ですまんがお願いします
912ご冗談でしょう?名無しさん:2012/06/16(土) 01:52:15.40 ID:LkZqCFwe
ちょっと追加です
電子だと放射光出してだめになるんですよね?
いきなりわからなくなってしまった
913ご冗談でしょう?名無しさん:2012/06/16(土) 02:20:25.81 ID:???
>>912
初歩って話ですらないが
重力ってのは保存力だからエネルギーが他の力 例えば空気抵抗みたいな場合と違って散逸しない
ただし一般相対論の効果まで含むと電磁場と同じように重力波を放出してエネルギーを失ってしまうが
太陽系レベルなら無視しても構わない
914ご冗談でしょう?名無しさん:2012/06/16(土) 12:50:26.34 ID:LkZqCFwe
>>913
なんとなくわかったかも
一瞬永久機関みたいに見えて混乱してしまった
915ご冗談でしょう?名無しさん:2012/06/16(土) 16:54:36.05 ID:???
フント則にしたがう多電子系において、基底状態では全合成角運動量J↑は、
全合成スピン角運動量S↑・全軌道角運動量L↑と向きが平行または反平行とあるのですが何故そういえるのでしょうか?
また、基底状態でないなら平行または反平行ではないケースもあり得るのですか?

よろしくお願いします
916ご冗談でしょう?名無しさん:2012/06/16(土) 17:28:28.01 ID:???
適当だけどスピン軌道相互作用のせいじゃね
917 忍法帖【Lv=40,xxxPT】 :2012/06/16(土) 18:27:12.13 ID:???
物理学VSキングダムハーツシリーズ!?♪。
918ご冗談でしょう?名無しさん:2012/06/16(土) 19:11:27.99 ID:???
電子スピンは外部に対して平行または反平行の自由度しかない。
ということは全軌道角運動量に対して平行または反平行。
全合成スピン角運動量も平行または反平行になる。
919ご冗談でしょう?名無しさん:2012/06/16(土) 19:50:31.52 ID:???
>>916
その可能性も考えたのですが……

>>918
そうなんですか!
スピンのx,y,z成分、再差運動等はスピンのベクトルとはまた違う概念なんですか?
角運動量のx,y成分がオブザーバブルではないことと関係してるのかな
920ご冗談でしょう?名無しさん:2012/06/16(土) 20:54:42.73 ID:???
横からですまんのだけど、
スピン1/2は平行か反平行かしかないってのは分かるんだけど、
二つ以上のスピンを合成してスピン1とか3/2とかになっても平行と反平行しかないの?
921ご冗談でしょう?名無しさん:2012/06/16(土) 21:46:08.59 ID:Zkn7LGhT
>>919
電子の再差周波数と言ってるのは平行と反平行の遷移エネルギーだよ
>>920
スピン1なら横向きの自由度もある
922ご冗談でしょう?名無しさん:2012/06/16(土) 21:58:58.00 ID:???
>>921
じゃあ、
> 全合成スピン角運動量も平行または反平行になる
っていうのはなんで?
923ご冗談でしょう?名無しさん:2012/06/16(土) 22:01:34.39 ID:???
|+x>と|-x>だけから
|+y>=|+x>+|-x>
|-y>=|+x>−|-x>
|+z>=|+x>+i|-x>
|-z>=|+x>−i|-x>
ができるのは不思議ですねー
924ご冗談でしょう?名無しさん:2012/06/17(日) 00:02:43.14 ID:???
>>921
あっ……それはそうでした、ごめんなさい
じゃあx,y,z成分についてはどうなるのでしょうか
>>923さんが書かれているように一変数の線型結合で書けるから、
自由度1で平行か反平行、という解釈で良いのかな…うーん
925ご冗談でしょう?名無しさん:2012/06/17(日) 00:03:05.79 ID:???

レポートに出てきたんだが
調和近似って何ぞ?
926回答お願いします:2012/06/17(日) 00:06:09.93 ID:L/mNk1/0
大学受験生です。

http://iup.2ch-library.com/i/i0663594-1339857801.jpg

画像のように、質量mの小物体が、水平面からの高さがhの点Pから初速ゼロで滑り降りて行きます。
点O1,O3はそれぞれ点B,Fの直上Rの位置にあり、点O2は点Dの直下rの位置にあります。また、弧ABCは
円O1の円弧の一部、弧CDEは円O2の円弧の一部、弧EFは円O3の円弧の一部で、円O1と円O2は点Cで、
円O2と円O3は点Eで接しています。従ってO1CO2,O2EO3はそれぞれ一直線上にあります。∠O3O1Pをθ1,
∠BO2Pをθ2とします。曲面はなめらかにつながっており、摩擦は考えません。


このとき、弧ABCでの垂直抗力Nは

N=(mv^2)/R+mgsinθ1--------@

弧CDEでの垂直抗力Nは

N=-(mv^2)/r+mgsinθ2-------A

927回答お願いします:2012/06/17(日) 00:06:43.72 ID:L/mNk1/0
【続き】

と表されるのですが、小物体が点Fまで斜面から離れずに運動するとき、点C(sinθ1=sinθ2)
における垂直抗力はどちらの式で見ればよいのでしょうか。なぜこのようなことを思ったのか
というと、『ある点Xにおいて、物体が面に沿って運動しているとき、点Xにおける垂直抗力の
大きさはただ一つに定まる』のは自明であるにも拘わらず、点Cにおいては、

@,A,sinθ1=sinθ2=R/(R+r)より、

(v^2)/R=-(v^2)/r

となり、v=0以外では絶対に成り立たない式が出てきてしまったからです。



点Cでの垂直抗力はどのように考えればよいのでしょうか。


回答お願いします><
928ご冗談でしょう?名無しさん:2012/06/17(日) 00:09:09.18 ID:???
>>925
ポテンシャルを調和振動子で近似することか、平衡点のまわりの近似
http://cphys.s.kanazawa-u.ac.jp/~oda/butsuri-jikken-text/node6.html
929ご冗談でしょう?名無しさん:2012/06/17(日) 00:16:19.56 ID:???
>>928

要するにテイラー級数で近似するんだよな

さんくす
930ご冗談でしょう?名無しさん:2012/06/17(日) 00:17:07.98 ID:???
教科書に載せたいでやってたけど一兆分の一秒を捉えるカメラってすげーなおい
懐中電灯を一瞬点けると玉になって進む光の動きがわかるぞ
それと金属球を互いに当てると原子同士が絡み合う映像とかやってたな
931 忍法帖【Lv=21,xxxPT】 :2012/06/17(日) 03:23:14.22 ID:???
ワープ技術とか超光速航法とかワームホールテクノロジーを可能にする物理学の理論を作るのは可能ですか?。
932ご冗談でしょう?名無しさん:2012/06/17(日) 03:53:23.86 ID:???
野球でのスローボールはどのくらいまで遅くできるのでしょうか?
重量141.7-148.8gのボールが、18.44m先へ到達するための最低限の速度が知りたいです
人間には不可能でも機械などを使った場合で結構です
時速何キロになるのかわかるのでしょうか
933ご冗談でしょう?名無しさん:2012/06/17(日) 04:04:52.25 ID:???
>>932
空気から受ける力(抵抗・マグナス力)まで考慮しなくて良いなら、45度での投射が距離最大になるので、
45度でその距離飛ぶ速度を求めればよい。
934ご冗談でしょう?名無しさん:2012/06/17(日) 04:12:36.06 ID:???
>>926
θ1・θ2の定義とか変に思えるけど、それはともかく
点C・点Eにおいては斜面から受ける力は不連続になるので、二つの式が一致しないのは当然じゃない?
935ご冗談でしょう?名無しさん:2012/06/17(日) 04:58:15.37 ID:???
>>932
ストライクゾーンまでの到達時間を遅らせるだけなら
めちゃくちゃ高く放り投げればいくらでも遅く出来る

球速を小さくしたいなら>>933のやり方でほぼよいと思う
時速68キロメートルくらいかな

リリースポイントが高ければもう少し遅くできるかもしれんが
ハエは止まる速さは無理かな
936ご冗談でしょう?名無しさん:2012/06/17(日) 09:38:29.74 ID:Pt3lsbSh
場の量のは、過去スレで解いといたよ。
937ご冗談でしょう?名無しさん:2012/06/17(日) 10:39:19.15 ID:???
俺がよく理解してないだけなんだと思うが
量子非破壊測定ってもしかして理論上は量子論から確率を排除出来るってことを意味してる?
938ご冗談でしょう?名無しさん:2012/06/17(日) 10:40:47.81 ID:???
>>935
岩田だが、呼んだか?
939ご冗談でしょう?名無しさん:2012/06/17(日) 11:14:10.98 ID:Pt3lsbSh
量子測定は、理論上は量子論から確率を排除出来るってことを意味してる
940ご冗談でしょう?名無しさん:2012/06/17(日) 11:45:54.63 ID:Pt3lsbSh
確定事項がなければ、物質は成立しない
941 忍法帖【Lv=40,xxxPT】 :2012/06/17(日) 11:55:55.17 ID:???
cpaってなんだかわからん
簡単に説明しやがれ
942ご冗談でしょう?名無しさん:2012/06/17(日) 13:41:35.32 ID:Pt3lsbSh
確定事項がなければ、物質は成立しない、そうしなければ、方向性も、ベクトルも成り立たないだろ。
943ご冗談でしょう?名無しさん:2012/06/17(日) 13:43:49.53 ID:Pt3lsbSh
登記
944ご冗談でしょう?名無しさん:2012/06/17(日) 13:44:24.91 ID:Pt3lsbSh
確定事項がなければ、物質は成立しない、そうしなければ、方向性も、ベクトルも成り立たないだろ。登記
945ご冗談でしょう?名無しさん:2012/06/17(日) 15:04:39.81 ID:???
>>935
45度で18.44mなら48キロじゃないんですか
946ご冗談でしょう?名無しさん:2012/06/17(日) 15:24:35.08 ID:???
殺 伐 と し た ス レ に あ っ ち ゃ ん が 降 臨 !
    \     ヽ    |     /     /
      \                  /
       \    ;;;--‐''''::::::::::::::::::ヽ        _,,−''
          /:::::::::::::::::::::::::::::::::::::::::::、    _,,−''
`−、、      /::::::::::::::::::::::::::::::::::::::::::::::::::::、        _,,−''
    `−、、 i':::::::::::::::::/、::::::::::::::::::::::::::::::::::::i         
         |::::::::::::::/  :、:::::::::::::::::::::::::::::::::|  
         |::::::::;/ ‐─ ヽ─ヾ::::::::::::::::::::|   なんでやねん
────   .|:::::i'   ヾ●) (●ノ   `i:::::::|  
         .゙:、:|    "" ノ 、 ゙゙     |:::/
            |      (__)     |     
    _,,−'     .i    ^t三三テ'    ,!   `−、、
_,,−''         ヽ、          ノ       `−、、
        .     \___    ___/
                 | ̄ ̄|     
(省略されました・・全てを読むにはここを押してください)
947ご冗談でしょう?名無しさん:2012/06/17(日) 15:39:52.81 ID:???
>>932
ボールの質量によってどう違いがでるのか計算して調べてみると物理の面白さに目覚めるかもしれないね。
948ご冗談でしょう?名無しさん:2012/06/17(日) 16:26:30.12 ID:???
光電効果の,ミリカンの実験について,お聞きしたいことがあります。
概要については
http://ja.wikipedia.org/wiki/%E5%85%89%E9%9B%BB%E5%8A%B9%E6%9E%9C
に乗っています。
ここで,もし、 eV > Ek ならば、光電子は陽極に到達することができなくて電流は流れない。
とあるのですが,なぜ陽極にたどり着くことができないんでしょうか?
949ご冗談でしょう?名無しさん:2012/06/17(日) 17:08:33.44 ID:???
>>948
電子が陽極に行くのを妨害する電圧が掛かってるから
950ご冗談でしょう?名無しさん:2012/06/17(日) 17:25:22.32 ID:???
>>949
ありがとうございます
951 忍法帖【Lv=22,xxxPT】 :2012/06/17(日) 19:15:32.76 ID:???
物理学の発達は楽しいですよ。
952ご冗談でしょう?名無しさん:2012/06/17(日) 19:26:24.25 ID:???
すみませんやっぱりわからなくなりました。
陽極の方が電位が低いんですよね?
じゃあそっちのほうにむしろ電子は集まりませんか?
953ご冗談でしょう?名無しさん:2012/06/17(日) 19:58:20.51 ID:???
滑車を糸がすべってるときの動摩擦って、どうやって計算するんですか?
954ご冗談でしょう?名無しさん:2012/06/17(日) 20:20:45.74 ID:???
>>953
その文言からでは何とも言えんな。
実際に計測するとなったら、滑車の完成モーメントと角加速度計測して計算するな。
(必要なら軸受けの抵抗も)
955ご冗談でしょう?名無しさん:2012/06/17(日) 20:40:04.01 ID:???
単純に物理の問題として、

質量mとMの重りを、おもさのない伸びちじみしない糸で連結して慣性モーメント I の滑車に吊るしたら、
重りの運動とともに糸が滑りながら滑車を回転させた、というような問題を解くばあい。
956ご冗談でしょう?名無しさん:2012/06/17(日) 20:50:28.92 ID:???
>>955
それこそ単純にIω'/r
957ご冗談でしょう?名無しさん:2012/06/17(日) 21:03:27.51 ID:???
ω’を求めるのに動摩擦が必要ですけど。
958ご冗談でしょう?名無しさん:2012/06/17(日) 21:20:08.48 ID:???
>>957
ω'は計測値、無けりゃ(もしくは他の拘束条件でもなけりゃ)動摩擦なんて解る訳無い。
959ご冗談でしょう?名無しさん:2012/06/17(日) 21:31:32.62 ID:???
でも直感的には決まってるはずだと思うんですよね。動摩擦係数にしたがって。
960ご冗談でしょう?名無しさん:2012/06/17(日) 21:47:32.55 ID:???
>>959
(滑車に糸が掛けられた状態での)動摩擦係数が既知なら何の問題も無い。
動摩擦係数が未知なら、運動を観察して計測するしかない。
観察も無しに動摩擦が出せるはずという直感ならそれは誤りだと思うな。
961ご冗談でしょう?名無しさん:2012/06/17(日) 22:07:14.15 ID:???
理論的に摩擦って出せないもんかね
962ご冗談でしょう?名無しさん:2012/06/17(日) 22:09:43.08 ID:???
>>960
動摩擦係数が決まってたらどうやって求めるんですか?
物理の問題ならたいがい決まってるものですが。
963ご冗談でしょう?名無しさん:2012/06/17(日) 22:13:55.63 ID:???
Rodriguesの公式で表したLegendre多項式で、
Pn(-z)=(-1)^n*Pn(z)
を示せって問題ができません
コーシーの微積分公式をすぐ前の問題で使っているので、それを使うと思います。

ちなみに、コーシーの微積分公式を使って表した形がこれです
http://beebee2see.appspot.com/i/azuYytrSBgw.jpg
964ご冗談でしょう?名無しさん:2012/06/17(日) 22:14:25.87 ID:???
>>962のつづきですけど、

多分垂直抗力は場所によってちがうから場所場所の垂直抗力Rを求めてμRを積分するんでしょうけど、
どうやっていいかわからなくなってしまって。

でも決まりものっぽいから、何か公式がありそうだなという感じもします。
965ご冗談でしょう?名無しさん:2012/06/17(日) 22:20:09.07 ID:???
>>963
zに-z入れればいいんじゃねーの
それ試して出来なかったんなら知らんわ
966ご冗談でしょう?名無しさん:2012/06/17(日) 22:22:20.76 ID:???
>>963
両辺の留数を計算すればいいだろう
967ご冗談でしょう?名無しさん:2012/06/17(日) 22:23:58.02 ID:???
>>965
それは試しましたが、式変形がうまくできませんでした…
968ご冗談でしょう?名無しさん:2012/06/17(日) 22:26:40.46 ID:???
>>966
留数ですか…ちょっと試してみます!
ありがとうございます!
969ご冗談でしょう?名無しさん:2012/06/17(日) 22:47:08.20 ID:???
>>963ですが、留数定理を使ってもコーシーの微積分公式に戻るだけでした…
970ご冗談でしょう?名無しさん:2012/06/17(日) 22:52:02.39 ID:???
>>969
> 両辺の留数
右辺は(-1)^nを除いたやつ
971ご冗談でしょう?名無しさん:2012/06/17(日) 22:55:49.42 ID:???
>>970うpした、積分で表した形の式を留数定理で計算するであってますよね?
972ご冗談でしょう?名無しさん:2012/06/17(日) 22:56:38.14 ID:???
>>971
うんだ
973ご冗談でしょう?名無しさん:2012/06/17(日) 23:08:31.17 ID:???
>>972やっぱり、留数自体がRodriguesの公式に戻ってしまいます…
974ご冗談でしょう?名無しさん:2012/06/17(日) 23:10:16.24 ID:???
>>973
左辺の留数を求める式を書け
975ご冗談でしょう?名無しさん:2012/06/17(日) 23:14:18.00 ID:???
>>974
すいません、文字を変えてます。
上の画像のs→zでz→aにしてます
n+1位の極だと思うので、これだと思います。
http://beebee2see.appspot.com/i/azuYq47UBgw.jpg
976ご冗談でしょう?名無しさん:2012/06/17(日) 23:19:31.43 ID:???
977ご冗談でしょう?名無しさん:2012/06/17(日) 23:23:14.37 ID:???
>>976
あ、ごめんなさい、間違えました。画像は留数定理の方でした。
978ご冗談でしょう?名無しさん:2012/06/17(日) 23:27:21.19 ID:M5Ca1re8
ファインマン経路積分って何ですか?
979ご冗談でしょう?名無しさん:2012/06/17(日) 23:29:02.32 ID:???
ggrks
980ご冗談でしょう?名無しさん:2012/06/17(日) 23:32:28.05 ID:???
また元に戻りました…
981ご冗談でしょう?名無しさん:2012/06/17(日) 23:42:39.45 ID:FFImLDPR
反発係数に付いて教えていただきたい
反発係数を e
衝突前のそれぞれの物体の速度を v1 、v2 、
衝突後のそれぞれの物体の速度を v1' 、v2' とすると
e = - (v2' - v1')/(v2 - v1)
で表せるとなるが、v1=v2の場合はどの様に求められるのでしょうか?
高校の時にも悩んだ記憶があるが答えを思い出せない...
982ご冗談でしょう?名無しさん:2012/06/17(日) 23:46:14.50 ID:???
>>980
頭冷やしてから考えたら
983ご冗談でしょう?名無しさん:2012/06/17(日) 23:47:27.93 ID:???
>>982
そうします。ありがとうございました
984ご冗談でしょう?名無しさん:2012/06/17(日) 23:48:08.95 ID:???
v1'とv2'はそれぞれv1, v2の関数として書けることが期待されるはずだろ?
だから e = -lim[v2→v1] { v2'(v1,v2) - v1'(v1,v2) } / { v2 - v1 } とでも定義すればいいんじゃないか?
985982:2012/06/17(日) 23:57:06.56 ID:???
>>983
極が違うけど

おやすみ
986ご冗談でしょう?名無しさん:2012/06/18(月) 00:01:43.48 ID:???
(v2' - v1') = -e(v2 - v1) でいいじゃん
987ご冗談でしょう?名無しさん:2012/06/18(月) 00:07:45.84 ID:???
>>984
ありがとうございます
教わった時点で微分や極限とか習ってなかったのかもしれない
今では解くこと出来ないので勉強し直す必要があるが
当時も見なかった事にしたような気もする

>>986
それでは常に0になって解けないと思うのですが
988ご冗談でしょう?名無しさん:2012/06/18(月) 00:10:59.47 ID:???
>>987
横だけど
反対方向から同じ測度で衝突すると「くっついて」終わり
989ご冗談でしょう?名無しさん:2012/06/18(月) 00:17:39.01 ID:???
v1=v2とは衝突が起きないということだと思うんだが
990ご冗談でしょう?名無しさん:2012/06/18(月) 00:33:47.07 ID:Es0miIzE
>>988
くっつかない場合はないのでしょうか?
常にくっつくのが考えづらいので

>>989
反対方向から同じ速度で衝突で跳ね返る場合を考えてました
991ご冗談でしょう?名無しさん:2012/06/18(月) 00:38:39.56 ID:???
>反対方向から同じ速度で
v1=-v2
992988:2012/06/18(月) 00:39:19.54 ID:???
>>990
同じ方向から同じ速度で動くので980だな
993ご冗談でしょう?名無しさん:2012/06/18(月) 00:50:37.51 ID:???
申し訳ありません
v1=-v2
ですね
お騒がせしました
基本がためでした
994 忍法帖【Lv=11,xxxPT】 :2012/06/18(月) 02:34:44.34 ID:HZQEDdcP
外力同士の問題だけど男女が性行為してるとき男の方が腰降ってる場合女にはやっぱり反作用くるよね
995ご冗談でしょう?名無しさん:2012/06/18(月) 02:35:00.16 ID:???
>>955
結局これはわからないんですか?
結構ありがちっぽい感じなんですけど。
996ご冗談でしょう?名無しさん:2012/06/18(月) 10:02:20.71 ID:???
まず、動摩擦も運動も抜きで、固定された円筒に糸がかかっていて、その両端が地面に結びつけられたモデルを考え、
このモデルにおいて円筒が糸の各点に及ぼす抗力の分布を考える

最初から糸が伸び縮みしないモデル考えたら、どう考えても決定不能だろ
未知変数は抗力の連続場だから無限個あるのに対し、式は水平・鉛直のつりあいの2つしか立たない

現実的な解法としては、伸び縮みする糸を考え、
つりあいの条件を満たしつつ弾性エネルギーを最小にする分布を考えることだろうが、
面倒だから計算は自分でやれ
997 忍法帖【Lv=40,xxxPT】 :2012/06/18(月) 11:16:46.70 ID:???
エターナルスピリチュアリズムエボリューション!?♪。
998ご冗談でしょう?名無しさん:2012/06/18(月) 13:10:11.29 ID:???
     l   :::::::::::;: -―‐- 、::::::::::::_,, ==-=、l
     { (_ } : : : =l-  ィo  v-‐v'´ ィo.  :-}=          なんやあああああああああ!!!
         ¨ : : : : :、     ノ/  ヾ、    : /
      ゙、_ :. . . . . . ̄¨¨´/  ヽ `ー--:‐′
         /;.. . . . ./(,o、  ,,ゝ ...:::/
        /   ',. . .   ′      `.::::::/
,.==ニ三/  / 〉、 :    ,.-‐―ァァ / /l三=-__
三三三,'  ,' : :ヽヽ:    ヾェェェイノ ノ .ノ l三三三三三 =-
999ご冗談でしょう?名無しさん:2012/06/18(月) 13:54:08.83 ID:???
台     風
1000ご冗談でしょう?名無しさん:2012/06/18(月) 14:21:40.40 ID:???
comming soon
10011001
このスレッドは1000を超えました。
もう書けないので、新しいスレッドを立ててくださいです。。。